Download as pdf or txt
Download as pdf or txt
You are on page 1of 89

INDEX FOR STUDY QUESTIONS

Subject

Pages

Number questions

General principles (optional) 1 4
Cell injury 2 4
Inflammation 2-3 9
Fluids and hemodynamics 3-7 27
Nutrition 8-9 11
Genetics (optional) 10-11 8
Neoplasia 11-12 8
Hematology: RBC/WBC 13-21 38
Lymphoproliferate 21-22 8
Coagulation 23-25 14
Blood bank 25-29 16
Cardiovascular 29-31 13
Respiratory 30-34 17
Gastrointestinal 35-38 21
Hepatobiliary Ipancreas 38-41 16
Renal/LUT 41-44 14
Gynecology 45-48 17
Endocrine 49-53 22
Musculoskeletal 53-58 20
Skin 58-62 20
CNS and special senses 62-69 35
342
Answers and explanations 69---88 I

Note: This material is copyrighted. All rights reserved. (Edward F. Goljan, M.D.) 2001

\

HIGH YIELD STUDY QUESTIONS IN PATHOLOGY

Total questions: 328

Optional: General principles in laboratory medicine questions: this material is covered in the Pathology High Yield pages 5-10

1. Assuming the use of 2 standard deviations to establish the reference interval of a test, in a test with a reference interval of 10-30 mgldL, 1 standard deviation would equal. ..

A. 2.5

B. 5.0

C. 7.5

D. 10.0

E. 20.0

2. If the prostate specific antigen (PSA) test for prostate cancer is lowered from a reference interval of 0-10 ng/ml, to 0-4 ng/ml., this will ...

A. increase the number of false negatives

B. decrease the number of false positives c. increase the test's specificity

D. increase the PV-

E. increase the PY+

3. Study the following schematic involving a control group and disease X.

Normal Disease X

D

Reference interval

Which of the following correctly describes test results in the space occupied by each of the lettered groups?

A. Group A: true negatives + false negatives

B. Group B: true negatives + false positives

C. Group C: true positives + false positives

D. Group D: true positives + false negatives

4. A pregnant woman in her first trimester complains of heat intolerance and palpitations. Physical exam reveals an enlarged, non-tender tbyroidgland. Her serum T4 is elevated and the TSH is normal. Which of the following best explains the thyroid function study results?

A. Thyroid binding globulin is increased

B. Free T4 hormone levels are increased

c. Estrogen increased the synthesis of thyroid hormone D.Progesterone increased the synthesis of thyroid binding globulin

E. Patient has Grave's disease

1

.- --- ... ...-. '~-..

Note: This material is copyrighted. All rights reserved. (Edward F. Goljan, M.D.) 2001

Cell Injury questions

1. Hyperplasia is primarily operative in which of the following growth alterations?

A. Appearance of the affected kidney in renovascular hypertension

B. Thickened bladder wall in a patient with urethral obstruction

C. Barrett's esophagus in a patient with gastroesophageal reflux

D. Enlarged left atrium in a patient with severe mitral stenosis

E. Galactorrhea in a woman with a prolactinoma

2. Which of the following disorders is an example of coagulation necrosis?

A. Lobar pneumonia in an alcoholic

B. Hepatic abscess in a patient with amebiasis

C. Pseudomembranous colitis ina patient on ampicillin

D. Diminished brain mass in a patient with Alzheimer's disease

E. Embolus to the superior mesenteric artery leading to bowel infarction

3. In which of the following diseases would you expect a low arterial P02 and a low oxygen saturation (Sa02)?

A. Carbon monoxide poisoning

B. Iron deficiency anemia

C. Decreased cardiac output

D. Respiratory acidosis

E. Cyanide poisoning

4.

Which of the following disorders is an example of metaplasia?

A. Increased goblet cells in the mainstem bronchus of a smoker

B. Squamous epithelium in the mainstem bronchus of a smoker

C. Proliferative endometrial glands in a woman on unopposed estrogen

D. Hyperkeratosis of the skin in a patient with psoriasis

E. Multinucleated giant cells in a granuloma

-,

)

Inflammation questions

1. A 4-year-old child with recurrent Staphylococcus aureus infections and an absent respiratory burst MOST LIKELY has alan ...

A. defect in spectrin in the cell membrane

B. defect in microtubule polymerization c. deficiency ofIgG gamma globulins

D. deficiency ofNADPH oxidase

E. deficiency of rnyeloperoxidase

2. Which of the following is MOST responsible for the anti-inflammatory activity of corticosteroids?

A. Increased leukocyte adhesion to endothelial cells

B. Inhibition of phospholipase AZ

C. Destruction of eosinophils

D. Inhibition of cyclooxygenase

E. Inhibition oflipoxygenase

2

,'-----------------------

Note: This material is copyrighted. All rights reserved. (Edward F. Goljan, M.D.) 2001

Items 3-5

A. Cellulitis

B. Suppurative inflammation

C. Fibrinous inflammation

D. Pseudomembranous inflammation

E. Granulomatous inflammation

3. A 52-year-old man with chronic renal failure has chest pain and a scratchy 3 component sound heard over the anterior chest

4. A febrile 8-year-old child, who has not received any immunizations, has a grayish-white exudate in the oropharynx and prominent cervical lymphadenopathy

5. A febrile 3 year old child has a diffuse, raised, hot red area of inflammation extending around the right periorbital tissue leading to swelling and closure of the eye

6. A newborn child has failure of separation of the umbilical cord. Histologic sections of the surgically removed cord reveal an absence of neutrophil margination and emigration into the interstitial tissue. The clinical and histologic findings in this case are MOST CLOSELY associated with a defect in ...

A. activation of the complement system

B. microtubule polymerization

c. the respiratory burst mechanism

D. an adhesion molecule in neutrophils

E. the production of myeloperoxidase

Items 7-9

A. Neutrophils

B. Macrophages c. Eosinophils

D. Mast cells

E. Plasma cells

7. They represent the epithelioid cells and multinucleated giant cells in a granuloma

8. C3a and C5a activate these cells

9. The granules of these cells contain refractile material that form Charcot-Leyden crystals in the sputum of asthmatics

Fluids and bemodynamics questions:

1. Which of the following characterizes early endotoxic (septic) shock rather than hypovolemic or cardiogenic shock?

A. Warm skin

B. Decreased cardiac output

C. Low mixed venous oxygen content

D. Increased total peripheral resistance

E. Decreased venous return to the heart

3

Note: This material is copyrighted. All rights reserved. (Edward F. Goljan, M.D.) 2001

2.. Which of the following edema conditions represents a transudate secondary to an decrease in oncotic pressure?

A. Patient with pneumonia who has a pleural effusion

B. Patient with cirrhosis who has dependent pitting edema

C. Patient with edema of the arm post-modified radical mastectomy

D. Patient with a pulmonary infarction who has a left pleural effusion

E. Patient with congestive heart failure with bilateral pleural effusions

3. In treating a patient with right heart failure who has dependent pitting edema, which of the following would be the MOST APPROPRIATE management of the patient's sodium and water intake?

Sodium intake Water intake

A. No change Decrease

B. Increase Increase

C. Decrease No change

D. Decrease Decrease

E. No change No change

4. A 58-year-old man with a small cell carcinoma of the lung presents with mental status abnormalities, A cr scan of the brain reveals cerebral edema but no space occupying lesions a Serum electrolytes exhibit a serum sodium of 110 mEq/L (136-145 mEq/L). There is no evidence of pitting edema or volume depletion. Which of the following is the BEST non-pharmacologic treatment of this patient?

SodiUIDintake VVaterintake

A. B. c. D. E.

Decrease Increase Decrease No change No change

Decrease Increase No change Decrease No change

s. A 62-year-old man has a ruptured abdominal aortic aneurysm with retroperitoneal hemorrhage.

Which of the following Swan-Ganz catheter findings BEST represents this patient's clinical disorder?

PCWP

SVR

Cardiac output

A. Normal Normal Normal Normal

B. Decreased Increased Increased Decreased

C. Decreased Decreased Increased Decreased

D. Increased Decreased Decreased Decreased

MV02 = mixed venous oxygen content, PCWP = pulmonary capillary wedge pressure (measure of left ventricular end-diastolic volume), SVR = systemic vascular resistance (peripheral arteriolar resistance)

4

:..._------.---------------~-,,----- ----,---------------~

Note: This material is copyrighted. All rights reserved. (Edward F. Goljan, M.D.) 2001

Items 6-12

POsm

ICF

ECF

Volume

Each set of options relating to volume and osmolality changes in the extracellular and intracellular fluid compartments is followed by several numbered items. For each numbered item, select the ONE lettered option that is most closely associated with it Each lettered option may be selected once, more than once, or not at alL Note: the height of the squares represent plasma osmolality (POsm), while the width of the squares represents volume in each compartment. ECF is the extracellular fluid compartment and ICF is the intracellular fluid compartment. The dark lines represent the normal POsm and volume in each compartment, while the hash marked lines represent the volume alteration.

• • • • •

A.

•••••••••••••••••••••••••••••••••••• •











B

• ...

• •

.. •

............ ---- ..... _...._ ...

...----- -- " .

c.

• • • • • • •

...__--_ __ :

. " .

• •

• •

• e·

D · e

.: ;

: :

• e

• •

• •

• •

• •

• •

• •

• •

• •

• •

•••••••• • •••••

••••••••••••••••••••••••••••••••• •







: .

• •

· :

• •

.. • •

E.

F.

• • • • • • • • • • • •

------------ :

6. The patient is a 28 yr. old man who is volume depleted secondary to traveler's diarrhea contracted while on a vacation outside the United States

7. The patient is a 58 yr. old man with a long history of ischemic heart disease. He now presents with bibasilar rales and dependent pitting edema. He is not taking diuretics.

8. The patient is a 29 yr old man who recently was hospitalized with head trauma secondary to a motorcycle accident. He now has polyuria and complains of excessive thirst throughout the day. The physical exam is normal.

5

I

... __ "._' ,_'. __ . .-. _ -_ .-- _ . ---._ _._

Note: This material is copyrighted. All rights reserved. (Edward F. Goljan, M.D.) 2001

9. The patient is a 45 yr old man who received excessive amounts of sodium bicarbonate during a cardiac resuscitation. He now has bibasilar rales, neck vein distention, and dependent pitting edema.

10. A 79 yr old woman received excessive amounts of 0.9% normal saline during repair of a femoral head fracture. She now has bibasilar rales, neck vein distention, and dependent pitting edema.

11. A 52 yr old man has been taking a loop diuretic and now presents with signs of volume depletion.

12. A 29 yr old man collapsed after finishing a marathon on a hot, humid day. His nmning companion states that he did not take any fluid supplements and had been sweating profusely throughout the race. Physical exam reveals signs of volume depletion.

13. A volume depleted patient with type I diabetes mellitus presents to the emergency room in diabetic ketoacidosis. Which of the following changes in electrolyte and volume status is most likely present in this patient?

POsm PNa + ECF ICF

Compartment Compartment

)

A. High Low Contracted Contracted

B. High Low Expanded Contracted

C. High High Expanded Contracted

D. Low Low Expanded Expanded

E. Low Low Contracted Expanded

ECF :: extracellular fluid, ICF = intracellular fluid,PNa + = plasma sodium, POsm = plasma osmolality

14. Which of the following is a blood component that is utilized in both an occlusive venous thrombus and in an arterial thrombus?

A. FactorV

B. Platelets

C. White blood cells

D. FactorVIll E.Prothrombin

15. A 29 yr. old man sustains bilateral femoral bone fractures and multiple pelvic fractures from a mountain bike accident. Forty eight hours later, he develops a sudden onset of dyspnea, petechial lesions, and mental status alterations. Laboratory studies reveal hypoxemia and thrombocytopenia. The mechanism for this patient's clinical disorder is most closely associated with ...

A. B~ c. D. E.

disseminated intravascular coagulation pulmonary embolism

fat embolization

air embolization

.

pneumonia

6

Note: This material is copyrighted. All rights reserved. (Edward F. Goljan, M.D.) 2001

Items 16-21

A. B. c. D. E. F.

pH

(7.35-7.45) 7.22 7.26 7.33 7.42 7.51 7.56

PaC02

(33-45 nun Hg) 69

26

68

22

48

24

HC03"

(22-28 mEqlL) 27

11

34

14

38

21

16. A 56 year old patient who has been vomiting

17. A 51-year-old woman with rheumatoid arthritis has salicylate intoxication

18. Patient with an anxiety attack

19. Patient with ethylene glycol poisoning

20. Patient with cystic fibrosis

21. Patient with barbiturate poisoning

Items 22-27

\

\ )

Serum sodium Serum potassium Serum chloride Serum bicarbonate
(136-145) (3.5-5.0) (95-105) (22-28)
A. 118 3.0 88 21
B. 152 2.8 110 - 33
c. 130 2.9 80 36
D. 128 5.8 96 18
E. 140 3.0 115 15
F. 140 5.0 95 15 22. Patient with primary aldosteronism

23. Patient with Addison's disease

24. Patient with diarrhea

25. Patient with an increased anion gap metabolic acidosis

26. Patient with inappropriate ADH syndrome

27. Patient with excessive vomiting or patient taking loop diuretics who is volume depleted

7

Note: This material is copyrighted. All rights reserved. (Edward F. Goljan, M.D.) 2001

Nutrition questions

1. A 25-year~ld woman has not had her period for the last 8 months. She is 5' 2" and weighs 90 pounds. A wine pregnancy test is negative. She states that she has been trying to lose weight for her upcoming wedding. You order a battery of tests and give the patient an intramuscular injection of progesterone. Ten days later the patient returns to your office and reports that she had no withdrawal bleeding. Laboratory tests reveal the following: serum prolactin is normal, serum FSH and LH are low, serum TSH is normal, serum estradiol is low, and serum cortisol and growth hormone are increased .. Based on these findings, you strongly suspect the patient has ...

A. primary ovarian disease

B. hypopituitarism

C. secondary hypothyroidism

D. weight loss syndrome

E. Cushing's syndrome

2. Follicular hyperkeratosis, night blindness, and a hemorrhagic diathesis are expected in a patient ...

A. with cystic fibrosis

B. with scurvy

C. with hypothyroidism

D. who is a pure vegan

E. who is bulimic

3. Which of the signs or symptoms characterize a fat soluble rather than a water soluble vitamin deficiency?

A. Perifollicular hemorrhage

B. Bone pain and tetany

C. Peripheral neuropathy

D. Ophthalmoplegia, confusion, and ataxia

E. Hyperpigmentation in sun-exposed areas

4. Which of the signs or symptoms are more prominently found in marasmus than kwashiorkor?

A. Hepatomegaly

B. Pitting edema

C. Reduced total lymphocyte count

D. Flaky paint dermatitis

E. Broomstick extremities

5. Pellagra will MOST LIKELY develop in a patient ...

A. who is taking isoniazid

B. whose diet primarily consists of com

C. who is a pure vegan

D. who is taking nicotinic acid to lower lipids

E. with maldigestion secondary to chronic pancreatitis

8

Note: This material is copyrighted. All rights reserved. (Edward F. Goljan, M.D.) 2001

6.

7.

8.

\

)

Which of the following is more often associated with anorexia nervosa than bulimia nervosa?

A. Hypokalemia

B. Metabolic alkalosis

C. Normal body image

D. Normal serum gonadotropins

E. Osteoporosis

Which of the following laboratory test abnormalities would you MOST expect in a patient with morbid obesity?

A. Increased serum TSH

B. Increased 24-hr urine for free cortisol

C. Increased fasting glucose

D. Increased 24-br urine for 17-ketosteroids

E. Increased serum DHEA-sulfate

Which of the following vitamin deficiencies would you expect in a child maintained on unfortified goat's milk?

A. Ascorbic acid

B. Thiamine

C. Niacin

D. Riboflavin

E. Folate

Which of the following vitamins would be deficient in a newborn child with anemia whose mother is a pure vegan?

A. Ascorbic acid

B. Thiamine

C. Niacin

D. Pyridoxine

E. B12

10. A 65-year-old woman complains of bleeding gums after brushing her teeth, easy bruising, and pain in her legs when walking .. Her platelet count is normal. The pathogenesis of her disease is MOST CLOSELY related to ...

A. a deficiency of ATP

B. a cofactor deficiency in collagenase

C. lack of hydroxylation of lysine and proline

D. a cofactor deficiency in lysyl oxidase

E. platelet dysfunction

11. A 30 year-old man develops an acute onset of confusion, ataxia, nystagmus, and ophthalmoplegia shortly after the administration of an intravenous solution containing 5% glucose and normal saline. Thepatbogenesis of this patient's neurologic disorder is most closely related to ...

A. central pontine myelinolysis

B. thiamine deficiency

C. Purkinje cell atrophy

D. viral encephalitis

E. B12 deficiency

9

I

1. A 51 year old woman delivers a full-term baby that has repeated vomiting of bile stained material.

A flat plate of the abdomen reveals air in the stomach and proximal duodenum and no air in the remainder of the bowel. The maternal serum a-fetoprotein level is low. The baby has 46 chromosomes. The mechanism of the child's disease is most closely associated with ...

A. a Mendelian disorder

B. a Robertsonian translocation

C. nondisjunction in meiosis

D. a point mutation ofanucleotide

E. a microdeletion disorder

Note: This material is copyrighted. All rights reserved. (Edward F. Goljan, M.D.) 2001

Optional Genetics questions: most of the genetics material is located in Pathology High Yield (pages 11-16). The material will be covered by a geneticist.

2. If an African American woman with sickle cell disease has children with a man lacking the abnormal ~-chain, you would expect ...

A. 25% of their children to have sickle cell trait

B. 50% of their children to have sickle cell trait

C. 25% of their children to have sickle cell disease

D. 50% of their children to have sickle cell disease

E. all of their children to have sickle cell trait

3. A 17-year-old adolescent presents with primary amenorrhea. Physical exam reveals Donna! secondary female sex characteristics. Discrete masses are noted in both inguinal canals. A speculum exam of the vagina indicates a blind pouch. You would expect this patient to also have ...

A. a prostate gland

B. seminal vesicles

C. an androgen receptor deficiency

D. one Barr body on a buccal smear

E. ovaries in the inguinal canal

4. While examining a 13 year old boy during a routine physical examination, you note bilaterally enlarged, non-tender testicles that do not transilluminate, a high arched palate, and a mid-systolic ejection click followed by a short murmur. You call the school counselor and find that the child has a moderately severe attention deficit syndrome. Which of the following studies would you reconunend on this boy that would best explain all of the abnormalities noted on the examination?

A. Echocardiogram

B. Buccal smear

C. Serum gonadotropins

D. Identification of triplet repeat

E. Chromosome study on his father

s. Prader-Wil1i and Angelman's syndrome have different clinical features, however they both share a defect at the same location on the same chromosome. This is an example of ...

A. variable expressivity

B. a Robertsonian translocation

C. genetic heterogeneity

D. genomic imprinting

E. a balanced translocation

10

Note: This material is copyrighted. All rights reserved. (Edward F. Goljan, M.D.) 2001

6. If the carrier rate for the sickle cell abnormality is 1 in 12, the prevalence of sickle cell disease is approximately I in. e._

A. 144

B. 288 c. 576

D. 720

E. 1440

7. Which one of the following transplacental infections is associated with sensorineural hearing loss, blindness, and periventricular calcifications?

A. Cytomegalovirus

B. Toxoplasmosis

C. Syphilis

D. Herpes genitalis

E. Rubella

8. A pregnant woman during her first trimester developed fever, a maculopapular rash, arthritis, and painful postauricular lymphadenopathy. Which of the following complications could potentially occur in her newborn child?

A.Periventricular calcification

B. Saddle nose deformity

C. Sensorineural hearing loss

D. Limb hypoplasia

E. Craniofacial abnormalities

\

)

Neoplasia questions

1. In which of the following sites is the MOST COMIvION primary cancer an adenocarcinoma?

SELECT 2

A. Esophagus

B. Bladder

C. Larynx

D. Cervix

E. Lung

2. Which of the following genes regulates kinases in the cell division cycle, hence assuming an important role in the development of human cancer?

A. e-ras proto-oncogene

Bo erb B proto-oncogene

c. Rb-l suppressor gene

D. c-myc proto-oncogene

E. p-53 suppressor gene

3. A primary cancer is more common than metastasis in which of the following sites?

A. Lymph node

B. Colon

C. Brain

D. Bone

E. Lung

11

---- ,&

.i.' V ce , 1 rus materia. l~ copyrighted. 1\11 TIghts reserved. (Edward F. Goljan, M.D.) 2001

4. A 55 year old woman with breast cancer has an infiltrating ductal carcinoma that is 2 em in size, ERA and PRA positive, metastatic to 5 out of 20 axillary lymph nodes, and metastatic to both the vertebral column and liver. Which of the following MOST influences the ultimate prognosis in this

. ? patIent.

A. Her age

B. ERAIPRA status c. Size of the tumor

D. Axillary node involvement

E. Bone and liver involvement

s. Which of the following is the second most common cancer and cancer killer in men and women?

A. Malignant melanoma

B. Lung cancer

C. Colorectal cancer

D. Malignant lymphoma

E. Stomach cancer

,6. A tumor that could potentially produce Cushing's syndrome and hyponatremia is most likely located in the ...

A. kidney

B. placenta c. liver

D. thyroid

E. lung

7. Both AFP and l3-hCG are most likely to be elevated in which primary tumor site?

A. Lung

B. Testicle

C. Liver

D. Colon

E. Ovary

8. Which of the following cancers is prevented by immunization with a commonly used vaccine?

A. Pancreatic carcinoma

B. Stomach carcinoma

C. Transitional cell carcinoma

D. Hepatocellular carcinoma

E. Cervical carcinoma

12

Note: This material is copyrighted. All rights reserved. (Edward F. Goljan, M.D.) 2001

Hematology questions

1. A 19-year-old African American woman presents with fatigue and exercise intolerance. She has a history of menorrhagia. A CBC reveals a mild microcytic anemia, a low normal WBC count, and a normal platelet count. A corrected reticulocyte count is <2%. Which of the following sets of

labo da b t th h t 1 . fi di . th· ti t?

A. B. c. D. E.

ratory ta est represen s e ema 0 Ogle n mgs In IS pa en .
Serum Iron TIBC % Saturation RBCCount Ferritin
normal normal normal high normal
I low low low low high
low high low low low
high low high normal high
I normal normal normal low normal 2. Which of the following characterizes pernicious anemia rather than B12 deficiency from other causes?

A. Increased plasma homocysteine levels

B. Increased urine methylmalonic acid

C. Decreased vibratory sensation

D. Hypersegmented neutrophils

E. High serum gastrin levels

3. A 28-year-old man has a motorcycle accident outside the emergency room. Physical exam in the emergence room reveals a weak, thready pulse, cold, clammy skin, and a blood pressure of 60/40 mm Hg, An open right femoral fracture is present as well as tenderness over the left lower ribs. Which of the following would you expect in this patient if a blood sample was drawn prior to insertion of an IV line with 0.9% normal saline? SELECT 3

Ae Normal hemoglobin and hematocrit

B. Normal effective arterial blood volume

C. Increased central venous hydrostatic pressure

De Decreased mixed venous oxygen content

E. Decreased pulmonary capillary wedge pressure

4. A 4 year old African American child with sickle cell anemia presents with a high fever and nuchal rigidity. Scleral icterus and hepatosplenomegaly are present. A CBC reveals a moderately severe

normocytic anemia and a WBC count of 21,000 cells/ul, with left shift A urinalysis is normal. A spinal tap reveals turbidity, increased protein, and neutrophils. You would expect a gram stain of CSF and a blood culture to reveal, ..

A. gram positive diplococci

B. gram negative diplococci

C. gram positive cocci

D. gram positive rods

E. gram negative coccobacilli

13

Note: 1 hIS material is copyrighted. All rights reserved. (Edward F. Goljan, M.D.) 2001

5. A febrile 23 year old college coed presents with fatigue and difficulty with swallowing. Physical exam reveals exudative tonsillitis, palatal petechia, cervical lymphadenopathy, and tender hepatosplenomegaly. A CBC reveals a mild microcytic anemia, lymphocytic leukocytosis with - 20% of the lymphocytes exhibiting atypical features, and a mild thrombocytopenia. You would expect this patient to have ...

A. a lowTIBC

B. a normal serum ferritin

C. an elevated total bilirubin

D. heterophile antibodies

E. normal serum AST and AL T titers

6. A 65-year-old man with an elevated RBC count has a history of gout, frequent headaches, and pruritus after bathing. He presents with a sudden onset of abdominal pain and bloody diarrhea.

WhO h fth f 11' ts fl bo d ta be t t thi ti t' hematologic disease?

IC 0 e 0 OWIng se 0 a ratory a s ·represen sp_aens
RBCMass Plasma Volume Sa02 Erythropoietin
A. Increased Normal Decreased Increased
B. Increased Normal Normal Increased
c. Increased Increased Normal Low
D. Normal Decreased Normal Normal Sa02 = oxygen saturation

)

7. A 28 year old woman presents with fever, scleral icterus, and crampy right upper quadrant pain.

Physical exam reveals tenderness to palpation in the right upper quadrant and mild splenomegaly. A hemogram exhibits a total leukocyte count of 23,OOO/rnm3, hemoglobin of 10.0 g/dl, (12.0-16.0 g/dl.), a corrected reticulocyte count of 6% (0.5-1.5%), a mean corpuscular volume (MeV) of 80 fJ.ffi3 (80-100 um'), and a mean corpuscular hemoglobin concentration (MCHC) of 38% (31-36% Hb/cell), The total bilirubin is 3.2 mgldL (0.1-1.0 mgldL) with a direct (conjugated) fraction of 0.4 mgldL (0.0-0.3 mg/dl.). Serum alanine aminotransferase is 20 UIL (8-20 UIL). An ultrasound reveals numerous stones in the gallbladder. The common bile duct is not dilated. Based on these findings, which of the apply to this case? SELECT 3

A. Increased RBC osmotic fragility

B. Calcium bilirubinate stones

C. Low serum ferritin

D. RBCs with a defect in spectrin

E. Positive direct Coombs'

F. Decreased unconjugate bilirubin

8. A significant number of employees in a pottery factor develop abdominal colic and diarrhea. In addition, a few of the employees complain of burning feet and muscle weakness in the upper and lower extremities. Based on these fmdings, which of the following are expected laboratory findings in these patients? SELECT 4

A Normocytic anemia

B Coarse basophilic stippling

C Low serum ferritin

D Increased blood Pb levels

E Increased RBC protoporphyrin levels

F. Increased plasma homocysteine levels

G. Ringed sideroblasts

14

------------------- ...

Note: This material is copyrighted.All rights reserved. (Edward F. Goljan, M.D.) 2001

9. A 33 year old African-American medical missionary, who recently returned from a 2 year tour in India, is diagnosed with leprosy. After 1 week of therapy, he develops fever, chills, low back pain, and dark colored urine. A CBC reveals a hemoglobin of 6 g/dl, (13.5-17.5 gldL), leukocyte count of 15,OOO/mm3 (4500-11,OOO/mm3), and a platelet count of 450,OOO/mm3 (150,000-400,OOO/mm3). A corrected reticulocyte count is 18% (0.5-1.5%). A direct and indirect Coombs' test is negative. The peripheral smear exhibits polychromasia and RBCs missing parts of their membrane. There is a positive urine dipstick for blood. The urine sediment is reported as normal. Which of the following statements apply to this case? SELECT 3

A. Heinz bodies are likely present

B.. Predominantly extravascular hemolysis

C~ Low RBC glutathione levels

D~ Patient is taking dapsone

E~ Autosomal recessive disease

I

I

I

I I

I

!

10. An afebrile 65 year old man presents with fatigue and substernal chest pain with exertion that is relieved by resting. Physical exam reveals a blood pressure of 100/70 mm Hg, pulse of 110

beats/minute (diminished amplitude), pale conjunctiva, and a harsh grade ill-IV systolic murmur with radiation into the carotid arteries. The intensity of the murmur increases with expiration and when the patient is lying down. A CBC exhibits a moderately severe anemia with a mean corpuscular volume (MeV) of 76 Jllll3 (80-100 fJlIl3), hemoglobin of 7.S gldL (13.5-17.5 gldL), leukocyte count of 8,OOO/mm3 (4500-11,OOO/mrn3), and a platelet count of 500,OOO/mm3 (lSO,000-400,OOO/nun3) The peripheral smear uncovers numerous fragmented RBCs. The corrected reticulocyte count is 15% (0.5-1.5%). There is a positive urine dipstick for blood but RBCs are not present in the sediment. Which of the following apply to this case? SELECT 4

A. Positive direct Coombs' test

B. Low plasma haptoglobin

C. Schistocytes

D. Hemoglobinuria

E. Aortic regurgitation

F. Low serum ferritin

11. An asymptomatic, normotensive 21 year old African-American woman is noted to have microscopic hematuria on an otherwise normal physical exam. The urine culture returns negative. A renal ultrasound is reported as normal. Her hemoglobin concentration is 12.5 g/dl, (12.0-16.0 gldL) and the peripheral smear is reported asnormal. Based on these findings, which of the following is the next best step in the management of this patient?

A. Sickle cell preparation

B. Reticulocyte COWlt

C. Renal biopsy

D. Cystoscopy

E. No further work-up

15

Note: This material is copyrighted. All rights reserved. (Edward F. Goljan, M.D.) 2001

12. A pregnant 21 year old African-American woman has a mild microcytic anemia with a normal RDW and slightly elevatedRBC count. A Hgb electrophoresis is reported to be abnormal. You suspect that the Hgb electrophoresis showed... SELECT 3

A. increased Hgb S

B. decreased Hgb A

C. increased Hgb A2

D. increased Hgb F

E. increased Hgb H

13. A 46 year old man with diastolic hypertension develops fever, jaundice, and a severe hemolytic anemia while taking methyldopa. The hematocrit is 15% (41-530/0) and the uncorrected reticulocyte count is 24%. Spherocytes and polychromasia are present in the peripheral smear. Which of the following most likely apply to this case? SELECT 3

A. Intravascular hemolytic anemia

B. Reticulocyte index is 4%

C. Unconjugated hyperbilirubinemia

D. Type ill hypersensitivity reaction

E. Autoantibodies against Rh antigens

14. A 65 year old man presents with fever and chills. Physical examination reveals generalized, nontender I ymphadenopathy, hepatosplenomegaly, and scattered petechia and ecchymoses over the

anterior chest. A CBC report indicates a hemoglobin of 8.2 gldL (13.5-17.5 gldL), leukocyte count of 70,OOO/mm3 (4,500-11 ,OOO/mml) with 90% mature and immature appearing lymphocytes, and a platelet count of 80,OOO/mm3 (ISO,000-400,OOO/mm3). The peripheral smear contains numerous smudge cells. A blood culture is positive for Streptococcus pneumoniae.The total serum protein concentration is 4.0 gldL (6.0-7.8 g/dl.). A serum protein electrophoresis exhibits a flat y-globulin peak and decreased albumin. Which of the following apply to this case? SELECT 3

A. Leukemoid reaction secondary to sepsis

B. Hypogammaglobulinemia

C. Chronic lymphocytic leukemia

D. Low leukocyte alkaline phosphatase score

E. B cell malignancy

15. A 4 year old boy presents with fever, epistaxis, and testicular pain. Physical exam reveals generalized, non-tender lymphadenopathy, hepatosplenomegaly, sternal tenderness to percussion, and widespread petechia and ecchymoses. Both testicles are enlarged, slightly tender, and do not transilluminate. The CBC report indicates a hemoglobin of 6 gldL (11.0-14.0 g/dl.), leukocyte count of 30,OOO/mm3 (5,500-1S,SOO/mm3), and a platelet count of 50,OOO/mm3 (lSO,000-400,OOO/mm3). A bone marrow aspirate reveals sheets of cells similar to those present in the peripheral blood. Which of the following apply to this case? SELECT 3

A. Cells are most likely CALLA positive

B. Cells are most likely Tdt positive

C. Acute lymphoblastic leukemia

D. Lymphoid leukemoid reaction

E. Leukemia derives from trilineagernyeloid stem cell

16

,""----

Note: This material is copyrighted. All rights reserved. (Edward F. Goljan, M.D.) 2001

16. A 45 year old man presents with fever, weight loss, and sweating. Physical exam reveals a normotensive individual with generalized, non-tender lymphadenopathy and massive hepatosplenomegaly. The CBC report indicates a hemoglobin concentration of 7 gldL (150,000-400,OOO/mm3), leukocyte count of lSO,OOO/mm3 (4,500-11,OOO/mm3), and a platelet count of 650,OOO/mm3 (150,000-400,OOO/mrn3). The peripheral smear demonstrates neutrophils at all stages of development (1% myeloblasts), mature and immature eosinophils and basophils, and thrombocytosis with giant platelets. A bone marrow aspirate is dry, however, the bone marrow biopsy reveals a hypercellular marrow with an increase in reticulin fibers. Imprints of the biopsy reveal a similar differential COWlt as that observed in the peripheral blood. Which of the following additional laboratory test abnormalities would you expect in this patient? SELECT 3

A Low leukocyte alkaline phosphatase score

B Positive tartrate resistant acid phosphatase stain

C Auer rods in myeloblasts

D Positive Philadelphia chromosome study

E Positive ber-fusion gene study

F. t-8;14 translocation

17. A 19 year old African American woman presents with fatigue and exercise intolerance. She has a history of menorrhagia and sporadically takes ferrous sulfate tablets. A CBC reveals a mild normocytic anemia, a low normal WBC count, a normal platelet count, and normal RBC morphology. A corrected reticulocyte count is <2%. The next most important step is a. ••

A. serum ferritin

B. Coombstest

C. serum folate/Bj,

D. Hgb electrophoresis

E. sickle cell preparation

18. An afebrile 80 year old man with the myelodysplastic syndrome is symptomatic with a 7 gm/dL hemoglobin. He is given 3 units of packed RBCs and on the following day has a hemoglobin of 8 gmJdL. His direct Coombs' test is negative. A dipstick of urine for blood is negative. The MOST LIKEL Y cause for only a 1 gm/dL rise in the post-transfusion hemoglobin concentration is •••

A.. destruction of the RBCs in the bone marrow

B. a microangiopathic hemolytic anemia

C. an autoimmune hemolytic anemia

D. destruction of the RBCs in spleen

E. a gastrointestinal bleed

19. A 22 year old college student presents with petechia, ecchymoses, epistaxis, generalized lymphadenopathy, and hepatosplenomegaly. A CBC reveals a normocytic anemia, a total WBC count of 30,000 cells/ul., and thrombocytopenia. Abnormal "blast cells" are noted in the peripheral smear. The PT and PIT are prolonged, and the D-dimer test is positive. Which of the following apply to this case? SELECT 2

A. Positive tartrate resistant acid phosphatase stain

B. Therapeutic response to transretinoic acid

C. Low leukocyte alkaline phosphatase score

D. t(15; 17) translocation

E. Invasion of the gums

17

r··

'4;.

Note: This material is copyrighted. All rights reserved. (Edward F. Goljan, M.D.) 2001

, , ,

t t t ,

, , ,

t ,

t ,

t t

t t. t t 4 t ,

• ,



20. Which of the following is more often associated with ~-thalassemia minor than iron deficiency?

SELECT 2

A. LowMCV

B. Increased ROW

C. Increased serum ferritin

D. Normal to high RBC count

E. AbnonnaI Hgb electrophoresis

21. A 28 year old woman presents with fever, fatigue, and scleral icterus. She is not taking any prescription or over-the-counter medications. Physical exam reveals generalized painful lymphadenopathy, hepatosplenomegaly, and a facial rash in a butterfly distribution. A CBC exhibits a Hgb of 6 g/elL, a slightly increased MeV, thrombocytopenia, and slightly low WBC count. The peripheral smear demonstrates spherocytes, RBC polychromasia, and occasional nucleated RBCs. No hypersegmented neutrophils are present. The uncorrected reticulocyte count is 150/0. Which of the following apply to this case?

A. Bilirubin in the urine

B. Positive direct Coombs' test

C. Unconjugated hyperbilirubinemia

D. Reticulocyte index < 2%

E.. Positive serum antinuclear antibody test

22. You would expect a mature RBC to ••• SELECT 1

A. utilize lactate dehydrogenase

B. utilize glucose 6-phosphatase

C. convert glucose into glycogen

D. utilize pyruvate dehydrogenase

E. have a net gain of (2) ATPand (0) NADH

23. An afebrile patient with SLE has a mild normocytic anemia with an elevated leukocyte count exhibiting neutrophilic leukocytosis, lymphopenia, eosinopenia, and a normal platelet count. There is no left shift or toxic granulation present in the smear. Her stool guaiac is positive. The hematologic findings are MOST CONSISTENT with •••

A. acute leukemia

B. a viral infection

C. abacterial infection

D. gram negative sepsis

E. corticosteroid therapy

t

t

24. Which of the following hemolytic anemias are primarily due to extravascular removal of RBCs?

SELECT 2

A. Microangiopatbic hemolytic anemia

B. Warm autoimmune hemolytic anemia c. Congenital spherocytosis

D. G6PD deficiency

E. Paroxysmal nocturnal hemoglobinuria

F. Cold autoimmune hemolytic anemia

,

18

----~--.,

Note: This material is copyrighted. All rights reserved. (Edward F. Goljan, M.D.) 2001

\

25. Which of the following correctly describe anemia? SELECT 2

A. Hypoxemia

B. Low oxygen saturation

C. Decreased oxygen content

D. Left shifted oxygen dissociation curve

E. Tissue hypoxia

26. A 65 year old man presents with joint pains, palpable purpura, and hepatosplenomegaly. There is no evidence of lymphadenopathy. A CBC exhibits a normocytic anemia, thrombocytopenia, and neutropenia. A few "blast cells" with irregular cytoplasmic borders are noted in the peripheral blood .. A bone marrow biopsy reveals a monomorphic infiltrate of cells with abundant cytoplasm having a "fried egg" appearance. A special stain is pending. Which of the following apply to this case? SELECT 2

A. t(9;22) translocation

B. Epstein-Barr virus association

C. B cell malignancy

D. Positive tartrate resistant acid phosphatase stain

E. Low leukocyte alkaline phosphatase score

27. An 82 year old man has a long history of a severe slightly macrocytic anemia and pancytopenia. He requires a transfusion of packed RBCs every 2 weeks in order to maintain his Hb concentration around 10 g/dL. His peripheral smear consistently shows a dimorphic RBC population associated with occasional rnyeloblasts and progranulocytes. A bone marrow aspirate reveals numerous ringed sideroblasts and a 10% myeloblast count. This patient MOST LIKELY has ...

A. acute myelogenous leukemia

B. agnogenic myeloid metaplasia c. the myelodysplastic syndrome

D. chronic myelogenous leukemia

E. a neutrophilic leukemoid reaction

28. A 53 year old woman with an abnormal mammogram of the left breast, palpable left axillary lymph nodes, and low back pain is noted to have a mild normocytic anemia, thrombocytosis, a slightly elevated WBC count, and a peripheral smear demonstrating nucleated RBCs, and occasional progranulocytes and myelocytes, The fmdings are consistent with a ... SELECT 2

A. chronic myelogenous leukemia

B. acute myelogenous leukemia c. marrow infiltrative disease

D. leukoerythroblastic smear

E. leukemoid reaction

29. Eosinophilia is commonly associated with ... SELECT 4

A. pinworms

B. bronchial asthma

C. malaria

D. amebiasis

E. Addison's disease

F. poison ivy

G. penicillin skin rash

H. strongyloidiasis

19

------------------_._- -

i\ote: 'I his material is copyrighteci. All rights reserved. (Edward F. Goljan, M.D.) 2001

30. Toxic granulation, left shift, and neutrophilic leukocytosis are expected in ... SELECT 4

A. acute appendicitis

B. Crohn's disease

C. acute diverticulitis

D. acute cholecystitis

E. acute myocardial infarction

F. rheumatoid arthritis

31. Pure RBC aplasia is most commonly associated with ... SELECT 2

A. parvovirus infection

B. chronic myelogenous leukemia

C. thymoma

D. sickle cell trait

E. autoimmune hemolytic anemia

32. A 2 yr old child has a microcytic anemia. Which of the following is the first step in the work-up of the patient?

A. Serum ferritin

B. Stool guaiac

C. Bone marrow

D. Direct Coombs test

E. Hgb electrophoresis

33. A S5 yr old man has a microcytic anemia. Which of the following is the first step in the work-up of the patient?

A. Serum ferritin

B. Stool guaiac

C. Bone marrow

D. Direct Coombs test

E. Hgb electrophoresis

34. In lead poisoning, the encephalopathy is directly attributed to ...

A. an increase in RBe protoporphyrin

B. an increase in o-aminolevulinic acid

C. Pb depositing in the brain

D. an alteration in Starling's forces

E. tissue hypoxia secondary to anemia

35. In a patient with Bl2 deficiency who is being treated with pharmacologic doses of folate, which of the following will be corrected?

A. Megaloblastic anemia

B. Glossitis

C. Pancytopenia

D. Vibratory sensation in the lower extremities

E. Proprioception

F. Dementia

20

Note: This material is copyrighted. All rights reserved. (Edward F. Goljan, M.D.) 2001

\

36. Newborns with sickle cell disease do not have hemolytic or vasoocclusive crises at birth since ... SELECT 2

A. the high concentration of Hgb F inhibits sickling

B. levels ofHgb S are not high enough to induce sickling

C. Hgb A inhibits sickling

D. splenic macrophages are of insufficient number to remove sickle cells

E. the spleen traps the sickle cells

37. A 19 yr old African American man with sickle cell disease develops bone pain in the femur. A radionuclide bone scan reveals a lytic lesion in the metaphysis of the femur. The pathogen most likely responsible for the patient's bone disorder is .....

A. Staphylococcus aureus

B. Streptococcus pneumoniae

C. Salmonella paratyphi

D. Pseudomonas aeruginosa

E. Hemophilus injluenzae

38. In a atient with sickle cell trait, ou would ex eet the MSm endonuclease studies to reveal ...

1.35 kb segment 1.1S kb fragment 0.2 kb fragment

c. none

Lymphoproliferative questions

Items 1-2

A 65 year old woman presents with fatigue and pain in her lower back and ribs. She states that her urine flow has also decreased dramatically in the last few days. Physical examination reveals sternal and vertebral percussion tenderness and bilateral conjunctival pallor. A CBC exhibits extensive rouleaux, a hemoglobin of 7.5 g/dL (12.0-16.0 g/dL), a total leukocyte count of 4,300/mm3 (4500-1 1 ,0 0 Ozmm') , and a platelet count of 125,OOO/mm3 (150,000-400,OOO/rnm3). A urinalysis exhibits a 2+ dipstick for protein and 4+ precipitation using sulfosalicylic acid (SSA). Renal tubular casts are noted in the sediment. A chest x-ray reveals generalized osteopenia in the ribs and vertebra as well as multiple lytic lesions in the ribs.

I. Based on the patient's history and preliminary laboratory findings, which of the following additional abnormalities would you expect? SELECT 3

A. Hypercalcemia

B. Normal erythrocyte sedimentation rate c. IgM monoclonal spike in serum

D. Increased light chains in the urine

E.Malignant plasma cells in a bone marrow aspirate

2. What additional tests should be performed to confirm the diagnosis? SELECT 2

A. Bone marrow aspirate

B. Renal biopsy

C. Radionuclide bone scan

D. Intravenous pyelogram crvP)

E. SerurnJurine immunoelectrophoresis

21

Note: This material is copyrighted. All rights reserved. (Edward F .. Goljan, M.D.) 2001

Items 3-7

A. Histocytosis X

B. Sezary syndrome

C. Hodgkin's disease

D. Burkitt's lymphoma

E. Immunoblastic lymphoma

F. Walden strom 's macroglobulinemia

Each set of options relating to lymphohistiocytic disorders is followed by several numbered items. For each numbered item, select the ONE lettered option that is MOST CLOSELY associated with it. Each lettered option may be selected once, more than once, or not at all.

3. A 9 year old girl presents with colicky pain secondary to entrapment of small bowel by enlarged para-aortic lymph nodes. A section of lymph node removed at laparotomy reveals a diffuse neoplastic infiltrate of small, round lymphocytes with a "starry sky" appearance.

4. A 68 year old man presents with multiple plaque .. like lesions on his skin, generalized lymphadenopathy, and hepatosplenomegaly. A biopsy reveals atypical lymphocytes infiltrating the epidermis. Neoplastic CD4 positive lymphocytes with prominent nuclear clefts are noted in his peripheral blood.

5. A 4 year old child presents with exophthalmos, polyuria, and multiple lytic lesions in the skull. A bone marrow aspirate reveals an infiltrate of neoplastic cells that are CD 1 positive.

6. A 60 year old man has a normocytic anemia, an elevated erythrocyte sedimentation rate, generalized lymphadenopathy, hepatosplenomegaly, hyperviscosity syndrome and an abnormal finding on a serum protein electrophoresis.

7. A 28 yr old woman has an anterior mediastinal mass and non-tender lymphadenopathy in the right supraclavicular node

8. Restrictive cardiomyopathy, macroglossia, and nephrotic syndrome are all associated with a disorder characterized by .•.

A. neoplastic histiocytes in a bone marrow aspirate

B. neoplastic plasma cells in a bone marrow aspirate

C. neoplastic lymphocytes with a t(8;14) translocation

D. lymphoplasmacytoid cells and a monoclonal increase in IgM

E. material exhibiting apple green birefringence under polarized light

22

Note: This material is copyrighted. All rights reserved. (Edward F. Goljan, M.D.) 2001

Coagulation questions

1. A 22-year-old African American woman with no previous bleeding history develops persistent bleeding of her gums and tooth socket following dental surgery. Additional history reveals problems related to heavy menses that significantly resolve when she is taking birth control pills and recur to the same level of severity when she discontinues the medication. Hematologic studies

reveal the following: partial thromboplastin time (PIT) 55 sec (28-40 sec), prothrombin time (PT) 12 sec (II-IS sec), factor VITI: coagulant 30% (50-150%), factor VITI: antigen 40% (60-150%), bleeding time 15 min (2-7 min), platelet count 300,000 mm' (150,000-400,000 mrrr'), hemoglobin 13.0 g/dL (12.0-16.0 g/dL). Which of the following laboratory tests is most useful in confirming the cause of her bleeding?

A. Leukocyte count

B. Ristocetin cofactor assay

C. Hemoglobin electrophoresis

D. Sickle cell preparation

E. Serum ferritin

~ ~j

I

; , ,

2. A 23-year-old man requires a root canal for an abscessed tooth for which he has been taking pain medication. On the day prior to the procedure, he develops a severe nosebleed, which prompts his dentist to order a few laboratory studies, which are as follows. PIT 35 sec (28-40 sec), PT 13 sec (11-15 sec), bleeding time 16 min (2-7 min), platelet count 200,000 mm' (150,000--400,000 rnm'), Hgb 15.5 g/dL (13.5-17.5 g/dL). Which of the following hemostasis abnormalities is most likely present in this patient?

A. Von Willebrand's disease

B. Hemophilia A

C. Factor IX deficiency

D. Acquired platelet defect

E. Acquired vascular defect

Items 3-4

A 63 year old man with urinary retention secondary to benign prostatic hyperplasia develops fever and chills shortly after insertion of an indwelling catheter. Physical exam reveals warm skin and a bounding pulse. Within 24 hOUTS, he begins oozing blood out of venipuncture sites and from his mucous membranes. Ecchymoses appear over his trunk and extremities. His urine output decreases to <400 mL/day. Laboratory studies reveal the following: Hgb 10 g/dL (13.5-17.5 g/dL), WBC count 2,OOO/mm3 (4,500-11,OOO/mm3), platelet count 140,OOO/mm3 (lSO,000-400,OOO/mm3), PIT 42 sec (28-40 sec), PT 18 sec (11-15 sec), plasma fibrinogen 150 mg/dL (200-400 mg/dL) , fibrin(ogen) degradation products >10 J.1g/mL «10 J.1g/mL) , D-dimers positive (negative), blood cultures pending, serum BUN 80 mg/dL (7-18 mg/dL), serum creatinine 8 mg/dL (0.6-1.2 mg/dL)

3. Which of the following apply to this case? SELECT 4

A. Thrombotic thrombocytopenic purpura

B. Disseminated intravascular coagulation

C. Consumption of coagulation factors

D. Prerenal azotemia

E. Secondary fibrinolysis

F. Decreased production of platelets

G. Increased total peripheral resistance

H. Endotoxic shock

23

Note: This material is copyrighted. All rights reserved. (Edward F. Goljan, M.D.) 2001

4. Which of the following is the most effective treatment for this patient?

A. Fresh frozen plasma

B. Packed red blood cells

C. Platelet concentrates

D. Low dose heparin

E. Antibiotics

F. Cryoprecipitate

5. Which of the following laboratory test results are more often associated with classical von Willebrand's disease rather than mild hemophilia A? SELECT 3

A. Normal prothrombin time

B. Low factor Vill:coagulant activity c. Abnormal ristocetin cofactor assay

D. Prolonged partial thromboplastin time

E. Prolonged bleeding time

F. Low VIII: antigen

G. Normal platelet count

H. Response to desmopressin acetate Items 6-12

Platelet count Bleeding time PTT PT
A. Decreased Prolonged Nonna! Normal
B" Decreased Prolonged Prolonged Prolonged
c. Normal Prolonged Prolonged Normal
D. Normal Normal Prolonged Normal
E. Normal Normal Prolonged Prolonged
AB. Normal Normal Normal Prolonged
AC. Normal Prolonged Nonna} Normal Each set of options relating to hemostasis studies is followed by several numbered items. For each numbered item, select the ONE lettered option that is most closely associated with it. Each lettered option may be selected once, more than once, or not at all.

6. A 4 yr old child has eaten rat poison and is hemorrhaging

7. A 54 yr oldman is on heparin to prevent deep venous thrombosis

8. A 28 yr oldman has been envenomated by a rattlesnake and is bleeding from all orifices and all needle stick sites

9. A child has eaten raw hamburgers and now has a hemolytic anemia and renal failure

10. A man has a family history of a bleeding disorder which began with his mother's father

11. An afebrile 8 year old boy presents with epistaxis 1 week after an upper respiratory infection.

Physical examination reveals scattered petechia and ecchymoses over his trunk, There is no lymphadenopathy or hepatosplenomegaly. A stool for occult blood is negative. His CBC exhibits a hemoglobin of 13 g/dl, (12.0-15.0 g/dl.), leukocyte COWlt of8,500/mm3 (4,500-13,500/mm3) with a normal differential count, and a platelet count of lO,OOO/mm1 (150,000-400,OOO/mm3).

24

Note: This material is copyrighted. All rights reserved. (Edward F. Goljan, M.D.) 2001

12. A 38 year old woman presents with fever, mental status alterations, and epistaxis. Physical examination reveals retinal hemorrhages, widespread petechia and ecchymoses, and a positive stool for occult blood. There is no lymphadenopathy or hepatosplenomegaly. The CBC report indicates the presence of a normal hemoglobin and leukocyte count. The corrected reticulocyte count is 12%. The peripheral smear exhibits numerous fragmented RBCs, polychromasia, and a reduced number of platelets. A bone marrow biopsy contains vascular channels containing platelet thrombi. There is a positive urine dipstick for blood and numerous RBCs are present in the sediment. The serum blood urea nitrogen is 40 rng/dL (7-18 mg/dL) and the serum creatinine is 4 mgldL (0.6-1.2 mg/dL).

13. An 18-year-old smoking male presents with deep venous thrombosis (DVT) involving the right lower calf. He has a family history of recurrent DVTs and pulmonary emboli in his mother and maternal grandfather. His PIT and PT are normal prior to receiving a standard dose of heparin intravenously- The PIT remains normal 1 hour after infusing heparin. The pathogenesis of his hemostasis abnormality is most likely related to ....

A. antiphospbolipid antibodies

B. a thrombohernorrhagic disorder

C. a deficiency of antithrombin ill

D. a deficiency of a vitamin K-dependent factor

E. antibodies directed against heparin

14. The pathogenesis of hemorrhagic skin necrosis associated with warfarin therapy is most closely associated with ...

A. antibodies directed against warfarin

B. a drug hypersensitivity reaction

C. immune vasculitis secondary to warfarin

D. protein C deficiency in the patient

E. antithrombin ill deficiency in the patient

Blood Bank questions

1. A 65 year old man in an intensive care unit is recovering from surgery for a ruptured abdominal aortic aneurysm. The patient required 12 units of packed red blood cells prior to surgery in order to stabilize his blood pressure. On the 5th postoperative day, he develops fever, scleral icterus, and low back pain. Physical exam reveals scattered rhonchi throughout both lung fields but no areas of consolidation. A urine sample taken from his indwelling urinary catheter exhibits pyuria. Cultures isolated E. coli. A CBC reveals a 3 g/dL drop in his hemoglobin concentration when compared to his postoperative levels. A direct Coombs' test is positive. No hemoglobin is noted in plasma. The total bilirubin is 4 mgldL (0.1-1.0 mgldL) with a direct (conjugated) bilirubin of 0.4 mg/dL (0.0-0.3 mg/dL). The serum alanine aminotransferase concentration is 20 UIL (8-20 U/L). Which of the following apply to the patient's clinical condition? SELECT 3

A. Febrile transfusion reaction

B. Posttransfusion hepatitis

C. Delayed hemolytic transfusion reaction

D. Type II hypersensitivity reaction

E. Conjugated type of hyperbilirubinemia

F. Extravascular hemolysis

25

2~ A blood group 0 Rh negative woman with a negative antibody screen delivers a blood group A, Rh positive baby. The baby is mildly anemic and develops an unconjugated hyperbilirubinemia in the first 24 hours. A direct Coombs' of cord blood returns weakly positive. Spherocytes are noted in the peripheral blood smear. The baby's stool is grossly bloody. An Apt test is performed on the stool sample and reveals adult hemoglobin. The mother's antibody screen is negative. Which of the following best describe the pathogenesis of the baby's anemia? SELECT 4

A. .ABO incompatibility

B. Rh incompatibility

C. Type II hypersensitivity reaction

D. Mother is not a candidate for Rh immune globulin

E. .ABO incompatibility protects against Rh sensitization

F. Positive direct Coombs is due to anti-D coating babies RBCs

G. Newborn has a GI bleed

, , , ,

t t t ,

t t ,

t t C t C C t f {

Note: This material is copyrighted. All rights reserved. (Edward F. Goljan, M.D.) 2001

\, ,

)

3. 'A group 0, Rh negative 32 year old woman develops fever of 1030 F while receiving a unit of 0, Rh negative packed red blood cells after a hysterectomy. A transfusion reaction workup on the patient reveals the following:

Antibody screen negative

Direct Coombs' negative

Plasma normal color

Urinalysis dipstick negative for blood

Pretransfusion hemoglobin 7 gldL (12.0-16.0 gldL)

Post-transfusion hemoglobin 7.S gldL (12.0-16.0 gldL)

Which of the following apply to this case? SELECT 2

A. Donor blood contamination with Yersinia enterocolitica

B. Type I hypersensitivity reaction

C. Patient should receive leukocyte poor blood for further transfusions

D. Hemolytic transfusion reaction due to antibody destruction of donor RBCs

E. Patient anti-HLA antibodies are directed against donor leukocytes

4. An 82 yr old woman with blood group A inadvertently receives blood group B blood. He does not develop a hemolytic transfusion reaction. This is most likely due to ...

A. absence of isohemagglutinins with old age

B. a defect in cellular immunity

C. Bruton's agammaglobulinemia

D. antithrombin m deficiency

E. absent anti-A IgM titers in the donor unit

5. Which of the following is more often associated with Rh hemolytic disease of the newborn due to anti-D antibodies rather than ABO incompatibility? SELECT 2

A. No risk for hemolytic disease of the newborn during the first pregnancy

B. Positive direct Coombs' on the babies cord RBCs

C. Spherocytes in the newborn's peripheral blood

D. Severe anemia requiring blood transfusion

E. Unconjugated hyperbilirubinemia '

F. Type n hypersensitivity reaction

26

Note: This material is copyrighted. All rights reserved. (Edward F. Goljan, M.D.) 2001

6. A blood group 0, Rh negative 65 old year old man with known diverticulosis presents with a massive lower GI bleed (hematochezia). He has an estimated blood loss of greater than 600 mL over the last few hours. An intravenous line with 0.9% normal saline is in place, while blood is being crossmatched in the blood bank. His Hgb is 6 g/dL. Physical exam reveals cold, clammy skin, a blood pressure of 70/40 mm Hg, and a weak pulse with a rate of 120 bpm, Owing to a shortage of 0, Rh negative blood only 1 of the 5 units of packed RBCs are 0, Rh negative, while the remaining 4 units are 0, Rh positive. The patient has no history of a previous transfusion or exposure to blood products. Midway through the second unit of blood, which is the first of the four units of 0, Rh positive blood, he develops wheezing, dyspnea, and swelling of his face. The transfusion is stopped, a subcutaneous injection of aqueous epinephrine at a 1: 1000 dilution is given to the patient, and a transfusion reaction work-up is ordered. The transfusion reaction work-up on a posttransfusion specimen of patient blood is as follows:

Patient temperature: 100.0 0 F

Patient blood pressure: 60/40 nun Hg

Patient pulse: 130 bprn

Patient plasma:clear

Patient antibody screen: negative

Patient direct Coombs': negative

Patient urine: negative dipstick for blood

The pathogenesis of this patient's transfusion reaction is most closely related to ... SELECT 2

A anti-D antibodies destroying donor D antigen positive RBCs

B patient anti-HLA antibodies reacting against donor leukocytes

C a patient 19E-mediated reaction against a donor allergen

D a type I hypersensitivity reaction

E an intravascular hemolytic anemia

7. A blood group 0, Rh negative woman with a negative antibody screen and no previous administration of Rh immune globulin during her pregnancy delivers a blood group B, Rh positive baby. The baby develops unconjugated hyperbilirubinemia a few hours after birth. The pathogenesis of the baby's jaundice is most closely related to ...

A. maternal anti-D antibodies destroying the babies Rh positive RBCs

B. intravascular hemolysis of fetalRBCs byanti-B IgM antibodies

C. intravascular hemolysis of fetal RBCs by anti-A,B IgG antibodies

D. extravascular hemolysis of fetal RBCs by anti-B IgG antibodies

E. extravascular hemolysis of fetal RBCs by anti-A,B IgG antibodies

8. A major crossmatch ... SELECT 3

A. prevents post-transfusion hepatitis due to hepatitis C

B. does not guarantee survival of the infused donor RBCs

C. rules out the possibility of a febrile transfusion reaction

D. does not prevent patient antibodies developing against donor RBC antigens

E. prevents antibodies in the donorfrom destroying patient RBCs

F. detects the presence of patient antibodies against donor RBC antigens

27

Note: This material is copyrighted. All rights reserved. (Edward F. Goljan, M.D.) 2001

Items 9-12

Forward type using-

anti-A anti-B

Back type DSing-

A RBCs B RBCs

A. positive negative negative positive
B. positive positive negative negative
c. negative positive positive negative
D. negative negative positive positive Each set of options relating to ABO blood group typing is followed by several numbered items. For each numbered item, select the ONE lettered option that is most closely associated with it. Each lettered option may be selected once, more than once, or not at all.

9. A 45 year old man with exertional dyspnea and extreme fatigue and a long history of duodenal ulcer disease is noted to have dark black, tarry stools. Physical exam reveals pale conjunctiva and a wide pulse pressure. Laboratory studies reveal a 6 g/dl, hemoglobin and a low MeV. Donor blood for transfusion of this patient based on the type of peptic ulcer disease he has would most likely have which of the above forward and back type results in the blood bank?

\ )

10. A 32 year old woman and her husband have 10 children. Her husband's blood group is A. All of their children are either A, B, or AB. In order to explain the blood groups in her children, you would expect the woman to have which of the above forward and back type results on a sample of her blood?

11. A 49 year old has a gastric adenocarcinoma. Based on his diagnosis, which of the above blood group types is best for transfusing the patient?

12. A patient with this blood group could develop a hemolytic transfusion reaction after receiving .AB blood or A blood

13. A phlebotomist inadvertently sticks himself with a needle after drawing blood from a patient with AIDS. Which of the following infections is the phlebotomist at most risk for contracting?

A. HIV

B. Hepatitis A

C. Hepatitis B

D. Hepatitis C

E. Syphilis

14. A phlebotomist inadvertently sticks himself with a needle after drawing blood from a patient.

Which of the following infections is the phlebotomist at most risk for contracting?

A. HN

B. Hepatitis A

C. Hepatitis B

D. Hepatitis C

E. Syphilis

28

t

t:

..

• •

t ~ ,

,

C t ,

t t C t f C t (

t

,

Note: This material is copyrighted. All rights reserved. (Edward F. Goljan, M.D.) 2001

15. Which of the following is the most common antibody encountered in clinical practice?

A. Anti-HA V -IgG

B. Anti-HBs

c. Anti-HeV -IgG D. Anti-CMV

E. Heterophile antibodies

16. A patient who has chronic hepatitis and has been transfused in the past most likely has antibodies directed against ...

A. hepatitis A

B. hepatitis B

C. hepatitis C

D. HN

E. CMV

Cardiovascular questions

1.

A 72-year-old man presents with a sudden onset of left flank pain. In the emergency room, the patient is hypotensive. A pulsatile mass is palpated in the abdomen. Which of the following is MOST responsible for the pathogenesis of this patient's condition?

A. Atherosclerosis

B. A defect in fibrillin

C. A defect in collagen

D. Long-standing hypertension

E. Immune destruction of elastic tissue

\

I

)

2. A 42 year old man with a history of cardiac death in his family presents with a sudden onset of severe, retrosternal chest pain with radiation of the pain into his back. His left pulse is absent A high pitched diastolic blowing murmur that increases with expiration is heard immediately after S2. There is widening of the aortic knob on a chest x-ray. The mechanism for this patient's condition is MOST CLOSELY related to •..

A. atherosclerosis

B. a defect in fibril1in

C. a defect in collagen

D. an infectious process

E. an acute myocardial infarction

3. A 26-year-old woman presents with a history of chest palpitations particularly when anxious.

Physical exam reveals a mid-systolic ejection click followed by a murmur. The click and murmur move closer to SI when the patient is standing and closer to S2 when the lying down. The mechanism for this patient's valvular disorder is MOST CLOSELY related to •..

A. a defect in fibrillin

B. an infectious process

C. immunologic damage

D. myxomatous degeneration

E. a defect in collagen synthesis

29



I

6.

Which of the following is present in BOTH left and right heart failure?

A. S3 heart sound

B. Pillow orthopnea c. Pulmonary edema

D. Neck vein distention

E. Dependent pitting edema

Note: This material is copyrighted. All rights reserved. (Edward F. Goljan, M.D.) 2001

4. A 28-year-old patient has a family history of sudden cardiac death at a young age. The patient has a systolic ejection murmur that decreases in intensity when the patient is lying down and increases in intensity when standing up. An echocardiogram reveals abnormal movement of the anterior mitral valve leaflet against an asymmetrically thickened interventricular septum. The patient MOST LIKELY has ...

A. mitral valve prolapse

B. hypertrophic cardiomyopathy

C. a congenital bicuspid aortic valve

D. a cardiac myxoma of the left atrium

E. infective endocarditis involving the mitral valve

5. A 65-year-old man on the Sth day of hospitalization for an acute anterior myocardial infarction has recurrence of chest pain and an increase in both CK.-MB and troponin-l.The patient MOST LIKELY has ...

A. papillary muscle dysfunction

B. a right ventricular infarct

C. a ventricular aneurysm

D. a myocardial rupture

E. reinfarction

i

)

7 . You would expect a patient with an atrial septal defect to have which of the following oxygen

saturation ( Sa02) findings obtained by cardiac catheterization?
i Normal Patient Patient Patient Patient Patient
I
I
I sso, 0/0 A B C D E
Right atrium 75 7S 80 75 75 80
Right ventricle 75 80 80 7S 7S 80
Pulmonary art 75 80 80 80 75 80
Pulmonary_ vein 95 95 95 95 9S 95
Left ventricle 9S 95 9S 95 80 80
Aorta 95 95 95 95 80 80 Sa02 = oxygen saturation

8. A 30 year old man with a viral myocarditis who develops hypotension, neck vein distention, a drop in blood pressure on inspiration, and muffled heart sounds most likely has ...

A. hypertrophic cardiomyopathy

B. constrictive pericarditis

C. hypovolemic shock

D. a pericardial effusion

E. a dissecting aortic aneurysm

30

Note: This material is copyrighted. All rights reserved. (Edward F. Goljan, M.D.) 2001

Items 9-11

A. Aortic stenosis

B. Aortic regurgitation

C. Mitral stenosis

D. Mitral regurgitation

E. Tricuspid regurgitation

9. A 70 year old man with diminished pulses and a history of angina and syncope with exercise has an ejection type murmur radiating into the carotid arteries

10. A 58 year old man with left heart failure has an S3 and S4 heart sound and a pansystolicmunnur located at the apex that increases with expiration

11. A 29 year old intravenous drug abuser has fever, a giant c-v wave, and a pansystolic murmur along the left sternal border that increases with inspiration

12. Which of the following types of hepatitis is associated with a vasculitis characterized by p-ANCA antibodies?

A.. Hepatitis A

B. Hepatitis B

C. Hepatitis C

D. Hepatitis D

E. Hepatitis E

13. A 7 year old boy presents with a low-grade fever, arthralgias, colicky abdominal pain, and a palpable purpuric rash limited to the lower extremities. Laboratory studies reveal a guaiac-positive stool, a urinalysis with red blood cell (RBC) casts, hematuria, and mild proteinuria, and a CBe with a normal Hb, Hct, and platelet count. Which of the following is the most likely diagnosis?

A. Idiopathic thrombocytopenia purpura (ITP)

B. Systemic lupus erythematosus (SLE)

C. Poststreptococcal glomerulonephritis

D. Rocky Mountain spotted fever

E. Henoch-Schonlein vasculitis

Respiratory questions

1. In which of the following clinical scenarios involving patients with lung disease would you expect pulmonary function studies to exhibit decreased compliance, increased elasticity, and an increased FEV lseclFVC ratio?

A. 6 year old child with recurrent respiratory infections and steatorrhea

B. 28 year old non-smoking male with bilateral lower lobe emphysema

C. 56 year old smoker with productive cough, dyspnea, and cyanosis

D. 10 year old girl with bronchial asthma requiring systemic steroids

E. 39 year old dyspneic African American with bilateral hilar nodes

31

,_._---

Note: This material is copyrighted. All rights reserved. (Edward F. Goljan, M.D.) 2001

2. A 45-year old woman 24 hours post-cholecystectomy develops fever and dyspnea. Physical exam reveals decreased percussion, increased tactile fremitus, and decreased breath sounds in the right lower lobe. The diaphragm is elevated and there is inspiratory lag on the right side. The patient MOST LIKELY has ...

A. atelectasis

B. a lung abscess

C. bronchopneumonia

D. a pulmonary infarction

E. a spontaneous pneumothorax

3. An afebrile 23-year-old man develops a sudden onset of left-sided, stabbing chest pain with dyspnea. Physical exam of the left chest reveals hyperresonance topercussion, deviation of the trachea to the left, elevation of the diaphragm, decreased tactile fremitus, and decreased breath sounds. The MOST LIKELY diagnosis is ...

A. pleural effusion

B. bronchopneumonia

C. tension pneumothorax

D. a pulmonary infarction

E. spontaneous pneumothorax

4. A newborn child develops dyspnea, tachypnea, intercostal muscle retractions, and cyanosis 4 hours after birth. The mother developed gestational diabetes mellitus and was in poor glycemic control throughout the pregnancy. A chest x-ray reveals a "ground glass" appearance in both lungs. The primary mechanism for this patient's respiratory problem is ...

A. aspiration of amniotic fluid

B. group B streptococcus pneumonia

C. decreased production of surfactant

D. Chlamydia trachoma tis pneumonia

E. heart failure from congenital heart disease

5. Which of the following describes a pneumonia due to Mycoplasma pneumoniae rather than Streptococcus pneumoniael

A. High fever

B. Insidious onset

C. Productive cough

D. Increased tactile fremitus

E. Neutrophilic leukocytosis

6. A 58-year-old smoker presents with weight loss and cough. Physical exam reveals a mild lid lag on the left and a pinpoint pupil, scattered sibilant rhonchi throughout all lung fields that clear with coughing, and an increased anteroposterior diameter. Based on these fmdings,you suspect the patient has ...

A. a Pancoast tumor

B. a thoracic outlet syndrome

c. the superior vena caval syndrome

D. obstructive lung disease without primary cancer

E. obstructive lung disease with metastatic cancer from another primary site

32

Note: This material is copyrighted. All rights reserved. (Edward F. Goljan, M.D.) 2001

\ \

\

7. A 65 year old man with urinary retention secondary to prostatic hyperplasia, develops spiking fever, and tachypnea. Physical exam reveals intercostal muscle retractions and bilateral inspiratory crackles. A chest x-ray exhibits bilateral interstitial and alveolar infiltrates. ABGs demonstrate severe hypoxemia. You expect the blood culture reveals ...

A. gram positive diplococci

B. gram negative diplococci

C. gram positive cocci

D. gram negative rods

E. gram positive rods

8. Inspiratory stridor is commonly associated with ...

A. a respiratory syncytial virus infection

B. a parainfluenza virus infection

C. aspirin-induced asthma

D. rhinovirus infections

E. choana! atresia

9. Chlamydia trachomatis and the respiratory syncytial virus are BOTH commonly associated with ...

A. an interstitial type of pneumonia

B. laryngotracheobronchitis (croup) c. the respiratory distress syndrome

D.. typical community-acquired pneumonia

E. hospital-acquired (nosocomial) pneumonia

\

)

10. Which of the following is more often associated with Klebsiella pneumoniae than Pseudomonas aeruginosa?

A. Upper lobe cavitation

B. Green-colored sputum

C. Association with cystic fibrosis

D. Association with respirators

E. Productive cough

11. In a 30 year old man who lives in Tennessee, you would expect a calcified solitary coin lesion in the lung to represent ...

A. a foreign body

B. an old granuloma

C. metastatic cancer

D. a primary lung cancer

E.. a bronchial hamartoma

12. A 55-year-old non-smoking coal worker has arthritis and nodular lesions in the lungs. His PPD skin test is negative. You suspect the patient has ...

A. systemic lupus erythematosus

B. Caplan's syndrome

C. metastatic lung disease

D. primary lung cancer

E. miliary tuberculosis

33'

Note: This material is copyrighted. All rights reserved. (Edward F. Goljan, M.D.) 2001

\

13. In a 62 year old man who has been a roofer for 25 years and a smoker for 10 years, which of the following cancers would he be most likely prone to developing?

A. Pleural mesothelioma

B. Primary lung cancer

C. Laryngeal carcinoma

D. Oral cancer

E. Pancreatic cancer

14. A bridge painter in Brooklyn, New York develops a pulmonary infiltrate. Which of the following groups of pathogens are on your differential list?

A. Histoplasma capsulatum and Cryptococcus neofonnans

B. Cryptococcus neofonnans and Coccidioides immitis

C. Blastomyces dennatitidis and Histoplasma capsulatum

D. Aspergillus fumigatusand Coccidioides immitis

E. Pneumocystis carinii and Aspergillus fumigatus

15. Which of the following is a hypersensitivity pneumonitis that primarily occurs in textile workers?

A. Silo filler's disease

B. Bagassosis

C. Farmer's lung

D. Byssinosis

E. Sarcoidosis

16. Which of the following is a hypersensitivity pneumonitis that commonly occurs in farmers who enter a closed room with fermenting com?

A. Silo filler's disease

B. Bagassosis

C. Farmer's lung

D. Byssinosis

E. Sarcoidosis

17. If a patient is breathing room air (21 % oxygen), the arterial PC02 is 80 mm Hg, and the arterial Pa02 is 40 mm Hg, what is the Alveolar-arterial (A-a) gradient of the patient?

A. 10

B. 15

C. 20

D. 25

E. 30

34

-----------__;..._--- ----,_-

I

Note: This material is copyrighted. All rights reserved. (Edward F. GoIjan, M.D.) 2001

\

Gastrointestinal questions

Items 1-6

A. Vibrio cholera

B. Bacillus cereus

C. Shigella sonnei

D. Salmonella typhi

E. Campylobacter jejuni

F. Yersinia enterocolitica

G. Staphylococcus aureus

H. Enterotoxigenic E. coli

1. An afebrile 22 year old man and several other members of his family developed severe vomiting without diarrhea -1-6 hours after eating potato salad at a picnic. They all recovered uneventfully 12-24 hours later.

2. A 23 year old man developed explosive, watery diarrhea with blood, leukocytes, and mucus-3 days after eating chicken that was improperly cooked. Comma to S-shaped organisms are noted in the fecal smear of stool along with RBCs and leukocytes.

3.

A febrile 10 year old child presents with severe right lower quadrant pain that is interpreted by the attending physician as acute appendicitis. At laparotomy, the surgeon notes that the appendix is normal. However, the mesenteric lymph nodes are markedly enlarged and have focal areas of microabscess formation on cut section.

., \

)

4. A 29 year old man develops watery diarrhea and volume depletion shortly after visiting the Gulf Coast states

5. Ona trip outside of the country, a man develops a high fever associated with bradycardia, absolute neutropenia, and splenomegaly. A blood culture is positive for a gram negative organism.

6. A 23 yr old man develops vomiting and diarrhea after eating refiied rice and tacos. Gram positive rods are noted in the stooL

Items 7-8

A. Yersinia enterocolitica

B. Staphylococcus aureus

C. Enterotoxigenic E. coli

D. Cryptosporidiumparvum

E. Mycobacterium avium-intracellulare

7. A 28 year old man with AIDS presents with chronic, recurrent, profuse, nonbloody, watery diarrhea. An Entero-Test (string test) reveals oocysts that are partially acid-fast positive.

8. A 25 year old medical student during Spring break outside the country develops fever, vomiting, abdominal cramps, and watery diarrhea -14 hours after eating a few tacos purchased from a street vendor. He recovers uneventfully in 48-72 hours.

35



lrP:)i....._ . ..,... ,,~ .

Note: This material is copyrighted. All rights reserved. (Edward F. Goljan, M.D.) 2001

9. Odynophagia in a HIV -positive 28 year old man with white plaque-like material on his tongue and buccal mucosa MOST LIKELY has an AIDS-defining lesion caused by ...

A. Epstein-Barr virus

B. Candida albicans

C. Kaposi's sarcoma

D. cytomegalovirus

E. Herpes simp/ex

10. An afebrile 25-year-old medical student presents with intermittent complaints of left and right lower quadrant abdominal pain and distention associated with alternating periods of mucoid diarrhea and constipation. He states that stooling relieves the pain. A flexible sigmoidoscopy and stool guaiac exam are both normal. The patient MOST LIKELY has ...

A. an intrinsic bowel motility disorder

B. inflammatory bowel disease

C. chronic appendicitis

D. melanosis coli

E. celiac disease

11. Which of the following correctly describes a gastric rather than a duodenal ulcer?

A. Association with Helicobacterpylori

B.Highest incidence of perforation

C. Small risk for adenocarcinoma

D. Pain awakens the patient at night

E. Association with Zollinger-Ellison syndrome

12. The MOST C0Ml\10N location for diverticula, polyps, and cancer in the gastrointestinal tract is the •..

A. ascending colon

B. sigmoid colon

C. esophagus

D. stomach

E. rectum

13. Which of the following characterizes ulcerative colitis rather than Crohn' s disease?

A. Discontinuous spread

B. Toxic megacolon

C. Fistula formation

D. Perianal disease

E. Obstruction

14. A 38-year-old Asian woman has a long history of explosive diarrhea and abdominal distention after eating dairy products. The pathogenesis of this patient's diarrhea is MOST CLOSELY related to ...

A. antigliadin antibodies

B. activation of cyclic Al\1P

C. intraluminal osmotically active solutes

D. mucosal injury with increased permeability

E. loss of the absorptive surface of the small bowel

. 36

---- .-._--

Note: This material is copyrighted. All rights reserved. (Edward F. Goljan, M.D.) 2001

\

15. A 62-year old man smoker presents with weight loss, a dragging sensation in his right upper quadrant, and crampy left lower quadrant abdominal pain. He has alternating bouts of constipation and diarrhea. In addition, he states that blood coats and is mixed in with his stools. He sometimes has pain with defecation. There is mild hepatomegaly. The rectal exam reveals non-thrombosed external hemorrhoids. The stool is guaiac positive. A complete blood cell count (CBC) reveals a mild microcytic anemia. The MOST LIKELY cause for this patient's condition is ...

A. angiodysplasia

B. an anal fissure

C. diverticulitis

D. hemorrhoids

E. colon cancer

16. A 50 year old man presents with flushing of the face, watery diarrhea, weight loss, and multiple mass lesions in his liver. The primary site for the cancer that led to the above symptom complex is the ...

A. lung

B. liver

C. stomach

D. appendix

E. terminal ileum

.,

)

17. Hematochezia in elderly patients that cannot be identified with a barium enema study is most commonly due to a disorder located in the .••

A. rectum

B. stomach

C. sigmoid colon

D.. proximal duodenum

E. cecum and ascending colon

18. A febrile 58 year old man with a long history of chronic constipation complains of a steady pain in the left lower abdomen. He has had frequent attacks of pain in the same area for several months and one episode of bloody stools that spontaneously resolved. Physical exam reveals rebound tenderness and a palpable mass in the left lower quadrant. A stool guaiac is negative. A CBC exhibits an absolute neutrophilic leukocytosis and left shift. A flat plate is reported to show no air under the diaphragm. The most likely diagnosis is ..

A. volvulus

B. colon cancer

C. ischemic colitis

D. acute diverticulitis

E. small bowel infarction

19. A 58 yr old woman complains of epigastric pain, weight loss, vomiting of coffee ground-like material, and dark black sticky stools. Physical exam reveals epigastric pain to deep palpation and non-tender, hard left supraclavicular lymph nodes. Both ovaries are enlarged and firm on bimanual pelvic exam. Which of the following scenarios best explains the signs and symptoms in this

patient? .

A. Stomach cancer with metastasis to the ovaries

B. Primary ovarian cancer with metastasis to the stomach

C. Primary pancreatic cancer with metastasis to the ovaries

D. Malignant lymphoma of the stomach with metastasis to the ovaries

E. Primary ovarian cancer with metastasis to the supraclavicular lymph nodes

37

Note: This material is copyrighted. All rights reserved. (Edward F. Goljan, M.D.) 2001

20. A 65 year old man with a chronic arrhythmia with an irregularly irregular beat presents with a sudden onset of severe abdominal pain associated with vomiting, abdominal distention, and bloody diarrhea. Physical exam reveals hypotension, absent bowel sounds, no rebound tenderness, and a guaiac positive, liquid stool. A complete blood cell count exhibits a white blood cell count of 35,000 cells/uL with an absolute neutrophilic leukocytosis and left shift. The serum amylase is elevated. The most likely diagnosis in this patient is ...

A. ischemic colitis

B. hemorrhagic pancreatitis

C. acute appendicitis with perforation

D. acute diverticulitis with perforation

E. hemorrhagic infarction of the small bowel

1. A febrile 12 year old boy with a viral infection lapses into coma. Physical exam reveals papilledema and hepatomegaly. The serum ammonia and transaminases are elevated and the PT is prolonged. The patient MOST LIKELY has ...

A. viral hepatitis

B. Reye's syndrome

C. salicylate intoxication

D. acetaminophen toxicity

E. a.-) antitrypsin deficiency

21. A 42 year old woman complains of increasing pain and difficulty swallowing liquids and solids over the past several months. She has lost 15 pounds in the past three months and has occasionally experienced acute substernal pain and regurgitation of food into her mouth when lying down at ni ght a Esophageal manometry reveals aperistalsis of the lower esophagus and increased pressure in the lower esophageal sphincter (LES). Which of the following apply to this case? SELECT 2

A. Elevation in anti-centromere antibodies

B. Involves striated muscle in upper esophagus

C. Absence of ganglion cells in the LES myenteric plexus

D. Bird's beak appearance on barium study

E. Adenocarcinoma of the distal esophagus

BepatobUiary/pancreas questions

2. Which of the following serologic data best represents a patient who has recovered from hepatitis B?

HBsAg HBeAg Anti--HBc-IgM Anti--HBc-IgG Anti-HBs

A. negative negative positive negative negative
B. positive positive positive negative negative
c. negative negative negative positive positive
D. negative negative negative negative positive 3. An afebrile 42 year old migrant worker from the border between Texas and Mexico presents with bloody diarrhea and right upper quadrant pain. The patient MOST LIKELY has ...

A. amebiasis

B. echinococcosis

C. acute cholecystitis

D. ascending cholangitis

E. metastatic colon cancer

38

.•

Note: This material is copyrighted. All rights reserved. (Edward F. Goljan, M.D.) 2001

4. A 48-year-old alcoholic with cirrhosis and chronic pancreatitis has steatorrhea and a prolonged PT.

The PT does not correct to normal after giving an intramuscular injection of vitamin K.Y ou conclude that the patient MOST LIKELY has ...

A. vitamin K deficiency

B. a circulating anticoagulant c. an isolated factor deficiency

D. inadequate synthesis of coagulation factors

E. vitamin K deficiency secondary to malabsorption

5. A 45-year-old man presents with increased skin pigmentation, steatorrhea, and diabetes mellitus.

The mechanism MOST LIKELY responsible for this constellation of findings is ...

A. amyloidosis

B. alcoholic cirrhosis

c. a defect in iron metabolism

D. alpha-entitrypsin deficiency

E. a defect in copper metabolism

6.

In which of the following diseases would you expect a conjugated bilirubin >50% of the total bilirubin?

A. Gilbert's syndrome

B. Chronic viral hepatitis

C. Crigler-Najjar syndrome

D. Stone in the common bile duct

E. Extravascular hemolytic anemia

\ I

)

7. An afebrile 62 year old man with a history of alcoholism, smoking, and chronic pancreatitis presents with weight loss, a slow onset of painless jaundice, and a normocytic anemia. Physical exam reveals a palpable gallbladder and a light-colored stool. The patient MOST LIKELY has ...

A. hepatocellular carcinoma

B. carcinoma of the gallbladder

C. primary sclerosing cholangitis

D. a stone in the common bile duct

E. carcinoma of the head of pancreas

8. A 38-year-old woman presents with chronic liver disease, greenish-brown deposits in the limbus of the eye, and choreoathetotic movements. Expected findings in this patient include ••• SELECT 2

A. high total copper levels

B. increased percent iron saturation

C. anti-Hey IgG antibodies

D. low ceruloplasmin levels

E. degenerative disease in the lenticular nuclei

39

Note: This material is copyrighted. All rights reserved. (Edward F. Goljan, M.D.) 2001

9.A 55-year-old woman presents with generalized pruritus, non-tender hepatomegaly, and yellow papular lesions scattered over her skin. She has marked elevation of serum alkaline phosphatase and y-glutamyltransferase, a normal total bilirubin, slightly elevated serum transaminases, and a severe hyperlipidemia. An endoscopic retrograde cholangiopancreatograpby (ERCP) study of the common bile duct is negative for stones. Which of the following are expected fmdings in this patient? SELECT 3

A. HyperLtiglyceridemia

B. Biliary cirrhosis

C. Antimitochondrial antibodies

D. Increased serum IgG levels

E. Granulomatous inflanunation

10. Which of the following complications is a direct by-product of portal hypertension secondary to alcoholic cirrhosis? SELECT 3

A. Ascites

B. Gynecomastia

C. Esophageal varices

D. Hepatic encephalopathy

E. Periumbilical caput medusae

11. A 25 year old woman develops jaundice. The conjugated bilirubin fraction is > 50% of the total bilirubin and the serum alkaline phosphatase is markedly elevated. You suspect the patient is taking which of the following medications? SELECT 1

A. Isoniazid

B. Allopurinol

C. Tetracycline

D. Acetaminophen

E. Oral contraceptives

12. An afebrile high school wrestler develops jaundice, light colored stools, dark urine, and mild nontender hepatomegaly. The urine is negative for urobilinogen and positive for bilirubin. The serum alkaline phosphatase is markedly elevated. His hepatitis serology tests and heterophile antibody test are both negative. You suspect the patient has ... SELECT 1

A. a stone in the common bile duct

B. been taking anabolic steroids

C. infectious mononucleosis

D. non A, non-B hepatitis

E. chronic cholecystitis

13. A 42 year old Asian man with postnecrotic necrosis secondary to chronic hepatitis B has a low grade fever, weight loss, and a rapid development of ascites. A peritoneal tap reveals bloody ascitic fluid. The fluid WBC count is normal and bacteria are not present on Gram stain. The a-fetoprotein level is 500 nglmL (normal < 6 nglmL). The MOST LIKELY diagnosis is ... SELECT 1

A. primary gallbladder cancer

B. hepatocellular carcinoma

C. metastatic liver disease

D. spontaneous peritonitis

E. cholangiocarcinoma

40



---------------------- .~---

------~----

14. A patient living in a Basque community in Southern Arizona presents with a history of recurrent right upper quadrant pain. He is a sheepherder and has a dog that helps him keep the sheep together. An ultrasound of his liver reveals a cystic mass in the liver with calcifications in the lining of the cyst. Which of the following additional findings would you expect in this patient? SELECT 1

A. His dog is an intermediate host

B. He is a definitive host

C. He ate an infected sheep

D. His dog ate an infected sheep

Note: This material is copyrighted. All rights reserved. (Edward F. Goljan, M .. D.) 2001

15. A weightlifter develops a sudden onset of abdominal pain along with hypovolemic shock. At surgery, his abdominal cavity is filled up with blood. The cause of the intraabdominal bleed is most likely associated with which of the following? SELECT 2

A. Anabolic steroids

B. Hepatocellular carcinoma

C. Liver cell adenoma

D. Cavernous hemangioma in the liver

E. Straining at stool

\

I

)

16. A 35-year-old executive of an up and coming computer company develops a sudden onset of severe epigastric pain with radiation of the pain into the back. He describes the pain as knife-like. In addition, he feels nauseous and has vomited non-bile stained fluid on two occasions. Physical exam reveals tenderness in the epigastric area but no rebound tenderness. There is mild tender hepatomegaly. A flat plate of the abdomen reveals a dilated loop of small bowel in the left upper quadrant. An ultrasound reveals no stones in the gallbladder and a normal diameter of the common bile duct. A stool guaiac is negative. Serum AST is higher than serum ALT. What test is most indicated in this patient? SELECT 1

A. Upper gastrointestinal endoscopy

B. Serum amylase and/or lipase

C. HIDA (radionuclide) scan to RIO a cystic duct stone

D. Hepatitis serologies

E~ Surgical consult to RIO small bowel obstruction

Renal questions

1 * An 82 year oldman presents with lower back pain and complaints of problems with voiding urine.

There is point tenderness over the lower lumbar vertebra. His bladder is percussed at the level of the umbilicus. Which of the following tests or procedures is indicated as the first step in the management of this patient?

A.. Radionuclide bone scan

B. Prostate specific antigen

C. Digital rectal examination

D. Serum alkaline phosphatase

E. Transrectal ultrasound with biopsy

41

................................................. ----------------------------

iote: This material is copyrighted. All rights reserved. (Edward F. Goljan, M.D.) 2001

A 28-year-old man with a history of removal of a right cryptorchid testis as a child is noted to have a painless enlargement of the remaining testicle in the left scrotal sac. The testicle is enlarged and does not transilluminate. The patient most likely has a ...

A. hydrocele

B. seminoma

C, varicocele

D.yolk sac tumor

E. chorioc~cinorrm

A 66 year old man with a 45 pack year history of smoking presents with hematuria, fever, and a palpable mass in the left lower quadrant. A chest x-ray reveals multiple nodular masses in the lungs. These findings most strongly suggest which of the following diagnoses?

A. Miliary TB involving the kidneys

B. Renal cell carcinoma with lung metastasis

C. Primary lung cancer with metastasis to the kidney

D. Choriocarcinoma with metastasis to lungs and kidneys

E. Acute pyelonephritis with metastatic abscesses in the lung

A febrile 23 year old woman presents with an acute onset of right flank: pain, suprapubic discomfort, dysuria, and increased frequency of urination. The urinary sediment examination reveals clumps of leukocytes, WBC casts, occasional RBCs, and numerous motile bacteria. The mechanism of this patient's urinary condition is most closely related to ...

A. a renal stone

B. ascending infection

C. immunocomplex disease

D. drug-induced interstitial nephritis

E. hematogenous spread of infection to the kidneys

A 2S year old male presents with a history of hemoptysis and malaise. His blood pressure is 140/90 mm Hg, He has periorbital edema and smoky colored urine. Pertinent laboratory data include a serum urea nitrogen of 40 mg/dl, (7-18 mg/dL) and a creatinine of 4.0 mg/dL (0.6-1.2 mg/dL). Urinalysis shows 2+ protein, 3+ blood, RBCs too numerous to count, and RBC casts. A chest radiograph demonstrates opacities in both lung fields. The mechanism of this patient's lung and renal disease is most closely related to a ...

A. type I hypersensitivity reaction

B. type IT hypersensitivity reaction

C. type m hypersensitivity reaction

D. type IV hypersensitivity reaction

E. type IT and ill hypersensitivity reaction

42

..

__ ----~--..-r-. __

Note: This material is copyrighted. All rights reserved. (Edward F. Goljan, M.D.) 2001

Items 6-7

A. IgA glomerulonephritis

B. Membranous glomerulonephritis c. Focal segmental glomerulosclerosis

D. Rapidly progressive crescentic glomerulonephritis

E. Type I membranoproliferative glomerulonephritis type I

6. A 74 year old man with a 80 pack history of cigarette smoking presents with hemoptysis and shortness of breath. Radiographs of the chest demonstrate a left hilar mass. He also has generalized anasarca with 4+ proteinuria, hypercholesterolemia, and fatty casts in the urine.

7. A 34 year old male who is a mown drug seeker and heroin addict comes to the emergency department in an agitated state. He is restrained and you note multiple needle tracks on both arms. He also has severe pitting edema of the lower extremities. Urinalysis is positive for protein and shows fatty casts on urine sediment.

8.

A 10 year-old-adolescent boy presents with a unilateral, tender subareolar mass in the left breast. Physical exam is otherwise unremarkable. Which of the following would you recommend for this patient?

A. Serum gonadotropins

B. Chromosome analysis

c. Serum ~-hCG

D. Surgical consult

E. No treatment

I

I I'

~

i

9. A 12 year old child presents with dyspnea and hypertension. Physical exam reveals a palpable right lower quadrant mass in the abdomen and scattered sibilant rhonchi.in the lungs. A chest x-ray reveals multiple nodular masses in the lungs. These findings most strongly suggest which of the following diagnoses?

A. Wilm's tumor

B. Neuroblastoma

C. Carcinoid tumor

D. Renal adenocarcinoma

E. Dysplastic kidney disease

10. A normotensive 8 yr old boy, with a history of an upper respiratory infection 1 week ago, now presents with generalized pitting edema. The urinalysis reveals severe proteinuria, fatty casts, and oval fat bodies. His condition rapidly improves with high doses of corticosteroids. Which of the following glomerular changes would you expect in this patient?

A. Fusion of the podocytes .

B. Diffuse proliferative changes

C. Diffuse membranous changes

D. Subepithelial immune deposits

E. Subendothelial immune deposits

43

Note: This material is copyrighted. All rights reserved. (Edward F. Goljan, M.D.) 2001

11. A 72 yr old man presents with hypovolemic shock secondary to a ruptured abdominal aortic aneurysm. Within 24 hrs of surgery, he develops oliguria. He died 3 clays later. Which of the following laboratory abnormalities is most likely present in this patient?

A. White blood cell casts

B. BUN/creatinineratio > 15/1

C. Urine Osmolality> 800 mOsm/kg

D. Fractional excretion of sodium> 1

E. Random urine sodium < 20 mEq/L

12. A 32 yr old man with hypertension has a systolic click followed by a murmur. He has a family history of hypertension at an. early age often progressing into chronic renal failure requiring either dialysis or a renal transplant. A few of the family members have died from a stroke related to an intracerebral bleed or subarachnoid hemorrhage. The patient most likely has ...

A. adult polycystic kidney disease

B. renal artery stenosis

C. renal dysplasia

D. IgA glomerulonephritis

E. diabetic nephropathy

\

)

13. A 32 yr old man with Wegener's granulomatosis has hematuria. A urine cytology reveals numerous RBCs and occasional atypical transitional cells. Which of the following best explains the renal problems in this patient?

A. Renal disease associated with Wegener's granulomatosis

B. Chemotherapy agent the patient is taking to treat his disease

C.. Renal adenocarcinoma unrelated to his disease

D. Adenovirus infection

E. Lower urinary tract infection

14. A tall, obese 13-year-old boy has bilateral gynecomastia, decreased testicular volume for age, and sparse axillary and pubic hair. He has had the usual childhood infections except for mumps. A CT scan of the sella turcica is normal. A chromosome study is pending. Based on the physical findings and CT scan results, which of the following serum follicle stimulating hormone (FSH), serum luteinizing hormone (LH), and serum testosterone levels is expected in this patient?

Serum FSH Serum LH Serum testosterone Chromosomes

A. High Normal Normal 47
B. Normal High Low 46
C. High High Low 47
D. Normal Normal Normal 46
E. Low Low Low 47 44

Note: This material is copyrighted. All rights reserved. (Edward F. Goljan, M.D.) 2001

Gynecology questions

Items 1-4

A. Infiltrating ductal carcinoma

B. Lobular carcinoma

C. Fibrocystic change

D. Intraductal papilloma

E. Intraductal carcinoma

F. Paget's disease

1. A 28-year-old woman during her breast exam is noted to have a bloody discharge. No masses are palpable.

2. A 65 year old woman with a history of breast cancer in her mother is noted to have retraction of skin in the right upper outer quadrant when abducting her right arm during her annual clinical exam. An indurated 3-crn mass is palpated directly beneath the skin retraction. Non-tender, firm right axillary lymph nodes present in the lower axillary chain.

3. A 26 year old woman without any family history of breast cancer complains of pain and "lumpiness" in her breasts that progressively increases throughout her menstrual cycle and is relieved after menses is completed. Physical exam reveals a painful, ill-defmed mass in the left upper quadrant. No palpable axillary lymph nodes are present

4. A 65 yr old woman has a rash on the nipple and a subareolar mass.

5. A 58-year-old woman with a 20-year history of smoking has a history of breast cancer in her mother. Her diet is poor in fiber and rich in saturated fats. Menarche occurred at 13 years of age and menopause began at 52 years of age. She has had 3 children. She had a cervical conization at 28 years of age for severe cervical dysplasia and has since had normal cervical Pap smears. Which of the following is her greatest risk factor for breast cancer?

A. Age

B. Family history

C. Smoking history

D. Severe cervical dysplasia

E. Low fiber, high saturated fat diet

6. During her monthly self-breast exam, a 19-year-old woman notes a moveable, finn, slightly tender mass in the left upper outer quadrant. There is no skin retraction or nipple discharge. She is currently taking birth control pills .. Her older sister was recently given a diagnosis of breast cancer for which she opted for breast conserving therapy. You would expect a fine needle aspiration of the mass to reveal ...

A. an infiltrating ductal cancer

B. an intraductal papilloma

C. benign cyst fluid

D. a fibroadenoma

E. lobular cancer

45

------------=----- --. -----

Note: This material is copyrighted. All rights reserved. (Edward F. Goljan, M.D.) 2001

7. A 22 year old woman, gravida 1, para 0, presents to the outpatient office for a routine prenatal visit at 34 weeks' gestation. Her blood pressure is 150/95 nnn Hg. On urine dipstick, she has 1 + glucose and 2+ albumin. Her blood pressure on the initial prenatal visit at 14 weeks' gestation was 120/75 mm Hg, Her maternal grandfather has adult-onset diabetes. Her mother and maternal grandmother both have chronic hypertension. Which of the following is the most . likely explanation for the findings in this patient?

A. Preeclampsia

B. Molar pregnancy

C. Abruptio placenta

D. Gestational diabetes

E. Primary renal disease

8. A 1 7 year old girl presents with a sudden onset of abdominal pain. Physical examination reveals a tender mass in the left adnexa. A pregnancy test is negative. An x-ray exhibits a mass lesion of the left ovary with focal areas of calcification. Which of the following best characterizes the ovarian mass?

A. Benign surface-derived tumor

B. Malignant surface-derived tumor

C. Follicular cyst of the ovary

D. Benign germ cell tumor

E. Malignant germ cell tumor

9.

A 66 year old nulliparous woman presents with abdominal distention despite a poor appetite. She underwent menopause 1 7 years ago. A rectal exam reveals induration in the rectal pouch of Douglas. She has a fluid wave in the abdomen and a left-sided pleural effusion. A thoracentesis shows clumps of malignant cells that stain positive for CA 12S.Which of the following is the most likely diagnosis?

A. Metastatic uterine leiomyosarcomas

B. Metastatic endometrial cancer

C. Metastatic ovarian cancer

D. Metastatic cervical cancer

E. Metastatic stomach cancer

)

10 A 28 year old woman, gravida 1, para 0, at 12 weeks' gestation presents with painless vaginal bleeding. Her blood pressure is 160/95mm Hg, there is 3+ proteinuria, and the uterus is large for gestational age. Which of the following is the most likely diagnosis?

A. Gestational diabetes

B. Placenta praevia

C. Abruptio placenta

D. Molar pregnancy

E. Twin placenta

46

1----------- __ .... __

Note: This material is copyrighted. All rights reserved. (Edward F. Goljan, M.D.) 2001

11. Place the following neoplasms of the female genital tract in decreasing incidence (morbidity) and in

decreasing mortality.

1. Cervix

2. Ovary

3. Endometrium

M d.i M

orbi tty ortality
A. 2, 1, 3 3, 2, 1
B. 1, 3, 2 2, 3, 1
c. 3, 2, I 2, 1, 3
D. 3,2, 1 3, 1, 2
E. 3, 2, 1 3, 2, 1 12. A 51 year old woman with a long smoking history is hospitalized for treatment of right pyelonephritis. An intravenous pyelogram shows right-sided hydronephrosis and a dilated ureter. During pelvic examination, you detect a malodorous vaginal discharge. A finn, irregular right adnexal mass extends to the pelvic side wall. The patient experiences vaginal bleeding after examination. What is the most likely diagnosis?

A. Perinephric abscess due to acute pyelonephritis

B. Endometriosis involving the right ovary

C. Invasive cervical carcinoma

D. Invasive endometrial adenocarcinoma

E. Invasive ovarian adenocarcinoma

13. A 22-year-old moderately obese woman presents with infertility problems. She has a long history of oligomenorrhea. Physical exam reveals hirsutism on the face and anterior chest. Pelvic exam exhibits bilaterally enlarged ovaries. The following laboratory tests are available: serum LH increased, serum FSH low, LHIFSH ratio >3/1, serum total testosterone slightly increased, serum free testosterone increased, serum DHEA-sulfate normal, serum prolactin nonnaI, and the serum TSH is normal, Based on these fmdings, you strongly suspect that the patient has ...

A. bilateral androgen-secreting tumors of the ovaries

B. deficiency of gonadotropin-releasing hormone c. polycystic ovarian syndrome

D. adrenal Cushing's syndrome

E. a gonadotropin-secreting pituitary tumor

14. Which of the following patients is at greatest risk for developing endometrial cancer?

Present Age Menarche Menopause Children Miscellaneous

A. B.

45 54

11 12

51

none 2

Hypertension

Obese, took birth control pills

Type IT diabetes mellitus

Obese, diabetes mellitus

Obese, took birth control pills

c.

56

12

50

none

D.

58

11

53

none

E.

55

11

50

none

47

..

Note: This material is copyrighted. All rights reserved. (Edward F. Goljan, M.D.) 2001

15. In which of the following clinical situations, does the patient have the greatest risk for developing cervical cancer?

Onset of sexual Number of

Present age activity sexual partners Miscellaneous history

A. 20 years old 14 years old 11 Obese, on birth control pills

B. 19 years old 10 years old 2 On birth control pills, smoker

c. 19 years old 12 years old 5 On birth control pills, obese, smoker

D. 21 years old 10 years old 10 Intravenous drug abuser, smoker, on birth control pills

E. 21 years old 10 years old 4 Obese, smoker, current boyfriend is an intravenous drug abuser

16. A 25 yr old woman presents with a history of severe colicky dysmenorrhea, painful stooling during menses, and problems with infertility. Laparoscopy reveals scarring around both fallopian tubes, a 3 ern cyst in the right ovary, and "powder bum" appearing areas on the uterine ligaments and serosal surface of the uterus. The mechanism for this patient's constellation of problems is most likely related to ...

A. spread of endometrial carcinoma

B. seeding from a primary ovarian tumor

C. reverse menses through the fallopian tubes

D. complications related to pelvic inflammatory disease

E. endometrial glands and stroma in the myometrial tissue

17. A short-statured 16-year-old girl presents with a history of primary amenorrhea. Physical exam reveals poor breast development and sparse axillary and pubic hair. Her skin is redundant at the base of the neck. Pelvic exam exhibits a small uterus and a solid left ovarian mass. She has no withdrawal bleeding after a progesterone challenge. Which of the following laboratory test findings would you expect in this patient?

Serum FSH Serum LH Serum estradiol Chromosomes

A. High Normal Low 45
B. Normal High Low- 46
c. High High Low 45
D. Normal Normal Normal 46
E. Low Low Low 45
FSH = follicle stimulating hormone, LH = luteinizing hormone 48

_ ...

Note: This material is copyrighted. All rights reserved. (Edward F. Goljan, M.D.) 2001

Endocrine questions

Items 1-3

T4 RTJUITJ3R Fl'a-index TSH 1311
A. Increased Increased Increased Decreased Increased
B. Increased Decreased Normal Normal Not indicated
c. Increased Increased Increased Decreased Decreased
D. Decreased Increased Normal Normal Not indicated
[ E. Decreased Decreased Decreased Increased Not indicated
. . . . 131 _ . . . .
- - RT3U- resm T3 uptake, T4BR - 14 binding ratio, I - radioactive iodine uptake

1. A 45-year-old obese woman presents with a history of weight loss, weakness, and palpitations that keep her awake at night. She states that she has lost over 50 pounds the last 8 months while attending a weight loss clinic. Physical exam reveals lid stare, a non-palpable thyroid gland, brisk deep tendon reflexes, systolic hypertension, and an irregularly irregular pulse. She is not taking any prescription medications other than the packet of pills given to her by the clinic.

2. A 22-year-old woman complains of intermittent fluttering in her chest. Physical exam reveals a normal thyroid, no lid stare or exophthalmos, a regular heart rate of 108 beats/minute, normal deep tendon reflexes, and blood pressure of 100/80 mm Hg. Amid-systolic click and murmur is heard at the apex that increases with expiration. She is currently taking birth control pills .

. " ..

3.

A 28-year-old woman presents with complaints of chronic constipation and progressive weight gain over the last 6 months in spite of being on a pure vegan diet .. She is currently on no prescription or over-the-counter medications. Physical exam exhibits a pale young woman with periorbital puffiness, dry, yellow-colored skin, normal sclera, a normal cardiovascular and respiratory exam, delayed deep tendon reflexes, and proximal muscle weakness in her lower extremities.

)

Items 4-5

PTH

B C

A D

Serum Calcium

Note: the square represents normal values

4. A 55-year-old woman with diastolic hypertension presents with a sudden onset of right flank pain with radiation of pain into the right groin. A urinalysis reveals a positive dipstick for blood and a negative dipstick for nitrite and leukocyte esterase. Sediment exam exhibits numerous RBCs and square crystals resembling the back of an envelope.

5. A newborn presents with tetany, heart failure, and an absent thymic shadow.

49



Note: This material is copyrighted. All rights reserved. (Edward F. GoIjan, M.D.) 2001

6. Which of the following clinical and laboratory abnormalities occur in BOTH Addison's disease and panhypopituitarism?

A. Low l l-deoxycortisol post-metyrapone test

B. Normal urine for 17-ketosteroids

C. Hypematremia

D. Hyperkalemia

E. LowTSH

7. Which of the following alterations in lipid metabolism is expected in a patient with diabetic ketoacidosis?

A. Activation of capillary lipoprotein lipase

B. Increased J3-oxidation of fatty acids

C. Decreased production of acetyl-CaA

D. Increased fatty acid synthesis

E. Inhibition of hormone sensitive lipase

\

)

8. A 22-year-old woman has been amenorrheic for the last 6 months. She complains of a milky discharge from her nipples that has been present for the last 7 months. The pregnancy test is negative and the serum TSH is normal. Which of the following additional tests should you order on this patient?

A. Serum prolactin

B. Metyrapone test

C. Serum T.

D. Serum cortisol

E. Serum gonadotropins

9. Which of the following tests distinguishes pituitary Cushings from both adrenal and ectopic Cushings?

A. 24-hour urine for 17 -ketosteroids

B.. Low dose dexamethasone test

C. Serum cortisol level

D. High dose dexamethasone suppression test

E. 24-hour urine for free cortisol

10. A 35-year-old pharmacist presents to your office with recurrent episodes of forgetfulness and tiredness. A serum glucose is reported to be 20 mg/dL (70-110 mg/dL). Additional studies on the same sample reveal a high serum insulin and high C-pepide level. Based on these findings, you suspect the patient has ...

A. a benign tumor involving J3-islet cells

B. an early phase of type I diabetes mellitus

C. a benign tumor involving a-islet cells

D. ectopic secretion of an insulin-like factor

E. surreptitiously injected human insulin

50

..

Note: This material is copyrighted. All rights reserved. (Edward F. Goljan, M.D.) 2001

11. A 39 year old type I insulin dependent diabetic has a burning sensation around his ankles and on the bottoms of both feet. Neurological examination reveals depressed Achilles and knee jerk reflexes bilaterally and decreased light touch sensation in both lower extremities. The mechanism for this is most closely associated with ...

A. osmotic damage

B. thiamine deficiency

C. pernicious anemia

D. syringomyelia

E. lumbar disk disease

12. Which of the following water deprivation test results would you expect in a patient with polyuria after head trauma?

POsm post water UOsmpost UOsmpost
deprivation water dq>rivation vasopressm
I normal 292 750 760
A. 312 98 120
B. 319 110 550
C. 288 760 780 \ )

13. Which of the following most clearly differentiates type I from type IT diabetes mellitus?

A. Insulitis

B. Positive family history

C. Hypoglycemic reactions

D. Post-insulin receptor defects

E. Increased glycosylated hemoglobin

14. An 8-year-old boy presents with headache and bilateral hemianopsia. A cr scan of the skull reveals enlargement of the sella turcica secondary to a cystic mass with multiple calcifications. Which of the following is an expected laboratory finding in this patient?

A.Low insulin growth factor-I levels

B. Hyponatremia and hyperkalemia

c. Normal urine for 17-hydroxycorticoids

D. High serumTSH and low serumT,

E. Normal serum l l-deoxycortisol after metyrapone

15. A 48-year-old man with diastolic hypertension complains of headaches, drenching night sweats, and palpitations that keep him awake at night. He has a family history of hypertension. Physical exam findings include an anxious man with a mean blood pressure of 152196 mm Hg and a regular pulse of 148 beats/minute. There is no evidence of tbyromegaly. Deep tendon reflexes are brisk. Which of the following tests is the best screening test for this patient?

A. Serum thyroid stimulating hormone

B.. 24-br urine for metanephrines

c. 24-hr urine for free cortisol

D. Plasma aldosterone

E_ Metyrapone test

51

Note: This material is copyrighted. All rights reserved. (Edward F. Goljan, M.D.) 2001

16. A 2 year old child with newly discovered diastolic hypertension has multiple nodular lesions on the skin. A biopsy of one of the lesions exhibits an infiltrate of small, round, hyperchromatic cells. The cells stain positive for S 100 antigen. A radionuclide bone scan is reported to have multiple lucencies consistent with metastatic disease. The primary origin of the patient's skin and marrow infiltrate is from which of the following sites?

A. Skin

B. Bone

C. Kidney

D. Thyroid

E~ Adrenal cortex

F. Adrenal medulla

17. Which of the following biochemical reactions is most responsible for hyperglycemia in a patient with diabetic ketoacidosis?

A. Glycogen ...•.•....•...••..• _.... glucose

B. Glycero13-phosphate ~ DHAP· .. ·· ........ · .... ·_.... glucose

C. Lactate __ pyruvate _.... glucose

D. Alanine ~ pyruvate _... glucose

E. Citrate ~ Oxaloacetate II- Malate ... nn •• _"" glucose + Acetyl CoA

DHAP - dihydroxyacetone phosphate

)

18. A 26 yr old woman with an abruptio placenta develops hypovolemic shock requiring multiple units of packed red blood cells. Her baby is delivered without consequence. Two days later, the mother experiences a sudden cessation of lactation. The pathologic process most likely responsible for this complication is ....

A. hemorrhage into a preexisting pituitary adenoma

B.. increased secretion of dopamine

C. coagulation necrosis of the anterior pituitary

D. benign pituitary adenoma with secretion of prolactin

E. hypothalamic dysftmction leading to hypopituitarism

19. A 22 yr old type I diabetic develops sweating, anxiety, excessive hunger, and light-headedness. The first step in the management of this patient is to ......

A. order a plasma HgbAiC

B. order a serum insulin and C-peptide level

C. order serum electrolytes

D. order a 24 hr urine for metanephrines

E. give the patient a candy bar

20. A 25 yr old man with non-tender thyromegaly and palpable cervical lymph nodes has an elevated serum calcitonin level. A fine needle aspiration of the thyroid is scheduled. He has a family history of thyroid disease, hypertension, and renal stones. Which of the following additional laboratory tests should be ordered on this patient?

A. Serum electrolytes and 24 hr urine for free cortisol

B. 24 hr urine for metanephrine and serum PTHIcalcium

C. Serum aldosterone and 24 hr urine for potassium

D. Serum gastrin and gastric analysis to measure basal acid output

E.. Serum vasointestinal peptide and somatostatin levels

52

Note: This material is copyrighted. All rights reserved. (Edward F. Goljan, M.D.) 2001

21. A 21 yr old man with a history of infertility is noted to have a large varicocele in the left scrotal sac.

His physician suspects a problem with the seminiferous tubules. Which of the following sets of laboratory test results would you expect in this patient?

Serum Serum Serum

FSH LH testosterone Sperm count

A. High Normal Normal Decreased

B. Normal High Low Decreased

C. High High Low Decreased

D. Normal Normal Normal Normal

22. A 45-year-old man with diastolic hypertension presents with muscle weakness and tetany.

Laboratory studies reveal mild hypernatremia, hypokalemia, and metabolic alkalosis. There is no evidence of pitting edema or volume depletion. The patient most likely has ...

A. Addison's disease

B. chronic renal failure c. primary aldosteronism

D. been taking a loop diuretic

E. type I renal tubular acidosis

Musculoskeletal and autoimmune questions

1. A sexually active 30 year old man presents with fever and pain in his right big toe that woke him up at night. He has been taking aspirin to relieve the pain without relief. His mother has severe osteoarthritis. The right toe is swollen, hot and exquisitely sensitive to touch. Laboratory studies reveal a neutrophilic leukocytosis and left shift. A synovial tap was performed, Based on the above history, his problem is most likely related to ...

A. joint inflammation secondary to a positively birefringent crystal

B. disseminated gonococcemia with septic arthritis

C. underexcretion of uric acid in the urine

D. osteomyelitis secondary to hematogenous spread of Staphylococcus aureus

E. an HLA B27 positive spondyloarthropatby

2. Which of the following characterizes joint disease associated with rheumatoid arthritis rather than osteoarthritis?

A. Cartilage fibrillation

B. Subchondral bone cysts

C. Osteophytes

D. Ankylosis of the joint

E. Bouchard's nodes

3. A 28 year old man who works in a summer camp in upstate New York presents with bilateral facial weakness involving both tbeupper and lower facial muscles. He also complains of joint pains in both knees, He has a history a few weeks ago of having had a peculiar rash on his right thigh that was circular, red colored and had a central area of clearing. The patient's disease is most likely caused by a ...

A. gram negative diplococcus

B. spirochete

c. gram positive coccus

D. virus

E. immune complex reaction

S3

Note: This material is copyrighted. All rights reserved. (Edward F. Goljan, M.D.) 2001

4. A febrile, sexually active 23 year old woman presents with a hot, swollen right knee and pustular lesions on the palm of her left hand. She recently returned from a camping trip in Colorado. A gram stain of the synovial fluid in the knee and from an aspirate of one of the pustules revealed 8 ...

A. gram positive diplococcus

B. gram negative rod

C. spirochete

D. gram negative diplococcus

E. intranuclear inclusion

s. A veterinarian develops tenosynovitis within 24 .. hrs of an animal bite. You suspect the offending

• •

organIsm IS ...

A. Yersinia pestis

B. Eikenella corrodens

C. Fusobacterium

D. Pasteurella multocida

E. Staphylococcus aureus

6. Which of the following clinicopathologic findings is commonly present in both rheumatoid arthritis and ankylosing spondylitis?

A. Sacroiliitis

B. Male dominance

C. Ankylosis of the joint

D. Overgrowth of synovial tissue

E. Autoantibodies against immunoglobulins

7.A 36 year old man, who has had problems with his lower back since his early twenties, presents with blurry vision in his right eye. Physical exam reveals a ciliary flush and apoor pupillary response to light in the right eye. The intraocular pressure is normal. He also has reduced anterior flexion and dry inspiratory crackles at both lung bases. The patient is presently taking indomethacin for back pain. You would expect an x-ray of the patient's back to reveal ...

A. radiodense lesions in the vertebra

B. osteophytes at the joint margins c. vertebral compression fractures

D. lytic lesions in the vertebra

E. fusion of the vertebra

8.

A 28 year old man has a ten year history of intermittent left lower quadrant pain associated with bloody diarrhea. He now complains of symmetrical joint pains in the knees, wrists, and ankles, as well as in the lower back. Which of the following is the inciting agent most likely responsible for his current rheumatologic problems?

A. Overproduction of uric acid

B. Invasive gastroenteritis

C. Colorectal cancer

D. Ulcerative colitis

E. Crobn's disease

f

(

54

« f f C t f ,

f

Note: This material is copyrighted. All rights reserved. (Edward F. Goljan, M.D.) 2001

\

9. A 3 year old girl has an acute onset of spiking fever, rash, generalized painful lymphadenopathy, hepatosplenomegaly, and polyarthritis. The cardiac exam is normal, A serum antinuclear antibody test, rheumatoid factor, and urinalysis are reported as negative. A CBC reveals an absolute neutrophilic leukocytosis, mild normocytic anemia, and a normal platelet COWlt. Which of the following best characterizes the nature of this patient's disease?

A. Non-disabling, non-infectious inflammatory joint disease

B. Progressively disabling infectious joint disease due to a bacteria

C. Progressively disabling non-infectious inflammatory joint disease

D. Non-disabling infectious joint disease due to an immunologic reaction against a bacteria

E. Progressively disabling non-infectious inflammatory joint disease associated with an IUAB27 haplotype

10. Which of the following clinicopathologic findings occurs in both osteoarthritis and rheumatoid

arthri . ? ns',

A. Blurry vision

B. Pannus formation c. Narrow joint space

D. Subchondral bone cysts

E. Metacarpophalangeal joint involvement

11. A 45 year old man has chronic myelogenous leukemia that has progressed into an acute blast crisis.

He is currently being treated with multiple antileukemic agents. One week into therapy, he develops oliguric renal failure. A urinalysis reveals an acid pH and numerous crystals. Which of the following best explains the mechanism for this patient's renal failure?

A. Drug nephrotoxicity

B. Overproduction of uric acid

C. Leukemic infiltration of the kidneys

D. Underactivity of 5~phosphoribosyl-l-pyrophosphate (pRPP)

E. Overactivity of hypoxanthine-guanine phosphoribosyltransferase (HGPRT)

12. A 65-year-old man enters the examining room with broad-based ataxia .. Positive neurological fmdings include a positive Romberg test, loss of pain and vibratory sensation in the lower extremities, and absent deep tendon reflexes in the lower extremities. Both pupils accommodate but do not constrict with direct light stimulation. The patient's right knee is swollen and soft tissue swelling is noted around the joint. An x-ray reveals extensive resorption of bone. Which of the following best explains the pathogenesis of this patient's joint disease?

A. Septic arthritis

B. Loss of pain sensation c. Immunocomplex disease

D. Upper motor neuron disease

E. Cavitary lesion in the cervical spinal cord

13. Disabling j oint disease, nodular lesions in the lung associated with dust borne diseases, xerostomia, and splenomegaly characterize a rheumatologic disease with which one of the following laboratory abnormalities?

A. Positive serum antinuclear antibody with a rim pattern

B. Positive band test on a skin biopsy

C. Anti-ribonucleoprotein antibodies

D. Anti-centromere antibodies

E. IgM antibodies against IgG

55

~-------------------------------------

""I

Note: This material is copyrighted. All rights reserved. (Edward F. Goljan, M.D.) 2001

, ,

t •

14. A 32 year old pregnant woman, who is under treatment for hypertension with hydralazine, develops an abrupt onset of a left-sided pleural effusion, photophobia, and joint pains in both of her hands. Which of the following antibodies is most likely present in this patient?

A. Anti-double stranded DNA

B. Anti-centromere

C. Anti-histone

D. Anti-Scl-70

E. Anti-Sm

,



15. A 24 year old woman with a previous history of morning stiffness of both hands presents with dyspnea, neck vein distention with inspiration, and muffled heart sounds. Urinalysis reveals RBC casts, hematuria, and mild to moderate proteinuria. A serum antinuclear antibody test is positive and has a titer of 1: 1280. Based on these findings, the patient most likely has ...

, A. mixed connective tissue disorder

B.. progressive systemic sclerosis

C. systemic lupus erythematosus

D. juvenile rheumatoid arthritis

E. adult rheumatoid arthritis

16. Which of the following clinicopathologic findings is present in both systemic lupus erythematosus and Sjogren's syndrome?

A. Sclerodactyly

B. Positive syphilis serology

C. Anti-SS-A (Ro) antibodies

D. Non-infectious endocarditis

E. Dysphagia for solids and liquids

17. Which of the following relationships are correctly matched? SELECT 5

Note: this is not a style of question on the USMLE, however it is an excellent learning tool for commonly asked small topics

A. Osteomyelitis Most common cause in sickle cell disease is Staphylococcus

B.

Osteomyelitis

aureus

Most common site is the metaphysis. Most common mechanism for spread to bone in children is local extension from a soft tissue abscess.

Orifice of sinus tract is subject to developing squamous cell

c.

Chronic osteomyelitis

D. F.

.

carcmoma

Staphylococcus aureus involving the vertebral colunm

Most common late manifestation is arthritis. Bilateral Bell's palsy may occur.

AD disorder with an osteoblastic defect. Too little bone. Pathologic fractures, dearness.

AD disorder with a defect in synthesis of type I collagen. Blue sclera due to underlying choroidal veins. Pathologic fractures.

Most common cause in women is estrogen lack. More bone is lost by osteoclastic activity than replaced by osteoblastic activity.

Most common fractures are vertebral compression fractures Routine bone x-rays are the most accurate method for diagnosing the condition

56

Pott's disease Lyme's disease

G.

Osteopetrosis

H.

Osteogenesis Imperfecta

I.

Osteoporosis

J. K.

Osteoporosis Osteoporosis

Note: This material is copyrighted. All rights reserved. (Edward F. Goljan, M.D.) 2001

\

18. Which of the following relationships are correctly matched? SELECT 5

A. Benefits of estrogen in Maintains the high HDL and low LDL. Increases bone

osteoporosis density rather than preventing loss of bone density. Danger of endometrial cancer if progesterone is not added.

B. Femoral neck fracture Danger of aseptic necrosis from compromise of medial femoral circumflex artery by a posterior dislocation

C. Scaphoid bone fracture Danger of osteomyelitis. Pain in the anatomical snuffbox.

D. Colles' fracture Dinner fork deformity of the distal radius. Common fracture when falling on the outstretched hand.

E. Supracondylar fracture Fracture of the distal humerus. Median nerve may be injured.

Brachial artery compromised leading to possible ischemic contracture of forearm muscles.

F. Aseptic necrosis Most conunon cause is sickle cell disease. Bone x-ray reveals increased density. l\1RI is the most sensitive test.

G. Legg-Perthes Osteomyelitis of the femoral head in children < 10 yrs old

H. Osgood-Schlatter's disease Inflammation of the proximal tibial apophysis with repair resulting in knobby knees. Leads to problems with bone growth in the leg.

19. Which of the following relationships are correctly matched? SELECT 5

A. Paget's disease of Bone thickens and fractures easily. Serum alkaline phosphatase is

bone elevated. Complications include osteogenic sarcoma, pathologic fractures, high output cardiac failure.

B. Osteochondroma Most conunon overall benign bone tumor

C. Ewing's sarcoma "Round (small) cell tumor" with x-ray findings of "onion skinning". Primarily affects young adults.

D. Chondrosarcoma Most common malignant cartilaginous tumor. Grade determines it biologic behavior. Most cormnonly affects pelvic bones.

E. Osteoma Radiograph demonstrates a radiolucent defect surrounded by densely sclerotic bone. Nocturnal pain is relieved by aspirin

F. Osteogenic sarcoma Association with Rb suppressor gene on chromosome 13.

"Sunburst appearance" and "Codman's triangle" on x-ray. Primarily affects the pelvic bones.

G. Muscle weakness Causes include upper/lower motor neuron disease, primary muscle disease, myasthenia gravis, and extrapyramidal disease

H. Dupuytren's Fibromatosis involving palmar tendon sheaths. Related to

contracture hyperestrinism in cirrhosis.

I. Desmoid tumor Fibrosarcoma of the abdominal wall often associated with Turcot's syndrome

57

Note: This material is copyrighted. All rights reserved. (Edward F. Goljan, M.D.) 2001

20. Which of the following relationships are correctly matched? SELECT 2

A. Embryonalrhabdomyosarcoma Most common adult striated muscle sarcoma, Presents

as a vaginal mass in women or bladder/prostate mass in

B. c.

Lipoma Leiomyoma

men.

Most common benign soft tissue tumor in women

Most common benign soft tissue tumor of the uterus and gastrointestinal tract

Most common benign tumor of the heart in children . with tuberous sclerosis

Most common childhood soft tissue sarcoma. Associated with radiation exposure and scars

D.

Rhabdomyomas

E.

Malignant fibrous histiocytoma

Skin questions

1. A 52-year-old man with congestive heart failure develops cough and swelling in the deep subcutaneous tissue. He is most likely taking ...

A. a thiazide diuretic

B. an ACE inhibitor

C. apbenotlUarine

D. digitalis

E. a calcium channel blocker

2. The most common fungal cause of tinea capitis with a negative Wood's lamp is ...

A. Trichophyton tonsurans

B. Microsporum canis

C. Trichophyton rubrum

D. Candida albicans

E. Malasseziafurfur

3. A 43-year-old woman presents with joint pains and a butterfly-like rash on the face. She is most likely taking ...

A. thiazides

B. oral contraceptives

C. hydralazine

D. doxycycline

E. barbiturates

4. A 42-year-old man has recurrent development of vesicular and bullous lesions in sun-exposed areas. He has had to avoid alcohol, because it seems to coincide with these episodes. You would expect this patient to have ...

A. a history of abdominal pain

B. an increase in &-aminolevulinic acid in his mine C.an increase in porphobilinogen in his urine

D. colorless urine during these attacks

E. a decrease in red blood cell uroporphyrinogen decarboxylase

58

Note: This material is copyrighted. All rights reserved. (Edward F. Goljan, M.D.) 2001

5. A 30-year-old female with a long history of alcohol and barbiturate abuse presents with diffuse colicky abdominal pain. Examination reveals numerous surgical scars on the abdomen. There is no history of photosensitivity. You suspect that her urine would ...

A~ contain an increase in uroporphyrin

B. contain an increase in coproporphyrin

c. be positive for blood

D. have an excess amount of urobilinogen

E. tum a port wine color after exposure to light

6. A baseball player develops a vesicular rash around the neck and on his forearms. He is most likely taking which of the following drugs?

A. Thiazide diuretic

B. ACE inhibitor

C. Tetracycline

D. Penicillin

E. Calcium channel blocker

7. A 25 yr old sexually active man has a verrucoid appearing lesion on the shaft of his penis.

The infectious agent that is responsible for this penile lesion belongs to which of the following subtypes of microbial pathogens?

A. Spirochete

B. Rickettsia

C. Chlamydia

D. Fungus

E. Virus

8. A 35 yr old black man has scaly lesions on his scalp and raised, plaque-like lesions on his elbow and trunk. The lesions have silver-colored scales on the surface. Which of the following is a common clinical finding associated with this skin lesion?

A. Positive Nikolsky sign

B. Nail pitting

C. Dermatographism

D. Wickham's stria

E. Squamous cell carcinoma

9. Which of the following most determines the prognosis ofa superficial spreading malignant melanoma?

A. Gender of the patient

B. Depth of invasion

C. Duration of exposure to sunlight

D. Size and color variation

E. Presence of a radial growth phase

59

.-.-._ --"_ - - ~-----

--------------------------......_

,::.'---

Note: This material is copyrighted. All rights reserved. (Edward F. Goljan, M.D.) 2001

10. An 4 month old infant has a weeping, eczematous rash on the cheeks. The child is a mouth breather and constantly has a stuffy nose and otitis media infections. Which of the following characterizes the pathogenesis of this lesion?

A. Autoantibodies against DNA

B. Type I hypersensitivity reaction

C. Type m hypersensitivity reaction

D. Type IV hypersensitivity reaction

E. Superficial dermatophyte infection

11. A 49 yrold woman has a scaly, eczematous skin lesion related to the nickel in the metal clip on her girdle. The pathogenesis of the skin lesion is similar to the pathogenesis of ...

A. a wheal and flare reaction associated with a bee sting

B. a maculopapular rash on a patient taking penicillin

C.. palpable purpura ina small vessel vasculitis

D. the rash in poison ivy

E. tinea corporis

12. A 25 )T old woman is noted to have pruritic circular lesions on her leg. They have a red, scaly border and a central area of clearing. Which of the following should be your first step in the diagnostic work-up of this patient?

A. Punch biopsy

B.KOH preparation

C. Tzanck preparation

D. Peripheral blood count for eosinophils

E. Urine sample for porphyrin analysis

13. A 4yr old child has multiple craterifonn lesions with "sandy-like" material in the area of umbilication located over the trunk. Which of the following best describes the pathogenesis of these lesions?

A. Viral infection

B. Ultraviolet light

C. Fungal infection

D. Bacterial infection

E. Type I hypersensitivity reaction

14. A 53 yr old farmer has raised, pearly gray lesions on the dorsum of his hands and forearms. Which of the following best describes the pathogenesis of the pearly white lesion on the hand?

A. Viral infection

B. Ultraviolet light

C. Fungal infection

D. Bacterial infection

E. Type I hypersensitivity reaction

60

...

\

Note: This material is copyrighted. All rights reserved. (EdwardF. Goljan, M.D.) 2001

15. A 25 yr old woman complains of multiple areas of skin that do not tan during the summer. Physical examination reveals areas of hyperpigmentation and hypopigmentation on her chest and back. A KOH preparation taken from a the border of a hypopigmented skin lesion reveals yeasts and hyphae. Which of the following skin disorder groups is associated with the pathogen noted in the KOH preparation?

A. Tinea capitis: Pityriasis rosea

B. Seborrheic dermatitis : Tinea versicolor

C. Tinea corporis: Onychomycosis

D. Pityriasis rosea : Tinea versicolor

E. Onychomycosis : Seborrheic dermatitis

16. A 26 yr old man presents with slightly pruritic oval ... shaped lesion on the trunk that has erythematous borders and a pale center. A few days later, a rash develops on the trunk that follows the lines of Langer. The patient most likely has ...

A. a superficial dermatophytoses

B. a viral exanthem

C. pityriasis rosea

D. an urticarial reaction

E. contact dermatitis

17. A febrile 8 yr old boy has a rash on the face, trunk, and extremities. Some of the lesions are flat, while others are vesicular, and others are pustular. A Tzanck prep is performed and reveals multinucleated squamous cells with intranuclear inclusions. The patient most likely has ...

A. an HSV -1 infection

B. varicella

C. variola

D. Herpes zoster

E. drug reaction

18. A 72 yr old man with chronic lymphocytic leukemia has a painful, vesicular rash located on the shoulder. It begins at the midline of the neck and extends over the deltoid area. The pathogenesis of the skin lesion in this patient is attributable to ...

A. reactivation of a latent viral infection

B. an allergic contact dermatitis

C. an ultraviolet light-related dermatitis

D. a group A streptococcus infection

E. invasion of skin by leukemic cells

19. A 16 yr old boy presents with yellow, crusted lesions over the face, neck, and upper body. Ten days after the rash, the patient develops periorbital puffiness, hypertension, oliguria, and smoky colored urine with hematuria and RBC casts. The pathogenesis of the hypertension and urinalysis finding in the patient is most closely associated with ...

A. sepsis due to Streptococcus pneumoniae

B. sepsis due to Streptococcus pyogenes

C. sepsis due to Staphylococcus aureus

D. an immunocornplex disease associated with group A streptococcus

E. an immunocomplex disease associated with antibodies against DNA

61

~---~-----

Note: This material is copyrighted. All rights reserved. (Edward F. Goljan, M.D.) 2001

20. A 17 yr old male presents with facial lesions characterized by erythematous papular lesions with white and black centers. Some of the lesions have a nodular-cystic pattern. The pathogenesis of the inflammatory component of the facial lesions in this patient is most closely associated with ...

A. a group A streptococcal infection

B. folliculitis secondary to Staphylococcus aureus

C. an immunocomplex disease associated with antibodies against DNA

D. production of lipases by Propionibacterium acnes

E. folliculitis secondary to Streptococcus pyogenes

eNS and special senses questions

1. Cognitive abnormalities, ataxia, nystagmus, ophthalmoplegia, and foot drop are all associated with .. 0

A. folate deficiency

B. multiple sclerosis

C. Alzheimer's disease

D. alcoholism

E. Parkinson's disease

2. A family history of chronic liver disease beginning at an early age and a movement disorder developing later in life characterizes a disease associated with ...

A. a triplet repeat mutation

B. low ceruloplasmin levels c. thiamine deficiency

D. excess alcohol intake

E. vitamin B12 deficiency

3. A 40-year-old neuropathologist developed a rapidly progressive dementia and died. The pathogenesis of his disorder is related to ...

A. a decrease in acetylcholine levels

B. a deficiency of dopamine

C. neuronal damage by amyloid

D. a slow virus diseaseinvolvingprions

E. subacute sclerosing panencephalitis

4. A 15-year-old boy develops fever, nuchal rigidity, and petechial lesions. A spinal fluid reveals increased protein, decreased glucose, increased neutrophils, and a positive gram stain. You would expect the CSF gram stain to reveal. ..

A. gram positive diplococci

B. gram negative coccobacilli

C. gram positive rods

D. gram negative diplococci

E. gram positive cocci

5. Which of the following represents a primary brain tumor that would more likely develop ina child rather than an adult?

A. Glioblastoma multiforme

B. Medulloblastoma

C. Malignant lymphoma

D. Acousticneuroma

E. Meningioma

62



"" ... _ ..... ,_ .. - p..-----------------------------

Note: '[his matenai is copyng.iteu. Ail rights reservcu, ~n(1vv'aro 1<, Ul)lJa,,~, .i..loLl.) LvJ 1

6. An afebrile 52 year old smoker with weight loss, cough, and hemoptysis, develops a severe headache associated with unilateral lid lag, ophthalmoplegia, and mydriasis. An MRI reveals multiple densities in the cerebral cortex. The pathogenesis of this patient's neurologic condition most closely relates to. a.

A. a demyelinating disease

B a primary CNS tumor

C. embolic stroke

D. cerebral edema with uncal herniation

E. cerebellar herniation into the foramen magnum

7. An autopsy is performed on a 40-year-old man with dementia. Examination of the brain reveals atrophy of the frontal and temporoparietal lobes and senile plaques on histologic examination of the tissue. The pathogenesis of the eNS disease in this patient is most closely related to ...

A. J3-amyloid protein

B. lacunar infarcts

C. decreased dopamine levels

D. a slow virus disease

E. a triplet repeat disorder

\ )

8. Recurrent episodes of blurry vision, scanning speech, and parestbesias is most closely associated with ...

A. multiple sclerosis

B. Wernicke's encephalopathy

C. slow virus disease

D. Huntington's disease

E. Wilson's disease

9. An encapsulated mass is removed from the right cerebellopontine angle of a 32-year-old male with right-sided sensorineural bearing loss and facial numbness in the distribution of the trigeminal nerve. The patient most likely has alan ...

A. ependymoma

B. medulloblastoma

c. demyelinating disease

D. meningioma

E. acoustic neuroma

10. A 45-year-old woman with a history of mitral stenosis and a chronic arrhythmia died suddenly at home. At autopsy, a hemorrhagic lesion was noted at the periphery of the temporal lobe. The patient most likely has alan ...

A. atherosclerotic stroke

B. intracerebral hematoma

C. embolic stroke

D. glioblastoma multiforme

E. A V malformation

63 _

Note: This material is copyrighted. All rights reserved. (Edward F. Goljan, M.D.) 2001

11_ A 62-year-old man complains of weakness in his left arm and tremor in his hands that prevents him from writing legibly. He has a shuffling gait when he enters the examining room. His deep tendon reflexes are normal. The patient's clinical findings most closely relate to •.

A. amyotrophic lateral sclerosis

B. atherosclerotic stroke

C. pure motor stroke

D. Parkinson's disease

E. post-poliomyelitis syndrome

12. A 30-year-old woman states that she burns her hands without feeling any pain. Physical exam reveals decreased pain and temperature sensation in the upper extremities, atrophy of the intrinsic muscles of his hand, and abnormal deep tendon reflexes in the upper extremity. The patient most likely has ...

A. syringomyelia

B. multiple sclerosis

C. amyotrophic lateral sclerosis

D. spinal cord tumor

E. Bl2 deficiency

\ \

)

13. A febrile 28-year-old man with AIDS and a CD4 T helper count of 50 cells/uL develops focal epileptic seizures. A CT scan reveals multiple ring enhancing lesions in the brain. The pathogenesis of this patient's eNS disorder is most closely related to ...

J\. cy1onaegalo~s

B. cryptococcosis

C. toxoplasmosis

D. Epstein-Barr virus

E. disseminated MAl

14. Which of the following is more often associated with Escherichia coli than Pseudomonas aeruginosal

A. Osteomyelitis from puncture wounds through rubber footwear

B. Malignant otitis extema in a diabetic patient

C. Pneumonia in an intensive care unit

D.. Death in a patient in a bum unit

E. Meningitis in a newborn

15. A 55 year old man has broad-based ataxia and loss of pain and temperature sensation in the lower extremities .. Both pupils accommodate but do not constrict with direct light stimulation. You would expect the spinal fluid to exhibit ....

A. encapsulated yeast with narrow-based buds

B. a neutrophil dominant cell count

C. a positive VORL

D. spirochetes

E. xanthochromia

64

..

Note: This material is copyrighted. All rights reserved. (Edward F. Goljan, M.D.) 2001

\

16. A biopsy finding in the cerebellum from a comatose 28 yr old man revealed Purkinje cells with an eosinophilic inclusion. The patient worked in a wild animal park. The vector most likely responsible for this patient's disease is a ...

A. dog

B. bat

C. raccoon

D. mosquito

E. skunk

17. A patient as well as other members of his family has skin findings characterized by axillary freckling and multiple pedunculated, pigmented polypoid lesions. In addition, there are fawn colored oval lesions noted as well. A few of the family members have hypertension and sensorineural hearing loss. This patient most likely has ...

A. tuberous sclerosis

B. the dysplastic nevus syndrome

C. acanthosis nigricans

D. multiple seborrheic keratoses

E. neurofibromatosis

18. A spinal tap in a 28 yr old man with AIDS has numerous yeast forms with a narrow based bud. You would expect which of the following groups of spinal fluid results in this patient? Note: the square represents the normal values for CSF glucose and CSF protein

CSF protein

B C

A On

CSF glucose

19. An autopsy performed on a 29 yr old man reveals a necrotic frontal1obeabscess. Histologic exam reveals numerous wide-angled, non-septate hyphae. Based on the morphology of the pathogen, the patient most likely was ...

A. an alcoholic

B. a cave explorer

C. a missionary in Africa

D. an insulin dependent diabetic

E. a pig fanner who ate raw bacon

20. A 23 yr old man, who died in a car accident, had a large clot in the epidural space overlying the left hemisphere. Which of the following is the most common mechanism responsible for the autopsy finding in this patient?

A. Cerebral ischemia with neuronal degeneration

B. Rupture of a congenital aneurysm

C. Skull fracture with an arterial bleed

D. Skull fracture with a venous bleed

E. Embolism from the left heart

65

-- ,.



i

66

, , , , , , , , , , , ,

t· ,

• • ,

,

• •

• t

• • • • • • • • • • • • • • • • • • • • •

Note: This material is copyrighted. All rights reserved. (Edward F. Goljan, M.D.) 2001

21. An elderly patient in a nursery home fell and hit her head. She died 3 days later in the hospital. At autopsy, a large blood clot overlying the right hemisphere was noted between the dura and arachnoid membranes. Which of the following best explains the autopsy finding in this patient?

A. Cerebral ischemia with neuronal degeneration

B. Rupture of a congenital aneurysm

C. Skull fracture with an arterial bleed D.Skull fracture with a venous bleed E.Embolism from the left heart

22. A S8 yr old man with severe diabetes mellitus died from a stroke. At autopsy, a large blood clot was noted in the area of the putamen and globus pallidus. Which of the following best explains the mechanism for the autopsy fmding in this patient?

A. Cerebral ischemia with neuronal degeneration

B. Rupture ofa congenital aneurysm

C. Arteriovenous malformation

D. Embolism from the left heart

E. Poorly controlled hypertension

\ I

)

23. A 58 yr old man with adult polycystic kidney disease experienced a severe occipital headache and died. At autopsy, the surface of the brain was covered by blood. Which of the following is the most often responsible for initiating the autopsy finding in this patient?

A. Rupture of a congenital aneurysm

B. Internal carotid artery atherosclerosis

C. Arteriovenous malformation

D. Embolism from the left heart

E. Neoplastic process involving astrocytes

24. An autopsy finding from a 35 yr old man with severe frontal headaches revealed a hemorrhagic and necrotic mass that traversed the corpus callosum and involved both sides of the brain. Which of the following best describes the pathogenesis of this patient's brain lesion?

A. Benign neoplasm derived from oligodendrocytes

B. Malignant neoplasm derived from astrocytes

C. Intracerebral hematoma due to hypertension

D. Embolism from the left heart

E. Atherosclerotic stroke

25 e An incidental finding noted at an autopsy performed on a 45 yr old woman was a hard, popcorn shaped lesion attached to the dura. It impinged on the cerebral cortex but did not invade brain tissue. Which of the following best describes the derivation of this brain lesion?

A. Astrocytes

B. Oligodendrocytes

C. Ependymal cells

D. Microglial cells

E. Arachnoid granulations

Note: This material is copyrighted. All rights reserved. (Edward F. Goljan, M.D.) 2001

26. Unilateral papilledema would most likely be associated with which one of the following clinical conditions?

A. Glaucoma

B. Uncal herniation

C. Multiple sclerosis

D.. Alzheimer's disease

E. Hypematremia

27. Which of the following clinical scenarios correlates best with the rapid increase in primary central nervous system malignant lymphomas in the United States?

A. Increase in Epstein-Barr virus infections

B. Increase in human immunodeficiency virus infections

C. Increase in prion-related central nervous system disease

D. Increase in slow virus diseases in immunodeficient patients

E. Increase in radiation therapy of metastatic cancers to the brain

)

28.. One week after an upper respiratory infection, a 25-year-old man develops weakness in the lower legs that is progressing into his upper torso. A spinal tap report indicates increased protein, a normal glucose, a negative gram stain, and 20-30 lymphocytes/mononuclear cells/ul., Which of the following categories of disease best explains the clinical and laboratory fmdings in this patient?

A. Inborn error of metabolism

B. Atherosclerotic disease

C. Demyelinating disease

D. Neoplastic disease

E. Infectious disease

29. A 32-year-old woman complains of recurrent episodes of ringing in the right ear along with a sense of fullness behind the ear drum. She occasionally experiences bouts of dizziness where "the room seems to spin around". On physical exam, the Weber test lateralizes to the left ear and the Rinne test demonstrates that air conduction is longer than bone conduction in both ears.. The tympanic membrane has a normal1ight reflex .. Horizontal nystagmus is present in the right eye .. Which of the following best explains the pathogenesis of this patient's disorder?

A. Right middle ear infection

B. Tumor of the acoustic nerve

C. Increased endolymph in the inner ear

D. Brain tumor in the cerebellopontine angle

E. Demyelinating disease involving the acoustic nerve

30. A few weeks ago, a 65 year old man presented with a painless unilateral loss of vision described as a "curtain suddenly going down and then coming up. n He now presents with a sudden onset of expressive aphasia, and contralateral hemiparesis and sensory loss. CT scans of the brain on admission and after 24 hours reveal no evidence of hemorrhage. Which of the following pathologic processes best explains the pathogenesis of this patient's clinical fmdings in the past and present?

A. Embolism

B. Atherosclerosis

C. Lacunar infarction

D. Intracerebral bleed

E. Subarachnoid hemorrhage

67

Note: This material is copyrighted. All rights reserved. (Edward F. Goljan, M.D.) 2001

31. A 45 yr old wife of a pig farmer develops focal new onset epileptic seizures. Her husband has a habit of eating raw bacon. An MRI of her brain reveals multiple calcified cysts. Which of the following applies to the pig, the husband, and the patient regarding the pathogen responsible for her disease?

Pig Husband Wife (patient)

A. Definitive host Intermediate host Defmitive host
B. Intel mediate host Definitive host Intermediate host
c. Definitive host Intel mediate host Intermediate host
D. Intermediate host Definitive host Definitive host
E. Intermediate host Intermediate host Intermediate host 32. A normotensive 29-year-old man with AIDS is beginning to have significant visual loss in both eyes. His CD4 T helper count is 50 cells/ul., Retinal exam exhibits cotton wool exudates in both eyes. The infectious agent that is responsible for this patient's central nervous system disorder belongs to which of the following subtypes of microbial pathogens?

A. Virus

B. Fungus

C. Bacteria

D. Helminth

E. Sporozoan

)

33. A 28-year-old man, who lives in the Northeast, presents with a sudden onset of drooping and drooling out of both sides of his mouth, inability to close both eyes, and slurred speech. His wife states that a few weeks ago he removed a tick from his right thigh. In a few days following removal of the tick, a concentric, erythematous rash developed in that same area. The infectious agent that is responsible for this patient's previous skin disorder and current neurological findings belongs to which of the following subtypes of microbial pathogens?

A. Virus

B. Fungus

C. Rickettsia

D. Protozoan

E. Spirochete

34. A 2-month-old infant has a progressive increase in head circumference. The pediatrician suspects hydrocephalus and orders an rvIRI. The ~ reveals dilatation of the third ventricle and lateral ventricles. Which of the following is the most likely cause of the patient's condition?

A. Obstruction of the arachnoid granulations

B. Obstruction of the aqueduct ofSylvius

C. Dandy-Walker syndrome

D. Amold-Chiari syndrome

E. Tuberous sclerosis

68

Note: This material is copyrighted. All rights reserved. (Edward F. Goljan, M.D.) 2001

35. A 45 yr old man notices weakening in his ability to open jars with his right hand. He also notes "twitching" in the muscles of his legs and back and an inequality in the size of his calf and thigh muscles, with the muscles on the left much smaller than those on the right. Physical exam reveals atrophy of the intrinsic muscles of both hands, the right greater than the left and absent deep tendon reflexes in the upper extremity and left leg. Muscle fasciculations are noted in the right forearm, left calf, and left thigh. The Babinski sign is negative. Sensory function is intact. Which of the following pathologic processes best explains the pathogenesis of the patient's neurological findings?

A. Triplet repeat disorder

B. Demyelinating disease

C. Degeneration of anterior hom cells D.Metabolic disorder involving copper E.Degenerative changes in the cervical spinal cord

ANSWERS AND DISCUSSIONS TO PATHOLOGY QUESTIONS

Optional General principles in laboratory medicine questions

)

1. Answer: B- mean of the test is 20 mg/dL, 2 SD = 10 mg/dl, therefore 1 SD = 5 mg/dL

2. Answer: D- increasing sensitivity (less FNs) by lowering the reference interval automatically increases the predictive value of a negative test result

3. Answer: C: group A = allTNs, group B = TNs + FNs, group C = FPs + TPs, group D = all TPs

4. Answer: A- estrogen increases the synthesis of thyroid binding globulin. This automatically increases the total T4 level, since TBG has T4 bound to it. However, the free T4 level remains unchanged. A similar question on the USMLE was involved an increase in serum cortisol in a pregnant woman who had no signs of Cushing's syndrome. Estrogen also increases the synthesis of transcortin, the binding protein for cortisol, hence the increase in serum cortisol, but no signs of hypercortisolisrn since free cortisol levels are normal.

Cell Injury questions

1. Answer: E- endocrine stimulation of target tissues is invariably hyperplasia. Choice A is atrophy, choice B is hypertrophy, choice C is metaplasia, choice D is hypertrophy

2. Answer: E- hemorrhagic infarction. Choices A, B, and C are liquefactive necrosis. Choice D is atrophy,

3. Answer: D- choice A has a low Sa02, choices B, C, and E have a normal P02 and Sa02, choice

4. Answer: B- goblet cells in the mainstem bronchus are an example of hyperplasia. If they were in the terminal bronchiole, it would be metaplasia. Choices C and D are hyperplasia. Choice E represents granulomatous inflammation.

Inflammation questions

1. Answer: D- patient with MPO deficiency would have a respiratory burst but would still have a microbicidal problem. In addition, there are no azurophilic granules in the cytoplasm of neutrophils.

2. Answer: B- inhibition of PLA2 leads to a reduction in synthesis of both prostaglandins and leukotrienes.

3. Answer: c- this is a pericardial friction rub

4. Answer: D- note, C. difficile pseudomembranous .colitis has a lot of similarities with diphtheria

69

..

Note: This material is copyrighted. All rights reserved. (Edward F. Goljan, M.D.) 2001

5. Answer: A- cellulitis is usually due to group A strep, since hyaluronidase (spreading factor) is produced and allows the infection to spread through the subcutaneous tissue. Staphylococcus aureus produces coagulase, which changes fibrinogen into fibrin and localizes the infection leading to an abscess with drainage to the surface through a sinus.

6. Answer: D- without adhesion molecules, neutrophils cannot adhere to the endothelial cells in the venules, hence they cannot emigrate into tissue. For the umbilical cord to falloff, it must have neutrophil infiltration of the tissue leading to necrosis.

7. Answer: B

8. Answer: D- causes them to release preformed chemicals like histamine and serotonin

9. Answer: C- recall that eosinophils have crystalline material in their red granules

Fluids and hemodynamics questions including optional acid-base questions

1. Answer: A- due to the decrease in TPR from vasodilatation (histamine, bradykinin, nitric oxide, release of anaphylatoxins)

2. Answer: B- in cirrhosis, there are 2 alterations in Starling's forces- an increase in hydrostatic pressure from portal vein hypertension and a decrease in synthesis of albumin, which decreases oncotic pressure. Choices A and 0 are examples of exudates. Choice C is lymphedema. Choice E is an increase in hydrostatic pressure causing a transudate

3. Answer: D- patients with heart failure and pitting edema have an excess of salt and water, hence these must be restricted.

4. Answer: D- the patient has inappropriate ADH syndrome. Since ADH reabsorbs solute free water from the kidneys, the excess water distributes in both the ECF and ICF compartments. Dilutional hyponatremia favors an osmotic gradient moving water into the ICF compartment. TheRx of choice is to restrict water, since the TBNa is normal.

S. Answer: C- the patient is in hypovolemic shock, hence the cardiac output is decreased, SVR is increased due to catecholamines, vasopressin, and AT n, the MV02 is decreased owing to the reduction in blood flow through the tissue, and the PCWP is decreased, since plasma volume is decreased.

MVOz PCWP SVR Cardiac output

Decreased Decreased Increased Decreased

MV01 PCWP SVR Cardiac output Interpretation

A. Normal Normal Normal Normal Normal

B. Decreased Increased Increased Decreased Left heart failure

D. Increased Decreased Decreased Decreased Septic shock

6. Answer: A- the patient has traveler's diarrhea, which is a secretory diarrhea with a loss of isotonic fluid. Serum sodium is normal, hence POsm is normal and there is no osmotic gradient.

7. Answer: D- the patient has both left (bibasilar rales) and right heart failure (dependent pitting edema). In heart failure, there is an increase in TBNa and TBW, the TBNa is restricted to the ECF compartment leading to pitting edema (ECF compartment is expanded due to the gain in salt), while the excess in TBW is distributed between the ECF and ICF compartments by osmosis. Since there is a greater increase in TBW than TBNa, patients have hyponatremia causing water to move into the ICF compartment (expanded).

8. Answer: E- the patient has central diabetes insipidus due to severance of the pituitary stalk from his motorcycle accident. There is a loss of pure water, therefore the patient develops hypematremia (increased POsm), which stimulates thirst. A pure water loss does not produce any signs of volume depletion, since the TBNa is normal, Since there is a loss of fluid, the ECF compartment is contracted and the osmotic gradient favors movement of water out of the ICF (contracted) into the ECF compartment.

70

..

Note: This material is copyrighted. All rights reserved. (Edward F. Goljan, M.D.) 2001

)

9. Answer: F- the patient has a hypertonic gain of salt from the excessive amounts of sodium bicarbonate received during a cardiac resuscitation. He has signs of both left and right heart failure. A hypertonic gain results in hypematremia and an increase in POsm. The gain in TBNa is primarily in the ECF compartment, so it is expanded. Hypernatremia favors an osmotic gradient with the movement of water from the ICF into the ECF, hence the ICF compartment is contracted.

10. Answer: C- the patient has an isotonic gain of fluid, hence the serum sodium and POsm remain normal. There is no osmotic gradient, so the fluid expands the ECF compartment.

11. Answer: B- the patient has a hypertonic loss of fluid, hence the presence of hyponatremia and a decrease in POsrn. Since the hypertonic fluid is lost in the urine, the ECF compartment is contracted. Hyponatremia favors an osmotic gradient of water moving into the ICF, so it is expanded.

12. Answer: E- sweating results in a hypotonic loss of more water than salt,hence hypematremia and increase in POsm is expected. Since there is a loss of hypotonic fluid, the ECF compartment is contracted. Hypem.atremia favors the movement of water out of the ICF into the ECF compartment, hence it is contracted. Note the difference between the patient in question 8 with diabetes insipidus vs this patient Due to loss of salt in this patient, there are signs of volume depletion, while the patient with COl is not volume depleted, since salt is not lost.

13. Answer: A- in DKA, the excess glucose increases the POsm leading to a gradient favoring the movement of water out of the ICF (contracted) into the ECF compartment. The water moving into the ECF compartment causes a dilutional hyponatremia, hence the serum sodium is decreased. Since the patient is volume depleted, osmotic diuresis related to glucose is causing the loss of a hypotonic salt solution in the urine, hence the ECF compartment is contracted. Note the unusual situation of an increase inPOsm due to hyperglycemia in the presence of hyponatremia.

POsm PNa+ EeF ICF

A. High Low Contracted Contracted

14. Answer: B- platelets are present in both venous thrombi and in arterial thrombi. In venous thrombi they are only one of the components utilized, the others representing coagulation factors like fibrinogen, V, VIII, and prothrombin. In arterial thrombi, platelets are the primary component of the thrombus.

15. Answer: C- the patient has fat embolization. Note the time delay before symptoms occur.

16. Answer: E- metabolic alkalosis

17. Answer:D- mixed: respiratory alkalosis + metabolic acidosis. This could also be a patient with

endotoxic shock

18. Answer: F- respiratory alkalosis

19. Answer: B- metabolic acidosis

20. Answer: C..._ chronic respiratory acidosis. Patient's have COPD.

21. Answer: A- acute respiratory acidosis. Note the effect of compensation on respiratory acidosis when comparing COPD (question 7) with this case. Other ABG answers: A = acute respiratory acidosis, B= metabolic acidosis, C = chronic respiratory acidosis

22. Answer: B- excess aldosterone enhances all the aldosterone pumps leading to hypernatrernia and hypokalemia, and increased secretion of protons into the urine, leading to retention of bicarbonate and metabolic alkalosis

23. Answer: D- destruction of the adrenal cortex leads to deficiency of aldosterone as well as cortisol and 17-ketosteroids. Aldosterone deficiency leads to loss of sodium in the urine (hyponatremia), hyperkalemia (cannot exchange potassium for sodium), and retention of protons (metabolic acidosis).

24. Answer: E- note that the anion gap is 10 mEq/L, which represents a normal AG metabolic acidosis. Adult diarrhea is usually secretory, hence there is an isotonic loss of fluid. Serum sodium levels, therefore, remain normal. There is a considerable amount of potassium (hypokalemia) and

71

Note: This material is copyrighted. All rights reserved. (Edward F. Goljan,M.D.) 2001

bicarbonate in diarrheal fluid. Loss of bicarbonate produces metabolic acidosis. The bicarbonate loss is counterbalanced by an equal gain in chloride ions, which results in a normal anion gap.

25. Answer: F- note that the anion gap is 20 mEq/L, which is an increased AG metabolic acidosis. An increased AG metabolic acidosis means that the anions of an acid have been added to the ECFe.g., lactate, salicylate, formate (methyl alcohol), oxalate (ethylene glycol), acetoacetate/J3- hydroxybutyrate (ketoacidosis), sulfate/phosphate (renal failure).

26. Answer: A- as a rule, any serum sodium < 120 mEq/L is SiADH. Note the dilutional effect on all the solutes

27. Answer: C- note the hypokalemic metabolic alkalosis in either of the conditions

Nutrition questions

)

4.

1. Answer: D- anorexia nervosa. Amenorrhea leaves the patient estrogen deficient and prone to osteoporosis. She should be placed on birth control pills. Note how the stress hormones (cortisol, GH) are increased ..

2.. Answer: A- vitamin A deficiency accounts for the first 2 symptoms, while vitamin K deficiency accounts for the hemorrhagic diathesis. Patients with CF have malabsorption and lack all the fat soluble vitamins.

Answer: B- vitamin D deficiency causes bone pain and tetany,since the bone is not mineralized properly and is soft (osteomalacia) leaving it prone to fractures. Hypocalcemia explains the tetany.

Choice A is scurvy, choice C is pyridoxine deficiency, choice D is thiamine deficiency, choice E is pellagra from niacin deficiency.

Answer: E- all the other findings characterize kwashiorkor, where there is a normal total caloric intake but a deficiency of protein

Answer: B- com contains niacin which is in a bound form that cannot be absorbed. Choice A is pyridoxine deficiency, choice C is B12 deficiency, choice D is a by-product of niacin, choice E-

maldigestion will cause fat soluble vitamin deficiencies A, D, E, K.

6. . Answer: E- due to a lack of estrogen. All the other findings are more commonly seen in bulimia.

7. Answer: C- increased adipose down-regulates insulin receptor synthesis predisposing the patient to type IT OM or glucose intolerance. A wine for free cortisol clearly separates obesity from Cushing's syndrome, the latter having an increase in free levels in the urine.

8. Answer: E- goat's milk is deficient in folate and pyridoxine

9. Answer: .E- pure vegans do not eat meat or dairy products, hence they are susceptible to Bl2 deficiency. Vegans who are pregnant should be placed on B12 supplements in addition to the usual prenatal vitamins containing folate and iron.

10. Answer: c- the patient has scurvy. Vitamin C hydroxylates lysine and proline. This is the binding site for the cross-bridges that strengthen collagen by creating a triple helix. Hence, in scurvy, collagen is weakened.

11. Answer: B- the symptom complex describes acute Wernicke's encephalopathy. Alcohol excess is the MCC of thiamine deficiency. Thiamine is a cofactor for the pyruvate to acetyl CoA reaction using pyruvate dehydrogenase. Giving the patient glucose in an IV causes the rest of the thiamine to be used up, hence precipitating acute Wernicke's encephalopathy. 'Always give N thiamine before hanging up glucose.

3.

5.

Optional genetics questions

1. Answer: B- the patient has Down syndrome based on the low serum a-fetoprotein and signs of duodenal atresia. 46 chromosomes indicates a Robertsonian translocation, where the mother had 45 chromosomes, where the 2 chromosome 215 are fused into 1 chromosome. The chromosome from

72



Note: This material is copyrighted. All rights reserved. (Edward F. Goljan, M.D.) 2001

the mother essentially has 2 chromosome 21 s+ the 1 chromosome from daddy leads to 46 chromosomes, however, there are 3 functional chromosome 21s. Nondisjunction is the cause of trisomy 21.

2. Answer: E- since the woman is homozygous for the sickle gene and the man is normal, all the children will have sickle trait

3. Answer: C- testicular feminization. This is a SXR (male) disorder with absent androgen receptors.

Therefore, fetal testosterone cannot stimulate the development of an epididymis, seminal vesicles, or vas deferens from wolffian duct structures. Similarly, dihydrotestosterone cannot fuse the labia into a scrotum and extend the clitoris into a penis and cannot lead to formation of a prostate gland. Left unstimulated by DHT, the external genitalia remain female in appearance. The lower 2/3rds of the vagina can develop, since it represents the urogenital sinus. All miillerian structures are destroyed by apoptosis, so patients do not have tubes, uterus, cervix, or the upper one-third of the vagina, hence the blind pouch.

4. Answer: D- patient has fragile X syndrome. This is a triplet repeat disorder, hence the disease gets worse in future generations in both the affected males and the female carriers.. Some geneticists, therefore call this a sex-linked dominant condition, since female carriers may express the disease. However, other's consider it a sex-linked recessive disorder, which is what the USlvILE thinks.

5. Answer: D- rnicrodeletion syndrome on chromosome 15. If the chromosome was maternally derived, the patient develops Angelman's syndrome ("happy puppy" syndrome). If the chromosome is paternally derived, the patient develops Prader Willi syndrome.

6. Answer: C: Sickle cell disease is an AR. disease and both parents must carry the abnormal gene,

therefore, 1/12 x 1/12 = 1/144 black couples are at risk. They have a 1/4 risk of having a child with sickle cell disease, therefore, 1/144 x 1/4 = 1/576. Hardy-Weinberg equation is the reverse of this in that the prevalence is given and the carrier rate has to be calculated. To simplify the equation, just work backwards. For example, 1/576 + 1/4 = 1/144 couples at risk. V 1/144 = 1/12.

7. Answer: A- urine is the best body fluid to culture CMV and to look for intranuclear inclusions in the renal tubular cells

8. Answer: C- mother has rubella. Sensorineural hearing loss is the most common abnormality, Saddle nose deformity refers to congenital syphilis, periventricular calcification to CMV, limb hypoplasia to thalidomide, and craniofacial abnormalities to isotretinoin acid used in treating cystic acne.

Neoplasia questions

1. Answers: A, E: adenocarcinoma of the distal esophagus related to Barrett's esophagus has replaced squamous cancer of the mid-esophagus as the Me cancer of the esophagus. Adenocarcinoma has also replaced squamous cell cancer as the Me primary lung cancer.

2. Answer: E- p-53 produces a product that inhibits active cyclin D-dependent kinase, which normally phosphorylates the Rb protein and allows the cell to enter the S phase. This allows the cell time to repair defects in DNA before allowing the cell into the S phase. Cells that are damaged beyond repair are removed by apoptosis.

3. Answer: B- all the other sites listed have metastasis more common than a primary cancer.

4. Answer: E- extranodal metastasis is worse than nodal metastasis. Remember the TNM staging system, which progresses from the least to most important prognostic factor in cancer.

S. Answer: C- since both men and women have colons, it should be no surprise that colon cancer is second in incidence and mortality in both men and women

6. Answer: E- small cell cancers of the lung produce ACTH and ADH

7. Answer: B- these tumor markers are always ordered in testicular cancer.AFP comes from yolk sac tumors (endodermal sinus tumors), while l3-hCG comes from choriocarcinoma.

8. Answer: D- patient's wiIlnot get HBV, HDV, or hepatocellular carcinoma due to HBV 73

Note: This material is copyrighted. All rights reserved. (Edward F. Goljan, M.D.) 2001

Hematology questions

1. Answer: C- menorrhagia gives away the answer as a cause of iron deficiency in young women. A = thalassemia- note the normal iron studies and high RBC count, B = ACD, D = iron overloadnote the low TIBC (recall that excess iron stores decreases transferrin synthesis), E = acute blood

los5- initially, the RBC count, Hgb and Hct are normal, since whole blood is lost Plasma is replaced before RBCs, so the RBC deficit is eventually uncovered, Giving the patient isotonic saline immediately uncovers the RBC deficit and the Hgb, Ret, and RBC count will be low.

2. Answer: E- other differences include achlorhydria (reason for high gastrin levels), autoantibodies against intrinsic factor and parietal cells, correction of Schilling's test with intrinsic factor.

3. Answers: A, D, E- the explanation for A is in question 1. MV02 is decreased because the patient is in hypovolemic shock and the tissue can extract more oxygen from the blood. The PCWP is decreased because plasma volume is decreased .

. 4. Answer: A - Streptococcus pneumoniae sepsis is the Me cause of death in children with sickle cell disease. Hence, the importance ofPneumovax and prophylactic antibiotics.

5. Answer: D- the patient has infectious mononucleosis

6. Answer: c- the patient has PRV. The other answers are: A = COPD or cyanotic CHD, B = ectopic EPO production (e.g., renal adenocarcinoma, squamous cancer of lung), D = relative polycythemia (volume depleted)

7. Answers: A, B, D- the patient has congenital spherocytosis. Increased extravascular hemolysis increases the amount of bilirubin in bile leading to calcium bilirubinate stones and cholecystitis.

8. Answers: B, D, E, G- they have Pb poisoning with peripheral neuropathy and the abdominal colic.

The paint used in pottery often contains Pb. Other scenarios used on boards include eating old paint, working in an automobile factory (incinerate batteries), drinking moonshine (lead lined radiators) .

9. Answers: A, C, D-patient has G6PD deficiency (SXR disease) with hemolysis secondary to dapsone. During acute hemolysis enzyme assays are usually normal, since only the cells containing the enzyme remain behind, while those without the enzyme are hemolyzed.

10. Answers: B, C, D, F- the patient has aortic stenosis and a macroangiopathic intravascular hemolytic anemia with schistocytes. Loss of Hgb in the urine has caused iron deficiency, Thrombocytosis is common in chronic iron deficiency,

11. Answer: A- the patient has sickle cell trait with microinfarctions in the renal medulla. O2 tension is low enough in the medulla to induce sickling.

12. Answers: B, C, D- the patient has 13-thalas semi a minor. Since J3-chain synthesis is decreased, a,chains can combine with o-chains to produce HgbA2 and combine with y-chains to produce HgbF. HgbA is decreased due to the lack of ~-chains.

13. Answers: B, C, E-autoimmune hemolytic anemia. Must correct for the anemia and the polychromasia. Type IT hypersensitivity reaction.

14. Answers: B, C, E- the patient has eLL with hypogammaglobulinemia. It is the Me leukemia and cause of generalized lymphadenopathy after 60 yrs of age

15. Answers: A, B, C: the patient has acute lymphoblastic leukemia

16. Answers: A, D,E- chronic myelogenous leukemia. Blasts do not contain Auer rods (only acute myelogenous leukemia has blasts with Auer rods)

17. Answer: A- early stage of iron deficiency. Sickle cell trait does not have anemia. Recall that all the iron studies (iron, TIBC, % saturation, ferritin) are abnormal before there is any anemia. The anemia is fIrst normocytic and then becomes microcytic.

74

------------

Note: This material is copyrighted. All rights reserved. (Edward F. Goljan, M.D.) 2001

18. Answer: E- think most cornrnon- lack of response to transfusion is most commonly due to a GI bleed. The Hgb should increase by 1 gm/dL for every unit of packed RBCs and the Hct should increase by 30/0.

19. Answers: B, 0- patient has acute progranulocytic leukemia with DIC. A positive TRAP stain is hairy cell leukemia and invasion of the gums is acute monocytic leukemia. A low LAP is chronic granulocytic leukemia.

20. Answers: D, E- remember that iron studies are normal in mild thalassemia (a or B). Increased RDW is characteristic of iron deficiency. Both have a low MCV.

21. Answers: B, C, E- patient with SLE and a warm AIHA, which is an extravascular hemolysis leading to an increase in UeB and jaundice. UeB cannot be filtered in the urine like conjugated bilirubin. The reticulocyte index = 4.5%

22. Answers: A, E- they have no mitochondria and use anaerobic glycolysis for energy

23. Answer: E- decrease adhesion molecule synthesis and destroy lymphocytes and eosinophils

24. Answers: B, C: NOTE- the others are primarily intravascular hemolysis

25. Answers: C, E- only the Hgb concentration is decrease. Gas exchange is normal, so the Pa02 and Sa02 should be normal.

26. Answers: C, D- the patient has hairy cell1eukemia

27. Answer: C- these patients frequently progress into an acute myelogenous leukemia. Small and large RBCs and blasts are common in the peripheral blood. Ringed sideroblasts are also a common feature in the bone marrow,

28. Answers: C, D leukoerythroblastic smear from metastasis to bone. Tumor is pushing normal marrow hematopoietic cells into the peripheral blood.

29. Answers: B, E, G, H:NOTE: among parasites, only invasive helminths produce eosinophilia.

Pinworms do not invade.

30. Answers: A, C, D, E- the first 3 are bacterial infections due to E. coli. Catecholamines and tissue necrosis explain neutrophilic leukocytosis in an AMI

31. Answers: A, C- parvovirus also produces aplastic anemia in patients with an underlying hemolytic

anemia (e.g., sickle cell disease, congenital spherocytosis).

32. Answer: B- most microcytic anemias in children are due to a bleeding Meckel's diverticulum

33. Answer: B- most microcytic anemias in adults over 50 are due to colon cancer

34. Answer: B- somehow, it increases vessel permeability leading to cerebral edema

35. Answers: A, B, C NOTE: the neurologic deficits remain, hence the importance of making the correct diagnosis

36. Answers:A,B Hgb A is never present in sickle cell disease

37. Answer: C- in children without sickle cell disease, Staphylococcus aureus is the MCC of osteomyelitis, not Salmonella

38. Answer: B: the abnormal chromosome has (1) 1.35 kb segment, while the nonna1 chromosome is cleaved into (1) 1.15kb fragment and (1) 0.2 kb fragment. Patient A has sickle cell disease, with (2) uncleaved 1.35 kb segments. patient C is normal, and both 1.35 kb segments have been cleaved into (2) 1.1S kb fragments and (2) 0.2 kb fragments

Lymphoproliferative questions

1. Answers: A, D, E:. the patient .has multiple myeloma. Note the urine findings that screen for BJ protein (disparity between the dipstick for protein and SSA, the latter detecting both albumin and globulins, while the former only detects albumin). Tubular casts in the urine imply that acute tubular necrosis has already occurred.

2. Answers: A, E: an IVP would further exacerbate the patient's renal disease. IEP identifies the abnormal Ig and light chain.

3. Answer: 0: B cell malignancy with a t(8; 14) of the c-myc proto-oncogene 75

Note: This material is copyrighted. All rights reserved. (Edward F. Goljan, M.D.) 2001

4. Answer: B- Sezary syndrome. It is called mycosis fungoides if the cells are not in the blood.

S. Answer: A-Hand-Schuller-Christian disease. CDl is a histiocyte marker.

6. Answer: F- Waldenstrom's macroglobulinemia- an IgM monoclonal spike must have been present.

Multiple myeloma does not usually have an IgM spike.

7. Answer: C: nodular _ sclerosing Hodgkin's disease. This is the classic presentation of nodular sclerosing HD in women or men

8.. Answer: E: these are all features of amyloidosis

Coagulation questions

1. Answer: B- the patient has von Willebrand's disease

2. Answer: D- the patient is most likely taking aspirin or other type of NSAID.The only R.x that could correct this problem would be a platelet transfusion, which is certainly not indicated in this case but would be in a life-threatening bleed.

3. Answers: B, C, E, H: the patient has ole and is in renal failure (BUN/creatinine ratio <15)

4.' Answer: E: in Ole, the most effective R.x 'is toRx the cause of the DIe, in this case, endotoxic shock secondary to E. coli. The blood components are also indicated, but essentially feed the fire and keep the patient alive until the underlying disease is eradicated.

5. Answers: C, E, F- good comparison question

6. Answer: E- rat poison is warfarin, which blocks all the vitamin K-dependent factors. Both PT and PIT are prolonged

7. Answer: E- heparin enhances ATIll, which neutralizes most intrinsic factors and most of the fmal common pathway factors, hence both PT and PI4r are prolonged

8. Answer: B: the patient has Ole and is consuming platelets, fibrinogen, prothrombin, V, vm

9. Answer: A: HUS due to 0157: H7 serotype of E. coli. Remember that only platelets are consumed, not coagulation factors

lO. Answer: D: hemophilia A, maternal father transmits the disease to all his daughter's (SXR. trait)

who are asymptomatic carriers. The daughters transmit the gene to 50% of their sons.

11. Answer: A- the patient has idiopathic thrombocytopenic purpura

12. Answer: A- the patient has thrombotic thrombocytopenic purpura

13. Answer: C: the patient has hereditary A'Im deficiency. Heparin cannot anticoagulate without ATm. The pearl is the lack of increase in the YIT with heparin.

14. Answer: D: heterozygote carriers for protein Cbecome homozygote when given warfarin in -6-8

hrs when the half-life of previously y-carboxylated protein C disappears. Now the patient is byperc oagul able.

Blood Bank questions

1. Answers: C, D, F: the patient has had a delayed HTR due to an antibody directed against an antigen on the donor RBCs.

2. Answers: A, B, C, E: ABO incompatibility does protect against Rh sensitization, but she still should receive Rh immune globulin. Mother's anti A,B IgG is coating the babies RBCs. The Apt test distinguishes fetal from adult Hgb. The baby has swallowed mommies blood and does not have a Glbleed.

3. Answers: C, E: febrile reaction: patient must have been exposed to blood products at some time in her life for antibodies to develop (common in multiparous women who commonly have fetomatemal bleeds during delivery)

4. Answer: A: elderly people lose their isohemagglutinins, hence they may not develop hemolytic TRs even with an ABO mismatch.

76

Note: This material is copyrighted. All rights reserved. (Edwaru L'. V01JcL., ~'fL1J.) .:.., v).

5. Answers: A, D: mother's get sensitized against Rh antigen in their first Rh incompatible pregnancy, but this does not affect that baby but future babies

6. Answers: C, D: allergic reaction

7. Answer: E: the mother is ABO and Rh incompatible with her baby. She has not been previously sensitized to D antigen, so it cannot explain the baby's jaundice, therefore, ABO hemolytic disease of the newborn is the most likely cause.

8. Answers: B, D, F- a major crossmatch involves mixing patient serum with donor RBCs to see if they are compatible.

9. Answer: D: blood group 0 people can only receive blood group O. Remember that the forward type with antisera identifies the blood group, while the back type using test RBCs, identifies the antibody associated with the blood group.

10. Answer: B: she must be AB to have A, B, or AB children. AB people are universal recipients, since they lack antibodies.

11. Answer: A: gastric adenocarcinomas are usually seen in group A patients. It is always best to give

the patient the same ABO group rather than 0 packed RBCs.

12. Answer: C: the patient must be blood group B

13. Answer: A: 1 :300 chance of becoming HIV positive

14. Answer: C: HBV has the greatest viral load in blood of all viruses. You cannot get syphilis from transfused blood.

15. Answer: D: it is almost impossible to find blood that is negative for antibodies against CMV

16. Answer: C: HeV is the MCC of transfusion hepatitis

Cardiovascular questions

,

1. Answer: A- ruptured abdominal aortic aneurysm. Note the rupture triad of left flank pain, hypotension, and pulsatile mass.

2. Answer: B- dissecting aortic aneurysm. Note the type of pain radiation into the back, absent pulse, murmur of aortic regurgitation and widening of the aortic mob.

3. Answer: D- mitral valve prolapse. When the L VEDV is decreased (standing, anxiety) the click and murmur come closer to S 1. When the L VEDV is increased (lying down, clenching fist), the click and murmur come closer to S2.

4. Answer: B- hypertrophic cardiomyopathy- MCC of sudden death in young people. The

asymmetric septum has abnormal conduction bundles.

5. Answer: E- reappearance of CK -MB after 3 days is the definition of reinfarction

6. Answer: A- choice B in LHF, choice C in LHF, choice D inRHF, choice E in RHF)

7. Answer: B- A = VSD, B = ASD, C = PDA, D = Tetralogy, E = transposition. Note that a step up in oxygen saturation in a chamber or vessel usually means a left to right shunt, while a step down in oxygen saturation indicates a right to left shunt (cyanotic CHD)

8. Answer: D- Beck's triad of pericardia! effusion. First step is to do an echocardiogram. If positive, then a pericardiocentesis must be done.

9. Answer: A- aortic stenosis is the Me valvular lesion associated with angina and syncope with exercise. It is also the MCC of microangiopathic hemolytic anemia ..

10. Answer: D- mitral regurgitation in heart failure is due to stretching of the mitral valve ring due to volume overload in the left ventricle

11. Answer: E- tricuspid regurgitation is the Me valvular lesion NDA. It is due to S. aureus.

Remember that all right sided murmurs and abnormal heart sounds increase on inspiration, while the reverse is true for left sided murmurs and abnormal heart sounds.

12. Answer: B- polyarteritis nodosa. An immunocomplex is formed with HBsAg leading to vasculitis.

C-ANCA is seen in Wegener's granulomatosis.

77

.,.".-- ... --_._.

..

Note: This material is copyrighted. All rights reserved. (Edward F. Goljan, M.D.) 2001

13. Answer: E- it is a type ill immunocomplex disease. Most clinicians think that HSP and IgA glomerulonephritis are the same.

Respiratory questions

1. Answer: E- the PFfs describe a restrictive lung disorder, in this case, sarcoidosis. Choice A describes cystic fibrosis, B describes o l-antitrypsin deficiency, C describes COPD, and D describes bronchial asthma

2. Answer: A- these are the classic physical findings of atelectasis. They are similar to consolidations noted in bronchopneumonia and lobar pneumonia. Remember that fever in the first 2448 hrs after surgery is atelectasis.

3. Answer: E- in spontaneous pneumothorax, the lung collapses causing the pressure in the pleural cavity to be the same as that in the atmosphere. A tension pneumothorax is a tear in the pleura, which opens and admits air into the pleural cavity during inspiration and closes during expiration. This increases the positive pressure in the pleural cavity and shifts the mediastinum and trachea to the contralateral side.

4. Answer: C- this is classic RDS. Poor glycemic control in the mother leads to hyperglycemia in the fetus and release of insulin, which inhibits surfactant synthesis by type II pneumocytes.

5. Answer: B- this question compares atypical with typical pneumonia, the former most often due to M. pneumoniae and the later S. pneumoniae. The former is an interstitial pneumonia and the latter a consolidation type of pneumonia. All other choices are those of typical pneumonia and would not be expected in an atypical pneumonia

6. Answer: A- a superior sulcus tumor, usually squamous cancer, invades the lower part of the brachial plexus and destroys the superior cervical ganglion leading to a classic Homer's syndrome.

7. Answer: D- the patient has gram negative sepsis due to E. coli (gram negative rod) which progressed into ARDS. Indwelling urinary catheters are the MCC of this type of sepsis. ARDS and DIe are complications of gram negative sepsis.

8. Answer: B- croup or laryngotracheobronchitis is due to parainfluenza virus. It leads to obstruction in the trachea, which on x-ray appears like a "steeple" of a church. Acute epiglottis due to H. injluenzae also produces inspiratory stridor.

9. Answer: A- RSV is the MCC of pneumonia and bronchiolitis in children. C. trachomatis produces a staccato cough, wheezing with trapping of air, and eosinophilia. There is no fever and bilateral conjunctivitis is usually present as well.

lO. Answer: A- choices B, C, D are features of P. aeruginosa. Both have productive cough (choice E).

11. Answer: B- most solitary nodules in the lungs are granulomas and in the Ohio-Tennessee valley, histoplasmosis is the most likely cause. In the Southwest it 'would be coccidioidomycosis, in the Southeast, blastomycosis, and in the Northeast, cryptococcosis,

12. Answer: B- this underscores the association of rheumatoid nodules in the lungs with a pneumoconiosis, in this case coal worker's pneumoconiosis.

13. Answer: B- the patient has had asbestos exposure, since roofmg material at that time had asbestos in its composition. Whether the patient was a smoker or not, a primary lung cancer would be more common than a mesothelioma. Mesotheliomas take much longer to develop than lung cancer and have no smoking relationship. The Me lung problem associated with asbestos is a benign pleural plaque.

14. Answer- A. Histoplasma capsulatum and Cryptococcus neofonnans. Starlings carry Histoplasma and pigeons carry Cryptococcus and both roost on bridges.

15. Answer: D- exposure to linen, hemp, or cotton products may cause a hypersensitivity reaction in the lung with dyspnea and infiltrates. Patients improve over the weekend and get depressed on Monday knowing that they will be sick, hence the term "Monday morning blues".

78

..

t

t

,

,

I

f



t



c •

f (

t

f

t

(

t

,

f • f

t

c • (

« 4 (

( t C

Note: This material is copyrighted. All rights reserved. (Edward F. Goljan, M.D.) 2001

16. Answer: A.Silo filler's disease is related to nitrogen dioxide is given off by the fermenting com. It causes an immediate reaction in the lungs. Farmer's lung is due to a hypersensitivity reaction against thermophilic actinomycetes out in the fields. Bagassosis is a reaction against sugar cane.

17. Answer- A- the formula to calculate the PA02 is PA02 = % O2 (713) - PaC02/0.8. In this problem the PA02 = 0.21 (713) - 80/0.8 = 50, therefore, A-a = SO - 40 = 10. A medically significant A-a gradient is > 30 mm Hg, Any lung disease would be expected to increase the gradient- e.g., pneumonia, COPD, sarcoidosis, AROS. Hypoxemia due to conditions not directly involving the lungs have a normal gradient. For example, you would expect a normal A-a gradient in paralysis of the diaphragms, barbiturate overdose, and obstructive sleep apnea due to enlarged tonsils.

Gastrointestinal questions

1. Answer: G- this is due to S. aureus producing a toxin which is ingested in the food. Culture the food, not the stool

2. Answer: E- Campylobacter is the MCC of invasive gastroenteritis in the U.S. It can produce crypt abscesses and simulate UC.

3. Answer: F- Yersinia is a cause of mesenteric adenitis, a granulomatous disorder, that can be confused with acute appendicitis.

4. Answer: A- Vibrio cholerae produces a secretory diarrhea due to toxin stimulation of c-AMP.

s. Answer: D- Salmonella typhi produces this classic triad during the second week of infection. A human reservoir is involved.

6. Answer: B- B. cereus produces a preformed toxin. Salmonella enteritidis is the MCC of food

. .

poisoning

7. Answer: D- Cryptosporidium, a sporozoan, is the Me pathogen in AIDS diarrhea

8. Answer: C- it is a secretory diarrhea due to a heat stable toxin that stimulates guanylate cyclase

9. Answer: B- Candida is the overallMC fungus infection in AIDS. Esophagitis is AIDS-defming, but not thrush,

10. Answer: A- irritable bowel syndrome

11. Answer: C- both are associated with H.. pylori. Duodenal ulcers have the greatest association with

H pylori and also have the highest incidence of perforation and an association with ZE in MEN I

12. Answer: B- self explanatory

13. Answer: B- all the other choices characterize CD

14. Answer: C- lactase deficiency. Lactase is a disaccharidase and a brush border enzyme. It produces an osmotic type of diarrhea.

IS. Answer: E- there is also evidence of liver metastasis

16. Answer: E- the patient has the carcinoid syndrome. Appendiceal carcinoids do not metastasize to the liver, so it would not produce the syndrome.

17. Answer: E- the patient has angiodysplasia. There is a relationship with aortic stenosis and von Willebrand's disease. Diverticulosis is the MCC of hematochezia, however, the barium enema is normal.

18. Answer: D- acute diverticulitis presents as a "left-sided appendicitis"

19. Answer: A- note Virchow's node and the history of weight loss. Remember that a postmenopausal woman should not have palpable ovaries. It could by primary ovarian cancer, or as in this case, . metastasis to the ovaries (Krukenberg tumor) from stomach cancer.

20. Answer: E- atrial fibrillation is the Me arrhythmia associated with embolization, in this case, to the superior mesenteric artery. Amylase is present in the small bowel, so do not be fooled into calling this hemorrhagic pancreatitis.

21. Answers: C, D- the patient has achalasia

79



'e 1

Note: This material is copyrighted. All rights reserved. (Edward F. Goljan, M.D.) 2001

, , •

Hepatobiliary/pancreas questions

1. Answer: B- varicella or influenza infections + salicylates may lead to Reye's syndrome

2. Answer: C- A= serologic gap: not infective sinceHBeAg and HBV DNA are absent, B = acute chronic HBV, D = vaccinated: note the difference from someone who has recovered- there is anti-HBc-IgG

3. Answer: A-Rx is metronidazole

4. Answer: D- if the PT had corrected, then the patient has vitamin K deficiency, which, in t. patient, could be due to malabsorption of K from chronic pancreatitis or bile salt deficiency rela: to his cirrhosis

5. Answer: c- the patient has hemochromatosis, an AR disease with increased iron absorption frc the small bowel, The term "bronze diabetes" describes the skin pigmentation andDM. Iron in tiss generates hydroxyl FRs which damage the tissue- cirrhosis, chronic pancreatitis, restricti cardiomyopathy. Serum ferritin is the best screening test. Hepatocellular carcinoma is the ~_. cause of death.

6. Answer: D- CB >500/0 always indicates obstructive jaundice. Choice A (Gilbert's) would have CB <20% (decreased uptake and conjugation), choice B would have a CB 20-50% (problem wi uptake/conjugation and necrosis of bile ducts causing release of CB), choice C (Crigler-Najja would have a CB <20% (deficiency of conjugation enzymes), choice E (ERA) would have a C <20%, due to macrophage destruction ofRBCs and UCB as the end-product ofHgb degradation.

7. Answer: E- note the obstructive signs of light colored stool and palpable GB (Courvoisier's sign)

8. Answers:D, E- the patient has Wilson's disease. Note that total copper is low becaus ceruloplasmin, the binding protein for copper is low. The Kayser-Fleischer ring is described.

9. Answers: B, C, E- the patient has primary biliary cirrhosis, where there is granulomatoi, destruction of bile ducts in the portal triads. IgM levels are increased as well as cholesterol due t obstruction to bile.

10. Answers: A, C, E- gynecomastia is due to estrogen excess and encephalopathy due to an increas

in ammonia and false neurotransmitters

11. Answer: E- birth control pills produce intrahepatic cholestasis

12. Answer: B- similar to question 11, anabolic steroids also produce intrahepatic cholestasis

13. Answer: B- this is the classic history for HCC. Note the increase inAFP.

14. Answer: D- the patient has echinococcosis. The cycle for the parasite is egg to larva to adult. The host with the larva is always the intermediate host (infected sheep, sheepherder), while the hos I with the adults is the definitive host (dog who ate the infected sheep). The sheepherder got the disease from the dog who has the adults that produce eggs. The eggs develop into the larva in the sheepherder, who is the intermediate host. Basque's and Greek's are famous sheepherders.

15. Answers: A, C- always assume that weightlifters, wrestlers, or professional football players are on steroids. Anabolic steroids and estrogen not only produce intrahepatic cholestasis but also produce benign tumors called liver cell adenomas. They have a tendency to rupture and produce intraperitoneal hemorrhage.

16. Answer: B- the patient is an alcoholic (AST> ALT) with acute pancreatitis (pain radiating into back and sentinel loop due to localized ileus).

Renal questions

1. Answer: c- the patient has metastatic prostate cancer. The best first test is a rectal, which would defmitely identify cancer. The other tests can then be ordered.

2. Answer: B- the key is the cryptorchidism. Even the uninvolved testicle is at risk.

80

..

..

Note: This material is copyrighted. All rights reserved. (Edward F. Goljan, M.D.) 2001

3. Answer: B-note the smoking history. Both lung and renal cancer are associated with smoking. In this case, hematuria and the flank mass indicates the kidney as the source. Renal metastasis is uncommon.

4. Answer: B- the patient has acute pyelonephritis (fever, WBC casts, flank pain). This is due to ascending infection and vesicoureteral reflux

5. Answer: B- the patient has Goodpasture's syndrome. Note how patients usually start the disease in the lungs and then the renal disease occurs. Anti-glomerular and pulmonary capillary basement membrane antibodies are responsible. Rapidly progressive crescentic GN is the most common type of renal disease.

6. Answer: B- association of lung cancer with membranous GN, which is the Me type ofGN causing the nephrotic syndrome in adults

7. Answer: C- focal segmental glomerulosclerosis is the Me renal disease in NDA and AIDS. It is a severe type of nephrotic syndrome.

8. Answer: E- pubertal gynecomastia. Recall that gynecomastia is nonnalat birth, puberty, and in old

age. It relates to hyperestrinism ..

9. Answer: A- note the history of hypertension and a unilateral abdominal mass

10. Answer: A- the patient has lipoid nephrosis

11. Answer: D- the patient has ischemic AlN. BUNlcreatinine ratio should be <15/1 (e.g., 8018), UOsm <350 mOsmlkg (no concentrating ability), and the random UNa > 40 mEq/L (tubular dysfunction)

12. Answer: A- APKD is an AD disorder, hence the family history. Mitral valve prolapse is a common association including intracerebral bleeds and subarachnoid hemorrhages from ruptured berry aneurysms.

13. Answer: C- cyclophosphamide is the R.x of choice for WG. It produces both hemorrhagic cystitis and transitional cell carcinoma ..

14. Answer: C- this is Klinefelter's syndrome

Gynecology questions

1. Answer: D- these develop in the lactiferous duct.

2. Answer: A- infiltrating ductal carcinoma is the Me primary breast cancer. Lobular cancer if the Me cancer that is bilateral

3. Answer: C- FCC is the MCC of a mass in the breast in a woman under 50 yrs old. Note that it is painful, while cancer is painless.

4. Answer: F- unlike Paget's disease of the vulvar, Paget's disease of the breast is an extension of a ductal cancer into the epithelium of the nipple.

5. Answer: B-first generation relatives (mother or sister) overrides age

6. Answer: D- fibroadenomas are stromal tumors that compress ducts. They are the MC tumor in

women < 35 yrs old

7. Answer: A- a similar presentation in the first trimester would be a molar pregnancy

8. Answer: D- cystic teratoma

9. Answer: C- in her age bracket, induration in the pouch of Douglas is due to seeding from a primary ovarian cancer. CAl25 is a tumor marker for surface derived ovarian cancers (e.g., serous cystadenocarcinoma) .

10. Answer: D- preeclampsia in the first trimester is a molar pregnancy.

11. Answer: C- cervical cancer is least common owing to detecting dysplasia on a Pap smear and treating it before it develops into cancer

12. Answer: C- cervical cancer infiltrates into the lateral wall of the vagina and blocks the ureterovesical junction causing hydronephrosis/pyelonephritis and death.

81



,

Note: This material is copyrighted. All rights reserved. (Edward F. Goljan, M.D.) 2001



13. Answer: C- this is as classic a presentation as it gets

14. Answer: D- early menarche, late menopause, nulliparity, obesity, type II DM are all increased risks for endometrial cancer, which relates to prolonged exposure to unopposed estrogen

15. Answer: D- early sexual exposure, multiple high risk sex partners, smoking, birth control pills are risk factors

16. Answer: c- the patient has classic endometriosis

17. Answer: C- the patient has Turner's syndrome, the Me genetic cause of primary amenorrhea.

, • ,



t



Endocrine questions

t

1. Answer: C- patient is taking excess thyroid hormone. Note the low 1311 . It would be high in Graves disease.

2. Answer: B- estrogen effect on increasing TBG without altering the free hormone level. Mitral

valve prolapse is an incidental fmding

3. .Answer: E- primary hypothyroidism A = Graves disease, D = Androgen effect

4. Answer: C- renal stone in primary HPTH

5. Answer: A- DiGeorge syndrome, which has absence of the thymus and parathyroid glands from failure of development of the 3rd and 4th pharyngeal pouches. Choice A could also be primary hypoparathyroidism. B = secondary hyperparathyroidism, D = malignancy induced and all other types of hypercalcemia

6. Answer: A- metyrapone blocks 11 hydroxylase, so ACTH and l l-deoxycortisol should be increased if the pituitary and adrenal cortex are normal. In hypopituitarism, neither ACfH or deoxycortisol is increased. In Addison's disease, ACTH is increased, but deoxycortisol is decreased. Choices B and C occur in neither condition. The 17 ketosteroids (DHEA and androstenedione) would be low in both of them. Both would have hyponatremia. In hypopituitarism, the loss of cortisol and thyroid hormone leaves ADH unchecked, therefore there is a mild hyponatremia. Remember that ACTH has no effect on aldosterone release. In Addison's, there is a loss of aldosterone, hence hyponatremia is much more severe. Choice D is only present in Addison's and choice E is only present in panhypopituitarism.

7. Answer: B- when insulin is absent, fatty acids undergo f3-0xidation. Choice A occurs in the presence of insulin and is inhibited in DKA. Choice C occurs when insulin is present in order to synthesize fatty acids or when it is absent (DKA) due to increased f3-oxidation of F As. Choice D occurs when insulin is present and is inhibited in DKA. Choice E occurs when insulin is present. The enzyme is activated in DKA due to glucagon and catecholamines,

8. Answer: A- the patient has a prolactinoma. Prolactin inhibits GnRH, hence the amenorrhea. Since the TSH is normal, primary hypothyroidism cannot be the cause.

9. Answer: D- in pituitary Cushing's, cortisol is suppressed with the high dose dexamethasone test and not in other types of Cushings. All the other choices are the same in all of them.

10. Answer: A- the high C-peptide proves that the patient has an insulinoma. It would be suppressed if the patient was surreptitiously taking human insulin, since it would suppress endogenous insulin release.

11. Answer: A- osmotic damage of Schwarm cells produces a sensorimotor peripheral neuropathy.

Glucose is converted by aldose reductase into sorbitol, which is osmotically active.

12. Answer: B- in patient's with either central or nephrogenic 01, water deprivation results in a low UOsm (ADH cannot concentrate the wine) and a high POsm, which correlates with hypernatremiaWhen vasopressin is given to the patient, those with central DI will have a rise in UOsm >50% from the baseline, while those with nephrogenic DI have less of a response (choice A). Patients with psychogenic polydipsia behave like a normal control, in that they can concentrate urine with water deprivation.

I

I

f



82

______ .. e. _III

" " •





iii

" (II

• •

• .,

., .,

• • • • • • ,.

:

• • •



Note: This material is copyrighted. All rights reserved. (EdwardF. Goljan, M.D.) 2001

13. Answer: A- insulitis is due to autoimmune destruction of the f3-cells. In type IT DM, the islets are fibrosed and often contain amyloid. Choices B and D are associated with type II DM. Choices C and E occur in both types.

14. Answer: A- the child has a craniopharyngioma with optic chiasm compression and hypopituitarism. IGF -1 would be low. Hypokalemia, low urine 17 -OH levels (cortisol and deoxycortisol), low serum TSH, and low serum deoxycortisol after metyrapone would be expected.

15. Answer: B- the patient has pheochromocytoma. The history is classic. Metanephrines are more sensitive than VMA as a screen.

16. Answer: F- the child has a neuroblastoma with metastasis. Neuroblastomas areAPUD tumors.

They derive from neural crest, have neurosecretory granules, and are S 100 antigen positive. Other examples are small cell cancers, carcinoid tumors, and melanomas.

17. Answer: D- gluconeogenesis is the key reaction that produces hyperglycemia in DKA, therefore the alanine to pyruvate to glucose is the best answer

18. Answer: C- the patient has Sheehan's postpartum necrosis. Prolactin is the first hormone to decrease, hence cessation of lactation .

19. Answer: E- these are the symptoms of hypoglycemia from too much insulin. A candy bar will suffice ..

20. Answer: B- the patient has MEN ITa with medullary carcinoma (calcitonin is the tumor marker), hyperparathyroidism, and pheochromocytoma.

21. Answer: A- since the seminiferous tubules have Sertoli cells that synthesize inhibin, which is the negative feedback for FSH, the FSH would be increased. Leydig cells synthesize testosterone, which has a negative feedback with LH, so both of these tests would be normal. The sperm count is low, since they develop in the seminiferous tubules. Choice B is Leydig cell dysfunction, choice C is both seminiferous and Leydig cell dysfunction.

22. Answer: C- primary aldosteronism. Aside from Cushing's syndrome, no other disorder has this

type of electrolyte profile + hypertension. Tetany is due to metabolic alkalosis. An alkaline pH increases the number of negative charges on albumin, hence more of the free ionized calcium can be bound to albumin. The total calcium remains the same, but the ionized calcium is decreased, hence the presence of tetany.

Musculoskeletal and autoimmune questions.

1. Answer: C- the patient has gout involving the big toe (podagra), It is most commonly due to underexcretion rather than overproduction of uric acid. The crystals are negatively birefringent, meaning that they are yellow when parallel to the slow ray of the compensator.

2. Answer: D- joint fusion due to pannus destruction of the articular cartilage is a characteristic of RA. OA is not an inflammatory arthritis. All the other choices are findings in ~A.

3. Answer: B- the patient has classic Lyme disease, due to the spirochete Borrelia burgdorferi). It is transmitted by the Ixodes tick. The rash is erythema chronicum migrans. Rx is doxycycline.

4. Answer: D- the patient has disseminated gonococcemia due to Neisseria gonorrhoeae, a gram negative diplococcus. Some of these patients are deficiency in complement components C5-C9.

5. Answer: D- cat bites are more likely to cause this than dog bites. Penicillin is the Rx of choice.

6. Answer: C- sacroiliitis and male dominance (HLA B27 positive) characterize AS. Pannus and rheumatoid factor (IgM antibodies against IgG) characterize RA.

7. Answer: E- the patient has ankylosing spondylitis and uveitis

8. Answer: D- the patient has ulcerative colitis which in an HLA B27 positive individual is the trigger for producing ankylosing spondylitis.

9. Answer: C- the patient has juvenile rheumatoid arthritis, specifically Still's disease.

83

..

Note: Thi,material is copyrighted. All rights reserved. (Edward F. Goljan, M.D.) 2001

10.

11.

12.

13.

14.

15. 16.

17.

Answer: C- both lose articular cartilage, but for different reasons. In OA, it is worn down by weight and ischemia, while in RA, it is destroyed by pannus. Only RA affects the MCP Ioim,

Subchondral bone cysts are a feature of OA. .

Answer: B- increased destruction of cells leads to increased metabolism of purines into uric acid leading to urate nephropathy. The patient should have been given allopurinol,

Answer: B- the patient has tabes dorsalis and a neuropathic (Charcot's) joint. The patient is unable to sense pain.

Answer: E- the symptom/sign complex characterizes rheumatoid arthritis. It describes associations with pneumoconiosis (Caplan's syndrome), Sjogren's syndrome (dry eyes and dry mouth, due to autoimmune destruction of the lacrimal glands and minor salivary glands), and Felty's syndrome (splenomegaly with neutropenia).

Answer: C- the patient has drug-induced lupus, with hydralazine as the offending agent. Procainamide is another drug.

Answer: C- the patient has SLE with a classic pericardial effusion.

Answer: C- sclerodactyly and dysphagia for solids/liquids is seen in PSS and CREST syndrome. A false positive syphilis serology due to anti-cardiolipin antibodies is seen in SLE as well as LibmanSacks endocarditis.

Answers: C, F, H, 1, J

A. Osteomyelitis

B.

Most common cause in sickle cell disease is Staphylococcus aureus: no, Salmonella species are the MCC

Most common site is the metaphysis. Most common mechanism for spread to bone in children is local extension from a soft tissue abscess: no, the Me spread is hematogenous to the metaphysis.

Orifice of sinus tract is subject to developing squamous cell carcinoma: yes, this is also true for third degree burn sites. Staphylococcus aureusinvolving the vertebral colunm: DO, it refers to TB of the vertebral bodies

Most common late manifestation is arthritis. Bilateral Bell's palsy may occur: yes, and bilateral Bell's palsy is almost pathognomonic of Lyme's disease.

AD disorder with an osteoblastic defect. Too little bone. Pathologic fractures, deafness: no, it is an osteoclastic defect, so there is too much bone (brittle bone disease) and DO bone marrow •

AD disorder with a defect in synthesis of type I collagen. Blue sclera due to underlying choroidal veins. Pathologic fractures: yes, this is a favorite question and slide.

Most common cause in women is estrogen lack. More bone is lost by osteoclastic activity than replaced by osteoblastic activity: yes, the lack of estrogen leaves interleukin-l unchecked, so osteoclastic activity overrides osteoblastic activity reducing overall bone mass.

Most common fractures are vertebral compression fractures:

yes, follewed by Cone's fracture

Routine bone x-rays are the most accurate method for diagnosing the condition: no, it only shows osteopenia. Dual beamabsorptiometry is the diagnostic test of choice. Rx is estrogen, vitamin D, calcium, and stress-types of exercise (walking, weight lifting, not swimming)

Osteomyelitis

c.

-

Chronic osteomyelitis

D.

Pott's disease

F.

Lyme's disease

-

G.

Osteopetrosis

H.

-

Osteogenesis imperfecta

I.

Osteoporosis

-

J.

Osteoporosis

-

K.

Osteoporosis

84

..

Note: This material is copyrighted. All rights reserved. (Edwaru f. volJan, 1v1.1).) i.\.}\) 1

18.

Answers: A, B, D, E, F

A. Benefits of estrogen in

-

Maintains high HDL and low LDL. Increases bone density rather than preventing loss of bone density. Danger of endometrial cancer if progesterone is not added.

Danger of aseptic necrosis from compromise of medial femoral circumflex artery by a posterior dislocation: yes, this is the MCC of aseptic necrosis of the femoral head followed by corticosteroid therapy.

Danger of osteomyelitis. Pain in the anatomical snuff box: no, the danger is aseptic necrosis, but yes, it does produce pain in the anatomic snuffbox.

Dinner fork deformity of the distal radius. Common fracture when falling on the outstretched hand.

Fracture of the distal humerus. Median nerve may be injured. Brachial artery compromised leading to possible ischemic contracture of forearm muscles: this is commonly asked on boards.

Bone x-ray reveals increased density. MRI is the most sensitive test: yes, the increased density is due to reactive bone formation.

Osteomyelitis of the femoral head in children < 10 yrs old: no, it is aseptic necrosis of the femoral head in this age bracket.

Inflammation of tbeproximaI tibial apophysis with repair resulting in knobby knees. Leads to problems with bone growth in the leg: yes, for the former sentence, but no, it does not affect bone growth in the leg like Legg-Perthes disease.

osteoporosis

B. Femoral neck fracture

-

c. Scaphoid bone fracture

D. Colles' fracture

-

E:. Supracondylar fracture

E: Aseptic necrosis

G. Legg-Perthes

H. Osgood-Schlatter's disease

,

19.

Answers:A,B, D, E, G

A: Paget's disease bone

of Bone thickens and fractures easily. Serum alkaline phosphatase is elevated. Complications include osteogenic sarcoma, pathologic fractures, high output cardiac failure: yes, it primarily affects elderly males. Lytic lesions are noted in bone and may be confused with multiple myeloma except they have irregular and fuzzy margins as opposed to myeloma, which has sharp, cookie cutter type margins.

Most common overall benign bone rumor: yes, it is an exostosis of bone that is capped by benign cartilage .

"Round (small) cell tumor" with x-ray findings of "onion skinning". Primarily affects young adults: yes for the type of tumor and the x-ray fmdings, but no, it is primarily a childhood tumor.

Most common malignant cartilaginous tumor. Grade determines it biologic behavior. Most commonly affects pelvic bones. Radiograph demonstrates a radiolucent defect surrounded by

densely sclerotic bone. Nocturnal pain is relieved by aspirin Association with Rb suppressor gene on chromosome 13. "Sunburst appearance" and "Codman's triangle" on x-ray.

Primarily affects the pelvic bones: yes for everything except it

primarily involves the knee area- lower femur or upper tibia 85

,

,

, • ~

t ,

t a a t# ,

t ,

t t

B.

-

Osteochondroma

c.

Ewing's sarcoma

D.

Chondrosarcoma

-

E.

-

Osteoma

F.

Osteogenic sarcoma

-

Most common adult striated muscle sarcoma. Presents as a vaginal mass in women or bladder/prostate mass in men: yes for everything except it is the most common childhood sarcoma.

Most common benign soft tissue tumor in women: yes for men but a leiomyoma of the uterus is the most common soft tissue tumor in women.

Most common benign soft tissue tumor of the uterus and gastrointestinal tract: yes, and in the GI tract it is most common in the stomach, where it is associated with bleeding.

Most common benign tumor of the heart in children with tuberous sclerosis: yes, cardiac myxoma is more common for adults while cardiac rhabdomyomas is more common children

Most common childhood soft tissue sarcoma. Associated with radiation exposure and scars: yes for everything, except it is the most common adult sarcoma followed closely by liposarcoma.

_' - • • • • .' ,

- • - e

e

• e

e 41 ,

• t

C t ,

t ,

4 t e C ,

t t C 4 «

- f

t C 4 ~ t ~

Note: This material is copyrighted. All rights reserved. (Edward F. Goljan, M.D.) 2001

in the metaphyseal area.

Causes include upperllower motor neuron disease, primary muscle disease, myasthenia gravis, and extrapyramidal disease Fibromatosis involving palmar tendon sheaths. Related to hyperestrinism in cirrhosis: yes for its location and classification but no for any relationship with hyperestrinism.

Fibrosarcoma of the abdominal wall often associated with Turcot's syndrome: no, it is a fibromatosis often associated with Gardner'S syndrome

1. Answer: B- ACE is also involved in the metabolism of bradykinin. Hence, an ACE inhibitor leads to an increase in bradykinin (increases vessel permeability), which is the mechanism for angioedema and cough.

2. Answer: A- T. tonsurans invades the inner hair shaft, hence the fluorescent metabolites cannot be detected by Wood's lamp light (UV A light). It is now theMCC of tinea capitis. M. canis used to be the Me cause of tinea capitis. It is Woods lamp positive.

3. Answer: C- procainamide is another drug that is associated with drug-induced SLE. Anti-histone

antibodies are pathognomonic of drug induced lupus.

4. Answer: E- the patient has porphyria cutanea tarda.

5. Answer: E- the patient has acute intermittent porphyria

6. Answer: C- note that the distribution of the rash is in sun-exposed areas, so a drug associated with photosensitivity is likely,

7. Answer: E- the patient has condyloma acuminata, which is due to HPV 6 or II. Topical podophyllin is the Rx of choice. Do not confuse this with condyloma I atum , which is secondary syphilis.

8. Answer: B- the patient has psoriasis. In blacks, the erythema of the lesion is masked, however, the silvery scales are present. Nikolsky sign is noted in pemphigus vulgaris, a bullous lesion with

G.

-

Muscle weakness

H.

Dupuytren's contracture

I.

Desmoid tumor

20.

Answers: C, D

A. Embryonal rhabdomyosarcoma

B. Lipoma

~ Leiomyoma

I!: Rhabdomyomas

E. Malignant fibrous histiocytoma

Skin questions

86

..

------------------------------------------_ .... _-" ...

..

Note: This material is copyrighted. All rights reserved. (Edward F. Goljan, M.D.) 2001

separation above the basal cell layer. Dermatographism is noted in chronic urticaria. Wickham's stria is a fine, lacy leukoplakic lesion on the buccal mucosa in lichen planus.

9. Answer: B- the Breslow system measures depth of invasion. Lesion > 1.7 mm in depth have the capacity to metastasize.

10. Answer: B- the patient has atopic dermatitis, Atopy is a characteristic feature in the family.

11. Answer: D- the patient has a type IV contact dermatitis to nickel, which is similar to the pathogenesis of poison ivy and a positive PPD. Choice A and B are a type I reaction involving histamine. Choice C is type ill immunocomplex disease. Tinea corporis is a superficial dermatophyte infection, most often due to Trichophyton rubrum.

12. Answer: B- the patient has tinea corporis, which is most often due to Trichophyton rubrum.

13. Answer: A- the patient has molluscum contagiosum, which is a DNA poxvirus

14. Answer:B- the patient has actinic (solar) keratosis, a UVB related precursor lesion for squamous cell carcinoma.

15. Answer: B- the patient has tinea versicolor, which is due to Malassezia furfur, as superficial dermatophyte that also causes seborrheic dermatitis (dandruff).

16. Answer: C- the initial rash is called a herald patch. The rash distribution is described as being in a "Christmas tree" distribution. It is best treated with UVB light.

1 7. Answer: B- the patient has chickenpox, which is a DNA Herpesvirus in the same family has HSV, CMV, and EBV. The inclusions of HSV and varicella look the same. Note the different stages of the rash, which is characteristic of varicella.

18. Answer: A- the patient has shingles. The virus is in the sensory ganglia in the thoracic area.

19. Answer: D- the patient had impetigo due to a nephritogenic strain of group A streptococcus which lead to post-streptococcal glomerulonephritis.

20. Answer: D- the patient has acne vulgaris

eNS and special senses questions

1. Answer: D- all of the SIS are due to thiamine deficiency and Wernicke's encephalopathy. Recall that the mamillary bodies and tissue around the ventricles have an orange discoloration due to hemorrhages and subsequent deposition of hemosiderin.

2. Answer: B- the patient has Wilson's disease. Degeneration occurs in the lenticular nuclei. If it occurs in the subthalamic nuclei, hemiballismus may occur.

3. Answer: D- the patient has Creutzfeldt-Jakob disease

4. Answer: D (Neisseria meningitidtsi. If the patient was over 18 yrs old, the answer would be A (Streptococcus pneumoniae)

5. Answer: B- this is the Me primary malignant CNS tumor in children, but a cystic astrocytoma of the cerebellum (benign) is the MC overall primary CNS tumor.

6. Answer: D- the patient has metastatic lung cancer to the brain with uncal herniation and compression of the oculomotor nerve leading to mydriasis, lid-lag, and ophthalmoplegia (eye deviated down and out)

7. Answer: A- the patient has Down syndrome. The extra chromosome 21 codes for more ~-amyloid protein, hence Alzheimer's disease at an early age

8. Answer: A- MS is the MC demyelinating disease. It is an autoimmune destruction of the myelin sheath. Recall that bilateral internuclear ophthalmoplegia is pathognomonic of MS.

9. Answer: E- sensory changes are due to Vlllth nerve tumor in cerebellopontine angle impinging on the Vth eN. There is an association of acoustic neuromas with neurofibromatosis.

10. Answer: C- atrial fibrillation is the Me arrhythmia leading to embolic disease. This is a hemorrhagic infarction (liquefactive necrosis), while atherosclerotic strokes are pale infarctions.

11. Answer: D- this is due to deficiency of dopamine in substantia nigra neurons

87

..

. -' '. -'", .~ - .~ ..

----.--.-----

Note: This material is copyrighted. All rights reserved. (Edward F. Goljan,M.D.) 2001

12. Answer: A- classic question between syringomyelia and ALS. ALS has no sensory changes, while syringomyelia does along with motor changes.

13. Answer: C- toxoplasmosis is the MCC ofa space occupying lesion in the brain in AIDS. Note the low helper cell count. Toxo is found in cat litter and in undercooked meat.

14. Answer: E- it is the second MCC of neonatal meningitis/sepsis, with group B streptococcus (S.

agalactiae) the MC. All the other disorders characterize Pseudomonas

15. Answer: C- the patient has tabes dorsalis (neurosyphilis). Note the Argyll-Robertson pupil.

16. Answer: E- the patient had rabies and the inclusion is a Negri body

17. Answer: E- the patient has neurofibromatosis with neurofibromas and cafe au lait spots.

18. Answer: B- the patient has cryptococcus. CSF protein is elevated and glucose is low.

19. Answer: D- the patient has rhinocerebral mucormycosis

20. Answer: C- the patient had an epidural hematoma

21. Answer: D- the patient had a subdural hematoma with rupture of the bridging veins between the arachnoid and dura

22. Answer: E- the patient had a hypertensive bleed

23. Answer: A- the patient had a subarachnoid hemorrhage from rupture of a congenital berry aneurysm., which developed over time due to hypertension secondary to APKD.

24. Answer: B- the patient has a glioblastoma multifonne, the MC primary brain tumor in adults.

25. Answer: E- the patient had a meningioma, which is the most common benign primary brain tumor in adults. It is a common cause of focal new onset seizures. Psammoma bodies are present in the tumor.

26. Answer: B-papilledema implies an increase in intracranial pressure

27. Answer: B- the virus itself is responsible for the cancer.

28. Answer: C- the patient has Guillain-Barre disease, an autoimmune disease with destruction of myelin in the peripheral nervous system

29. Answer: c- the patient has Meniere's disease with sensorineural hearing loss

30. Answer: B- the patient had an atherosclerotic stroke preceded by amaurosis fugax, which is embolization of atheromatous debris from plaque most commonly located at the bifurcation of the carotid artery

31. Answer: B- the patient has cysticercosis. The patient's husband must have eaten a undercooked pork with larva of Taenia solium in the meat (intermediate host). Adults that laid eggs developed in the farmer (definitive host). The eggs were transferred to the wife and they developed into larvae (cysticerci, wife is the intermediate host) in the GI tract which disseminated to the brain.

32. Answer: A- the patient has CMV retinitis. It should be treated with ganciclovir. If that does not work, foscamet should be tried.

33. Answer: E- the patient has Lyme's disease with bilateral Bell's palsy. Borrelia burgdorferi is a spirochete transmitted by an Ixodes tick. The skin lesion was erythema chronicum migrans.

34. Answer: B- this is an obstructive hydrocephalus.

35. Answer: C- this is classic amyotrophic lateral sclerosis (ALS)

88



~----------------------------------------------------------~------------~

You might also like